INS 3103 Risk Management and Insurance FINAL (Rejda)

Ace your homework & exams now with Quizwiz!

A peril is A) a moral hazard. B) the cause of a loss. C) a condition which increases the chance of a loss. D) the probability that a loss will occur.

b

A peril is a. a moral hazard b. the cause of loss c. a condition which increases the chances of loss d. the probability that loss will occur

b

A situation or circumstance in which a a loss is possible, regardless of whether a loss occurs, is called a a. deductible b. loss exposure c. loss avoidance d. peril

b

ABC Appliance offers a warranty requiring an annual fee. The warranty may be purchased at the time of sale or at any time within the first year after the appliance was purchased. The warranty fee after the date of purchase is twice the time-of-purchase fee. When asked why the fee was higher after the date of purchase, ABCȇs president said,ȱȈBuying a warranty is voluntary. Weȇve noted that those who buy the warranty after the purchase date have a greater need for service.ȈȱCharging the same rate or a lower rate after the date of purchase would expose ABC to what problem that also impacts private insurers? A) excessive premiums B) reduced claims C) bad investments D) adverse selection

d

ABC appliance offers a warranty requiring an annual fee. The warranty may be purchased at the time of sale or at any time within the first year after the appliance was purchased. The warranty fee after the date of purchase is twice the time-of-purchase fee. When asked why the fee was higher after the date of purchase, ABC's president said, "buying a warranty is voluntary. We've noted that those who buy the warranty after the purchase date have a greater need for service." Charging the same rate or a lower rate after the date of purchase would expose ABC to what problem that also impacts private insurers? a. excessive premiums b. reduced claims c. bad investments d. adverse selection

d

Adverse selection occurs A) when an insurance company loses money on its investments. B) when individuals intentionally bring about a loss in order to collect from an insurer. C) when catastrophic losses occur as a result of a natural disaster. D) when applicants with a higher-than-average chance of loss seek insurance at standard rates.

d

Adverse selection occurs a. when an insurance company loses money on its investments b. when individuals intentionally bring about a loss in order to collect from an insurer c. when catastrophic losses occur as a result of a natural disaster d. when applicants with a higher-than-average chance of loss seek insurance at standard rates

d

All of the following are characteristics of the liability risk that most people face EXCEPT A) a lien may be placed on your income and assets to satisfy a legal judgment. B) substantial legal expenses may be incurred defending the claim. C) there is no upper limit on the amount of the loss. D) owning liability insurance eliminates the possibility of being held legally liable.

d

All of the following are characteristics of the liability risk that most people face EXCEPT a. a lien may be placed on your income and assets to satisfy a legal judgement b. substantial legal expenses may be incurred defending the claim c. there is no upper limit on the amount of loss d. owning liability insurance eliminates the possibility of being held legally liable

d

Which of the following types of risks is normally uninsurable by private insurers? a. personal risks b. property risks c. liability risks d. market risks

d

Why is a large number of exposure units generally required before a pure risk is insurable? A) It prevents the insurer from losing money. B) It eliminates intentional losses. C) It minimizes moral hazard. D) It enables the insurer to predict losses more accurately.

d

Why is a large number of exposure units generally required before a pure risk is insurable? a. it prevents the insurer from losing money b. it eliminates intentional losses c. it minimizes moral hazard d. it enables the insurer to predict losses more accurately

d

The employees of an industrial heating and cooling company spent four days installing a large gas furnace in a newly-constructed building. While testing the furnace one week later, an explosion occurred, damaging the building. Damage to the building was covered under the heating and cooling company's general liability insurance. When the heating and cooling company also sought reimbursement for the time its employees spent installing the furnace, the insurer denied coverage and noted a specific applicable exclusion. What exclusion denies coverage for such reimbursement?

damage to the insured's work

All of the following statements about liability coverage under the businessowners policy are true EXCEPT

employees acting within the scope of their employment are excluded from coverage

The phrase "general liability" is used to refer to the legal liability of a business arising from all of the following EXCEPT

employees injured on the job

Tony has a PAP which provides medical payments coverage. Under which of the following circumstances would the injured person be eligible for benefits under Tony's policy? I. A friend in the car is injured while Tony is driving a covered auto. II. A passenger on a motorcycle driven by Tony is injured when Tony hit another vehicle.

I only

Which of the following losses would be covered under a Commercial General Liability Policy that has no endorsements? I. The named insured rented a building and an employee of the named insured negligently started a fire at the rented building. II. The cost to recall defective products

I only

The worst loss that could ever happen to a firm is referred to as the a. maximum possible loss b. probable maximum loss c. frequency of loss d. severity of loss

a

Purpose of Other Provisions

The purpose of these provisions is to prevent profiting from insurance and violation of the principle of indemnity

Acme Company has three identical manufacturing plants, one on the Texas Gulf Coast, one in southern Alabama, and one in Florida. Each plant is valued at $50 million. Acmeȇs risk manager is concerned about the damage which could be caused by a single hurricane. The risk manager believes there is an extremely low probability that a single hurricane could destroy two or all three plants because they are located so far apart. What is the probable maximum loss associated with a single hurricane?

$50 million

11) Which of the following statements about the coverage for other structures (Coverage B) under the Homeowners 3 policy is true? A) The amount of insurance on other structures is equal to a percentage of the coverage on the dwelling. B) Structures attached to the dwelling by a fence or utility line are considered to be part of the dwelling rather than other structures. C) Coverage applies even if the other structure is used for business purposes. D) There is coverage for a private garage only if it is rented to a tenant of the dwelling.

A

11) Which of the following statements about the regulation of insurance company investments is (are) true? I. The purpose of regulating insurance company investments is to prevent insurers from making unsound investments which could threaten their solvency. II. Life insurers can invest an unlimited amount of their assets in common stocks. A) I only B) II only C) both I and II D) neither I nor II

A

15) Which of the following statements about the extra expense coverage form is (are) true? I. It provides coverage for the increased cost that must be paid to continue operations during a period of restoration. II. It provides coverage for lost profits for up to 90 days following a loss. A) I only B) II only C) Both I and II D) Neither I nor II

A

29) Which of the following statements about experience rating is (are) true? I. The insured's past loss experience is used to determine the premium for the next policy period. II. Its use is generally limited to small firms whose actual experience lacks credibility. A) I only B) II only C) both I and II D) neither I nor II

A

3) The basis for current state regulation of insurance is A) the McCarran-Ferguson Act. B) Paul v. Virginia. C) the South-Eastern Underwriters Association case. D) the National Association of Insurance Commissioners.

A

30) Which of the following statements about life insurance cash values is (are) true? I. Cash values are a result of the level premium method of purchasing life insurance. II. The cash value of a policy must always exceed the policy's legal reserve. A) I only B) II only C) both I and II D) neither I nor II

A

37) The multi-car discount is based on the assumption that A) two cars owned by the same person will not be driven as frequently as one car. B) multiple vehicle owners are safer drivers. C) owners of more than one vehicle are more likely to be financially responsible. D) lower liability limits are needed if there are multiple autos insured.

A

37) Which of the following statements is (are) true concerning the home business insurance coverage endorsement? I. It increases the coverage on business personal property at the insured residence from $2,500 to the Coverage C limit. II. It excludes coverage for liability arising out of the home business. A) I only B) II only C) both I and II D) neither I nor II

A

39) All of the following statements about current assumption whole life insurance are true EXCEPT A) It is a form of participating whole life insurance that pays annual dividends. B) An accumulation account is credited with an interest rate based on present market conditions and company experience. C) Under the low-premium version, the premium is subject to change after an initial guaranteed period. D) Under the high-premium version, the premium may vanish after a period of time.

A

46) Which of the following is authority given to the Federal Insurance Office created by the Dodd-Frank Act? A) to represent the federal government in international discussions of insurance regulation B) to license and charter new insurance companies that plan to operate nationally C) to be the primary monitor of insurance company solvency D) to be the primary regulator of all aspects of insurance

A

9) A contingent beneficiary in a life insurance policy has the right to A) receive the policy proceeds if the primary beneficiary dies before the insured. B) share the policy proceeds with the primary beneficiary. C) change the beneficiary designation under specified circumstances. D) exercise policy rights if the insured is incapacitated.

A

9) All of the following situations are excluded from coverage under Section II of the homeowners policy EXCEPT A) the rental of a spare bedroom which is used by the tenant as an office. B) the use of a rented airplane to take a vacation. C) the ownership of a ten-unit apartment house as an investment. D) the performance of professional services by the insured at the residence premises.

A

9) The policyholders' surplus of an insurer is defined as the difference between its A) assets and its liabilities. B) premium income and its expenses. C) reserves and its liabilities. D) assets and its nonadmitted assets.

A

6) Which of the following statements about the elements of negligence is (are) true? I. The negligence of the tortfeasor may arise from a failure to act. II. The damage that results must be in the form of property damage. A) I only B) II only C) both I and II D) neither I nor II

A) I only

8) Which of the following statements about the definition of the insured is (are) true? I. In some cases, a person who is not specifically named may be classified as an insured. II. Under no circumstances can more than one person be named as an insured. A) I only B) II only C) both I and II D) neither I nor II

A) I only

9) Which of the following statements about brokers is (are) true? I. They legally represent the insured rather than the insurance company. II. They are prohibited from being licensed as agents. A) I only B) II only C) both I and II D) neither I nor II

A) I only

5) All of the following statements about the eligibility requirements for group insurance are true EXCEPT A) Most plans cover both full-time and part-time employees. B) An employee must be actively at work on the day the employeeʹs group life insurance becomes effective. C) An employee can sign-up for insurance during an eligibility period without furnishing evidence of insurability. D) One purpose of a probationary period is to lower administrative costs by eliminating coverage on transient workers.

A) Most plans cover both full-time and part-time employees.

30) What is the practical effect of an insurance policy being a conditional contract? A) The insurer can refuse to a pay claim unless the insured has complied with all policy provisions. B) The insured can assign the policy only with the insurer's consent. C) The insurer can sue the insured for failure to pay any premiums. D) The insured gets the benefit of the doubt if a policy contains any ambiguities or uncertainties.

A) The insurer can refuse to a pay claim unless the insured has complied with all policy provisions.

17) Under common law, which of the following persons is most likely to be classified as an invitee? A) a mail carrier B) a social guest C) a door-to-door salesperson D) a solicitor for a charitable organization

A) a mail carrier

*All of the following are covered autos under the liability section of the PAP EXCEPT:*

A) a nonowned van which is driven by the insured on a regular basis

42) Roger owns some farmland that he rents to a tenant. The tenant lives in an old farmhouse on the property and raises crops on the land. Roger is concerned about possible legal liability if the tenant injures someone. Roger requires the tenant to have liability insurance and to add himself to the liability coverage through an endorsement. Under the tenant's liability insurance, Roger is a(n) A) additional insured. B) first-named insured. C) second-named insured. D) other insured.

A) additional insured.

5) The underwriting process begins with the A) agent. B) desk underwriter. C) inspection report. D) acceptance of the application.

A) agent.

Some physicians, hospitals, and health care organizations agree to make medical services available to insureds at discounted prices. Insureds are not required to use these entities, but if they do, care costs are less than if these entities are not used. Such health care entities are called A) preferred providers. B) Health Maintenance Organizations (HMOs). C) Blue Cross/Blue Shield Plans. D) health savings accounts (HSAs).

A) preferred providers.

50) Gwen is in charge of accounting at Integrity Insurance Company. Integrity is a publicly-traded insurer. In describing her job, Gwen said, "There aren't too many businesses where you are required to keep two sets of book." Gwen's comment most likely refers to her company A) preparing accounting statements using statutory and GAAP accounting. B) preparing one set of records for the insurer's managers and another set for the policyholders. C) preparing one set of books using dishonest values and another set using current market values. D) preparing one set of accounting statements considering investment income and another set of accounting statements not considering investment income.

A) preparing accounting statements using statutory and GAAP accounting.

30) A defendant who is only slightly liable may be required to pay the full amount of damages under which of the following? A) the joint and several liability rule B) the collateral source rule C) arbitration D) res ipsa loquitor

A) the joint and several liability rule

15) Julian, age 45, would like to determine how much life insurance to purchase using the human life value approach. He assumes his average annual earnings over the next 20 years will be $40,000. Of this amount, $20,000 is available annually for the support of his family. Julian will generate this income for 20 more years and he believes that 5 percent is the appropriate interest (discount) rate. The present value of one dollar payable for 20 years at a discount rate of 5 percent is $12.46. What is Julian's human life value? A) $184,600 B) $249,200 C) $360,800 D) $400,000

B

15) MedProf Insurance markets medical malpractice insurance. The company's combined ratio in 2012 was 95.4. Its expense ratio was 25.4. What was the company's loss ratio? A) 60.4 B) 70.0 C) 88.2 D) 120.8

B

15) Which of the following property is covered under the personal property coverage (Coverage C) of the Homeowners 3 policy? A) a pet iguana owned by the named insuredʹs spouse B) a bicycle owned by a foster child living with the named insured C) music CDs located in the named insuredʹs automobile D) a stamp collection insured under a separate policy

B

18) Which of the following is an expense for a life insurance company? A) loss reserves B) death benefits paid to a beneficiary C) unrealized capital gains D) realized capital gains

B

2) Which of the following statements about eligibility requirements for homeowners insurance is (are) true? I. A contract can be written on a six-unit apartment complex as long as the tenants are families. II. Separate forms are available for renters and condominium owners. A) I only B) II only C) both I and II D) neither I nor II

B

25) All of the following are implied warranties in ocean marine insurance EXCEPT A) seaworthiness B) clear tile C) no deviation D) legal purpose

B

34) Michelle had four matching end tables in her home. A fire damaged the home, destroying two of the end tables. Michelleʹs home was covered by an unendorsed Homeowners 3 policy. Which of the following is true with regard to the settlement for the end tables in this case? A) Loss to a pair or set is excluded under the policy. B) The insurer will pay the actual cash value of the loss. C) The insurer will pay the replacement cost of the loss. D) If a partial loss to a pair or set occurs, the insurer is liable for replacement of the entire pair or set.

B

35) New Jersey's dollar-a-day auto insurance coverage is limited to A) elderly individuals. B) Medicaid recipients. C) drivers under age 25. D) high-risk drivers.

B

36) A credit-based score that is highly predictive of future claims cost is an individual's A) CLUE score. B) insurance score. C) loss ratio score. D) underwriting score.

B

38) Metro City has six different zip codes. XYZ Insurance Company markets coverages in Metro City. Any applicant who lists one particular zip code is automatically quoted a premium that is twenty percent more than the average premium for applicants from the other five zip codes, even if the loss exposure is identical. Which regulatory objective is not being met given XYZ's premium structure? A) Rates must be adequate. B) Rates must not unfairly discriminate. C) Rates must be responsive. D) Rate must not be excessive.

B

38) One of the "Tips for Buying a Homeowners Policy" is to consider purchasing a personal umbrella policy. What coverage is provided by a personal umbrella policy? A) blanket property coverage for any damage caused by water B) excess liability coverage and coverage for some liability claims excluded by underlying policies C) all-risk, replacement cost, property coverage D) coverage for business and professional liability

B

38) Which of the following statements about a variable universal life insurance policy is (are) true? I. There is a minimum guaranteed interest rate for the cash value. II. The policyowner has a variety of investment options for the investment of premiums. A) I only B) II only C) both I and II D) neither I nor II

B

4) Which homeowners policy is designed for the tenants of rented premises? A) Homeowners 2 B) Homeowners 4 C) Homeowners 6 D) Homeowners 8

B

4) Which of the following statements about the insurer's obligation to provide a legal defense under the personal liability coverage (Coverage E) of the homeowners policy is (are) true? I. The insurer agrees to defend the insured only if the suit is not groundless, false, or fraudulent. II. The insurer's obligation to defend the insured ceases after the amount paid for damages from an occurrence equals the policy limit. A) I only B) II only C) both I and II D) neither I nor II

B

40) A navigation error caused a cargo ship to run around on a reef. The ship, valued at $1.5 million, was carrying $1.5 million in grain and $1.5 in electronic devices. The ship sustained $150,000 in physical damage. If this loss is a "particular average" loss, how will the loss be settled? A) the owners of each of the three interests are responsible for $50,000 B) the owner of the ship is responsible for the entire $150,000 loss C) the two cargo owners are each responsible for $75,000 of the loss D) the owner of the ship is responsible for the first $100,000 of the loss, and the two cargo owners are each responsible for $25,000 of the loss

B

40) Ellen believes the value of the loss to her home is $30,000. The insurer has offered $18,000 to settle the loss. If Ellen and the insurer cannot agree on the value of the loss, which homeowners policy provision specifies how this dispute will be settled? A) insurerʹs option B) appraisal clause C) loss payment clause D) mortgage clause

B

45) The Dodd-Frank Act created a federal body with some limited regulatory authority. For example, the organization can represent the federal government in international negotiations regarding insurance and it can preempt state law where it conflicts with negotiated international agreements. This body is called the A) National Insurance Bureau. B) Federal Office of Insurance. C) Department of International Insurance. D) International Insurance Bureau.

B

45) Which of the following statements about industrial life insurance (also called home service life insurance) policies is true? A) Most policies have a face value exceeding $100,000. B) Most industrial life insurance policies are cash value coverage. C) Industrial life insurance is group term insurance coverage marketed to employers. D) This popular product accounts for over 40 percent of the life insurance sold today.

B

45) Which of the following statements is (are) true regarding how the Homeowners 3 policy handles the peril of collapse? I. Collapse is specifically excluded, and there are no exceptions to the exclusion. II. Collapse that is caused by a Coverage C peril is covered. A) I only B) II only C) both I and II D) neither I nor II

B

46) Michael wants to make sure that life insurance proceeds are available to pay his outstanding mortgage balance if he dies. He purchased a type of life insurance in which the amount of coverage gradually declines, just as his outstanding mortgage balance gradually declines. This type of life insurance is called A) modified life insurance. B) decreasing term insurance. C) re-entry term insurance. D) current assumption whole life.

B

37) An Inter-Ocean Transfer cargo ship was forced to jettison some cargo in heavy seas. The various interests in the voyage at the time the property was jettisoned were If $400,000 worth of iron ore was jettisoned, for how much of this amount would Inter-Ocean Transfer be responsible under general average? A) nothing B) $160,000 C) $200,000 D) $400,000

B) $160,000

45) Steve was involved in an auto accident. Both drivers were partially at fault for the accident. Steve's actual damages were $50,000. He was judged to be 60 percent at fault. If Steve's state has a pure comparative negligence law, how much will Steve collect? A) $0 B) $20,000 C) $30,000 D) $50,000

B) $20,000

31) ABC Company insured its building on a replacement cost basis for $700,000 under a property insurance policy that included an 80 percent coinsurance clause. The building had a replacement cost of $1 million when it sustained a $40,000 loss. How much will ABC Company receive from its insurer, assuming no deductible applies? A) $33,333 B) $35,000 C) $36,000 D) $40,000

B) $35,000

Kristen has an individual medical expense policy with a $1,000 calendar-year deductible, a $5,000 out-of-pocket limit, and a 20 percent coinsurance requirement. Kristen was hospitalized for a surgical procedure in March, her first health care treatment received during the year. The total bill was $20,000. Considering the deductible and coinsurance, how much of this amount must Kristen pay? A) $4,400 B) $4,800 C) $5,000 D) $5,100

B) $4,800

All of the following statements about the tax treatment of Health Savings Accounts (HSAs) are true EXCEPT A) Contributions to a qualified HSA are tax deductible. B) Distributions from a qualified HSA used to fund medical expenses are taxable income. C) Investment income in a qualified HSA accumulates income tax free. D) Distributions from a qualified HSA prior to age 65 for nonmedical purposes are subject to a 10 percent penalty tax.

B) Distributions from a qualified HSA used to fund medical expenses are taxable income.

26) Which of the following statements about ocean marine insurance is true? A) The coverage is narrow and excludes perils of the sea. B) Hull insurance includes collision liability covering the ship's owner if the ship collides with another ship. C) Protection and indemnity (P&I) insurance covers the shipper of goods for cargo losses. D) A particular average loss is a loss incurred only if a ship is totally lost or destroyed.

B) Hull insurance includes collision liability covering the ship's owner if the ship collides with another ship.

*Which of the following persons is (are) insured under the uninsured motorists coverage of the PAP?* I. A pedestrian struck by a covered auto if he or she has no insurance to pay medical expenses II. The spouse of a named insured who is killed by an uninsured motorist

B) II only

*Which of the following situations would be covered under the liability section of the PAP?* I. A mechanic is sued by a pedestrian who is injured when the mechanic has an accident while road testing the insured's auto. II. The daughter of the named insured is sued after she has an accident when new friend she just met at a campus hangout lets her drive his car.

B) II only

*Which of the following statements about the Miscellaneous-Type Vehicle Endorsement to the PAP is (are) true?* I. It provides bodily injury liability coverage for any vehicle rented by the insured. II. To lower premiums when a motorcycle is insured, bodily injury to passengers can be excluded.

B) II only

1) Fundamental purposes of the principle of indemnity include which of the following? I. To reduce physical hazards II. To prevent the insured from profiting from insurance A) I only B) II only C) both I and II D) neither I nor II

B) II only

10) Which of the following statements about comparative negligence laws is (are) true? I. Under the pure rule, any negligence by the plaintiff automatically bars recovery for damages. II. Under the 50 percent rule, parties who are equally at fault are each allowed to recover damages. A) I only B) II only C) both I and II D) neither I nor II

B) II only

11) Which of the following statements is (are) true concerning the value reporting form? I. Failure to report accurately suspends coverage. II. If the insured underreports the property values at a location, and a loss occurs at that location, recovery is limited to the proportion that the last value reported bears to the value that should have been reported. A) I only B) II only C) both I and II D) neither I nor II

B) II only

15) Which of the following statements about supplemental group major medical insurance is (are) true? I. Most plans have a lifetime limit of $50,000 to $100,000. II. Most plans contain an out-of-pocket maximum that places a dollar limit on the maximum amount an individual must pay out of his or her own pocket. A) I only B) II only C) both I and II D) neither I nor II

B) II only

17) Which of the following is (are) characteristics of HMO managed care plans? I. Unlimited choice of physicians and hospitals II. Emphasis on controlling the cost of covered services A) I only B) II only C) both I and II D) neither I nor II

B) II only

2) Which of the following items is (are) contained in the common policy conditions page of the commercial package policy? I. The policy period II. A provision describing the insurer's right to audit the insured's books and records A) I only B) II only C) both I and II D) neither I nor II

B) II only

36) Which of the following statements about cafeteria plans is (are) true? I. Unspent flexible spending account balances are refunded to the employee, tax -free, at year-end. II. Cafeteria plans enable employees to select benefits that meet their specific needs. A) I only B) II only C) both I and II D) neither I nor II

B) II only

37) Which of the following statements is (are) true about savings bank life insurance (SBLI)? I. Each depositor at the savings bank receives life insurance equal to his or her savings account balance. II. The goal of SBLI is to provide low-cost life insurance to consumers. A) I only B) II only C) both I and II D) neither I nor II

B) II only

37) Which of the following statements is (are) true with respect to catastrophe bonds? I. The bonds are issued by the U.S. Government. II. The bonds have relatively high interest (coupon) rates. A) I only B) II only C) both I and II D) neither I nor II

B) II only

Which of the following statements about health savings accounts (HSAs) is true? A) There are no limits to annual contributions that an individual may make to his or her HSA. B) Once an individual has reached age 65 or is covered by Medicare, no additional contributions to the HSA may be made. C) The health insurance plan covering the HSA account beneficiary is not permitted to use a deductible. D) HSAs offer no tax benefits for the account beneficiary.

B) Once an individual has reached age 65 or is covered by Medicare, no additional contributions to the HSA may be made.

26) Why are insurance contracts said to be contracts of adhesion? A) The values exchanged by the parties to the contract are not equal. B) One party writes the contract, and the other party must accept the entire contract as written. C) Only one party makes a legally enforceable promise. D) Conditions are placed on the insurer's promise to perform.

B) One party writes the contract, and the other party must accept the entire contract as written.

14) All of the following requirements must be met to satisfy the doctrine of res ipsa loquitur EXCEPT A) The injured party has not contributed to the accident in any way. B) The injured party must prove negligence on the part of the defendant. C) The event is one that normally does not occur in the absence of negligence. D) The defendant has exclusive control over the instrumentality causing the accident.

B) The injured party must prove negligence on the part of the defendant.

17) The unearned premium reserve of an insurer is A) an asset representing the investments made with premium income. B) a liability representing the unearned portion of gross premiums on outstanding policies. C) a liability representing claims that have been filed, but not yet paid. D) the portion of the insurer's net worth belonging to policyowners.

B) a liability representing the unearned portion of gross premiums on outstanding policies.

2) What information is contained in the insuring agreement of an insurance policy? A) a description of the property or life to be insured B) a summary of the major promises of the insurer C) a summary of the obligations of the insured D) a list of the property, losses, and perils that are not covered

B) a summary of the major promises of the insurer

12) A common situation involving strict liability includes which of the following? A) operating a motor vehicle B) blasting operations C) manufacturing a product D) mining operations

B) blasting operations

32) All of the following are categories of torts EXCEPT A) intentional torts. B) breach of contract. C) strict liability. D) negligence.

B) breach of contract.

43) Nancyʹs employer provides an interesting employee benefit plan. Each employee is given 250 employee benefit credits to spend. A wide array of benefits are available, and the employee uses benefit credits to select the benefits that he or she wants. This type of employee benefit plan is called a(n) A) defined benefit plan. B) cafeteria plan. C) employee selection plan. D) contributory plan.

B) cafeteria plan.

7) The policy provision requiring the filing of proof of loss with the insurer is an example of a(n) A) declaration. B) condition. C) insuring agreement. D) miscellaneous provision.

B) condition.

31) RST Insurance is an interesting company. It doesn't have any agents. Instead, the company sells insurance through radio ads, telemarketers, and newspaper and magazine inserts. This distribution method is called A) reciprocal exchange. B) direct response system. C) mass merchandising. D) multiple distribution system

B) direct response system.

26) All of the following are proposed solutions to the medical malpractice problem EXCEPT A) shortening the statute of limitations for filing lawsuits. B) eliminating arbitration panels to resolve disputes. C) placing limitations on contingent fees charged by attorneys. D) placing limitations on damage awards.

B) eliminating arbitration panels to resolve disputes.

29) All of the following are examples of tort reform proposals EXCEPT A) modifying the collateral source rule. B) eliminating caps on noneconomic damages. C) regulation of attorney fees. D) imposing penalties to deter frivolous lawsuits.

B) eliminating caps on noneconomic damages.

The Affordable Care Act has provisions that improve the quality of health care and provide several advantages. All of the following are examples of these provisions EXCEPT A) reducing paperwork and administrative expenses. B) increasing the number of physician specialists and reducing the number of general practitioners. C) compensating physicians based on value rather than service volume. D) encouraging integrated health care systems.

B) increasing the number of physician specialists and reducing the number of general practitioners.

2) Insurers obtain data which can be used to determine rates from A) pricing pools. B) insurance advisory organizations. C) banks. D) reciprocal exchanges.

B) insurance advisory organizations.

43) Maria's property was damaged by an earthquake. Maria is unsure whether earthquake is covered by her property insurance. Which two sections of the property insurance policy should Maria review to determine whether earthquake damage to her property is covered? A) exclusions and declarations B) insuring agreement and exclusions C) definitions and insuring agreement D) conditions and declarations

B) insuring agreement and exclusions

19) Which of the following is characteristic of a typical mass merchandising plan? A) higher commission scales for agents and higher administrative expenses B) payment of premiums through payroll deduction C) group rather than individual underwriting D) contributions by the employer to the cost of coverage

B) payment of premiums through payroll deduction

44) Peggy is an independent insurance agent who places substantial amounts of business with XYZ Insurance Company. XYZ pays Peggy a bonus when she meets a sales goal. XYZ also allows Peggy to recruit and train sub-agents. She receives additional commissions based on the sales generated by the sub-agents. Based on this description, we can conclude that Peggy is a A) direct writer. B) personal producing general agent. C) multiple line exclusive agent. D) broker.

B) personal producing general agent.

39) A special coverage policy is a policy that A) has no exclusions. B) provides open-perils coverage. C) provides replacement cost coverage. D) has coverage for multiple lines of insurance.

B) provides open-perils coverage.

48) Granite Insurance Company entered into a treaty reinsurance agreement with Rock Solid Reinsurance (RSR). Granite's retention limit is $400,000 and RSR agreed to provide reinsurance for up to $2.0 million. If Granite writes an $800,000 policy, RSR is responsible for 50 percent of the losses. If Granite insures a $1.6 million risk, RSR is responsible for 25 percent of any losses. What type of reinsurance arrangement did Granite enter into with RSR? A) facultative reinsurance B) surplus share reinsurance C) quota share reinsurance D) excess of loss reinsurance

B) surplus share reinsurance

3) Under one method of estimating a loss reserve, the reserve is based on life expectancy, duration of disability, and similar factors. This method of estimating loss reserves is called the A) judgment method. B) tabular value method. C) loss ratio method. D) average value method.

B) tabular value method.

The Affordable Care Act includes a provision designed to help small employers make health insurance coverage available to their employees. This provision allows small employers to directly reduce their federal income tax by a percentage of the employer's contribution to funding health care for employees. This subsidy, in the form of reduction of income taxes, is called a A) marginal tax rate. B) tax credit. C) tax bracket. D) tax deduction.

B) tax credit.

5) Under which of the following rules is actual cash value determined by taking into consideration all relevant factors an expert would use to determine the value of the property? A) the circumstantial evidence rule B) the broad evidence rule C) the property indemnity rule D) the objective value rule

B) the broad evidence rule

*All of the following are covered as supplementary payments under the liability section of the PAP EXCEPT*

B) the cost of a bail bond for a traffic violation when no accident is involved.

*Which of the following statements about the uninsured motorists coverage of the PAP is true?*

B) the coverage applies only if the uninsured motorist is legally liable.

19) What is the intent of the family purpose doctrine? A) to impose liability on children for the care of elderly parents B) to impose liability on the owner of an automobile for the negligence of immediate family members operating the automobile C) to impose liability on a parent for any negligence caused by a child D) to impose strict liability on the owner of a wild animal for any injuries caused by the animal

B) to impose liability on the owner of an automobile for the negligence of immediate family members operating the automobile

13) All of the following are reasons why employers self-insure medical expense plans EXCEPT A) to reduce certain costs, such as premium taxes and commissions. B) to provide mandated state benefits. C) to retain funds until needed to pay claims. D) to eliminate the need to comply with separate state laws.

B) to provide mandated state benefits.

5) Exclusions are used in insurance policies for all of the following reasons EXCEPT A) to reduce moral hazard. B) to waive policy conditions. C) to eliminate coverage for uninsurable perils. D) to eliminate coverage not needed by typical insureds.

B) to waive policy conditions.

1) LMN Mutual Insurance Company has total liabilities of $300 million. The company has total assets of $380 million. What is LMN's policyholders' surplus? A) $680 million B) $340 million C) $80 million D) -$80 million

C

14) The Homeowners 3 policy limits the amount of coverage provided on certain personal property (e.g. coin collections and silverware). This personal property can be adequately insured by A) lowering the deductible which applies to personal property losses. B) storing the property away from the home at another location. C) adding a schedule to the policy listing the property with specific amounts of insurance. D) adding an appraisal clause to the policy.

C

14) The building and personal property coverage form provides several optional coverages. Under one optional coverage, no deduction is taken for depreciation when settling a covered loss. This optional coverage is called A) extra expense B) agreed value C) replacement cost D) inflation cost

C

15) Which of the following statements about the additional coverages included in Section II of the homeowners policy is true? A) Damage to property of others is covered only if the insured is legally liable. B) Damage to property arising out of a business engaged in by the insured is covered. C) Interest that accrues on a judgment after the judgment is awarded but before the judgment is paid is covered under claims expenses. D) First aid expenses incurred by the insured for a bodily injury covered under the policy are not covered.

C

16) To protect policyholders, state laws place limitations on a life insurance company's investments. The assets backing interest-sensitive products, such as variable life insurance and variable annuities, are not subject to these restrictions. Assets backing interest-sensitive products are placed in a special account called the life insurer's A) policy reserve account. B) policy loan account. C) separate account. D) policyholders surplus.

C

24) Which of the following is (are) included among the duties of an insured following a loss covered under the Homeowners 3 policy? I. To file a proof of loss within a specified time period after the insurerʹs request. II. To prepare an inventory of damaged personal property. A) I only B) II only C) both I and II D) neither I nor II

C

25) All of the following losses are covered under Section II of an unendorsed Homeowners 3 policy EXCEPT A) The homeowner accidentally dropped a bowling ball, injuring another bowler's foot. B) A baby sitter slipped and fell at the insured's home, breaking her ankle and incurring medical expenses. C) The insured slandered a city council member at a city council meeting. D) The insured's dog bit a neighbor.

C

25) All of the following statements about class rating are true EXCEPT A) Exposures with similar characteristics are placed in the same underwriting class. B) The rate charged for each class reflects the average loss experience for that class. C) The complexity of class rating makes it inappropriate for personal lines coverages. D) It is based on the assumption that future losses to insureds will be determined by the same classification factors currently in use.

C

29) The net amount at risk for an ordinary life insurance policy is the difference between the A) present value of future benefits and the present value of future premiums. B) face amount of the policy and the total premiums that have been paid. C) face amount of the policy and the legal reserve. D) annual premium and the annual policyholder dividend.

C

32) Cal is renting an apartment. Which homeowners policy is specially designed to insure renters? A) HO-2 B) HO-3 C) HO-4 D) HO-6

C

33) The inventory at Frank's business fluctuates. Frank's property insurance requires periodic reporting of inventory values. Frank believes he can save money by under-reporting the inventory. Last period, Frank reported $200,000 when he value was really $400,000. Shortly after filing report, when the value was $500,000, the inventory was destroyed. How much will Frank's insurer pay, assuming no deductible? A) nothing, as underreporting voids coverage B) $200,000 C) $250,000 D)$400,000

C

40) All of the following statements about insurance regulation are true EXCEPT A) Insurance commissioners are appointed in some states and elected in some states. B) Insurers are subject to regulation by certain federal agencies and laws. C) The National Association of Insurance Commissioners (NAIC) can force states to adopt the model laws that it drafts. D) An insurance commissioner can revoke or suspend an insurer's license to do business in his or her state.

C

Which of the following statements about the medical payments coverage of the PAP is true?

Covered expenses include the cost of funeral services.

48) Tamara purchased a term insurance policy when she had high life insurance needs and limited income. Now Tamara can afford whole life insurance. What term life insurance provision will permit Tamara to switch her term insurance to whole life insurance without having to show that she is still insurable? A) renewal provision B) tax-free exchange provision C) conversion provision D) free look provision

C

50) Ann is considering the purchase of a life insurance policy with these characteristics: flexible premium payments, the insurance and savings components are separate, the interest rate credited to the savings is tied to a market interest rate but a minimum rate is guaranteed, and a monthly administrative fee is charged. Ann is considering buying A) whole life insurance. B) variable life insurance. C) universal life insurance. D) current assumption whole life.

C

8) What is the effect of the optional agreed value provision in the building and personal property coverage form? A) It provides replacement cost coverage B) It increases insured values along with inflation C) It suspends the coinsurance clause D) It reduces the deductible to $50

C

9) Which of the following items would not appear in the income section of an insurance company's income and expense statement? A) gain on sale of securities B) common stock dividends received C) commissions D) premiums

C

33) Frank's property insurance requires periodic reporting of inventory values. Frank believes he can save money by under-reporting the inventory. Last period, Frank reported $200,000 when the value was really $400,000. Shortly after filing the report, when the value was $500,000, the inventory was destroyed. How much will Frank's insurer pay, assuming no deductible? A) nothing, as underreporting voids coverage B) $200,000 C) $250,000 D) $400,000

C) $250,000

37) Malcolm was involved in an auto accident. He was judged to be 20 percent at fault in the accident, and the other party was judged to be 80 percent at fault. Malcolm's actual damages were $40,000. Under a pure comparative negligence rule, how much will Malcolm receive for his injuries? A) $8,000 B) $24,000 C) $32,000 D) $40,000

C) $32,000

32) XYZ Company insured its building on a replacement cost basis for $450,000 under a property insurance policy that included an 80 percent coinsurance clause. The building had a replacement cost of $500,000 when it sustained a $50,000 loss. How much will XYZ Company receive from its insurer, assuming no deductible applies? A) $42,500 B) $45,000 C) $50,000 D) $56,250

C) $50,000

33) Laura's major medical insurance policy includes a $500 deductible. Laura is required to pay 20 percent of covered expenses in excess of the deductible, and her insurer will pay 80 percent of covered expenses in excess of the deductible. Laura was hospitalized and her covered medical expenses were $10,500. How much of the $10,500 will be paid by the insurer? A) $7,500 B) $7,900 C) $8,000 D) $10,000

C) $8,000

1) LMN Mutual Insurance Company has total liabilities of $300 million. The company has total assets of $380 million. What is LMN's policyholders' surplus? A) $680 million B) $340 million C) $80 million D) -$80 million

C) $80 million

26) Under the Health Insurance Portability and Accountability Act, what is the maximum length of a preexisting conditions exclusion if an employee enrolls when initially eligible? A) 60 days B) 6 months C) 1 year D) 2 years

C) 1 year

11) JKL Insurance Company reported the following information on its accounting statements last year: Premiums Written $90,000,000 Loss Adjustment Expenses $5,000,000 Underwriting Expenses $30,000,000 Premiums Earned $100,000,000 Incurred Losses $70,000,000 What was JKL's expense ratio last year? A) 5.0 percent B) 30.0 percent C) 33.3 percent D) 50.0 percent

C) 33.3 percent

28) All of the following statements about inland marine forms are true EXCEPT A) A mail coverage form covers securities in transit by first-class mail, registered or certified mail, or express mail. B) A commercial articles coverage form is used to insure photographic equipment and musical instruments. C) A jewelers block coverage form is used by individuals to insure jewelry limited in coverage by the homeowners form. D) The signs coverage form covers neon, mechanical, and electrical signs.

C) A jewelers block coverage form is used by individuals to insure jewelry limited in coverage by the homeowners form.

All of the following statements about optional disability income benefits are true EXCEPT A) Under a cost-of-living rider, benefits are periodically adjusted for inflation. B) A Social Security rider pays additional benefits if the insured is turned down for Social Security disability benefits. C) Adding a return of premium rider results in a lower initial premium. D) Under an option to purchase additional insurance, the insured has the right to buy additional insurance at specified times without evidence of insurability.

C) Adding a return of premium rider results in a lower initial premium.

34) James purchased liability insurance with a $100,000 limit from Insurer A. When Insurer A denied a claim that James thought should be covered, he bought a second liability insurance policy with a $150,000 limit from Insurer B. Before he cancelled the policy with Insurer A, a $60,000 loss occurred. If this loss is settled on a pro rata basis, how much must each insurer pay? A) Insurer A will pay $10,000 and Insurer B will pay $50,000. B) Insurer A will pay $20,000 and Insurer B will pay $40,000. C) Insurer A will pay $24,000 and Insurer B will pay $36,000. D) Insurer A will pay $40,000 and Insurer B will pay $20,000.

C) Insurer A will pay $24,000 and Insurer B will pay $36,000.

20) All of the following statements about regulatory objectives of insurance rate making are true EXCEPT A) One purpose of rate adequacy is to maintain the solvency of insurers. B) Rates unfairly discriminate if loss exposures that are similar with respect to losses and expenses are charged substantially different rates. C) Insurers know in advance if the coverages marketed will be profitable, so rate regulation is not needed. D) Rates are excessive if policyholders are paying substantially more than the actual value of their protection.

C) Insurers know in advance if the coverages marketed will be profitable, so rate regulation is not needed.

31) Monopoly Insurance is the only company marketing a certain line of insurance in a state. After complaints from several consumers, the State Insurance Department investigated Monopoly's rates. The regulators determined that Monopoly was taking advantage of being the only insurer offering the line by charging more than double the actuarial cost of the coverage. Which regulatory rating objective was Monopoly violating? A) Rates must be adequate. B) Rates should encourage loss control. C) Rates must not be excessive. D) Rates must not unfairly discriminate.

C) Rates must not be excessive.

*John occasionally borrows the car of his friend, Sophie. Sophie has a PAP with liability limits of 100/300/50. John also has a PAP, and his liability limited 250/500/50. John had an accident while using Sophie's car and was found to be legally liable for $300,000 in bodily injury liability. How much will be paid by each policy?*

C) Sophie's police will pay $100,000, John's policy will pay $200,000

35) All of the following are reasons why mutual insurance companies convert to stock insurance companies EXCEPT A) Stock companies can offer stock options to attract and retain key personnel. B) Stock companies can raise new capital more easily. C) Stock companies are exempt from state insurance regulation. D) Stock companies offer greater flexibility to expand through acquisitions.

C) Stock companies are exempt from state insurance regulation.

1) Which of the following statements about stock insurers is true? A) They issue assessable policies. B) They are not permitted to write property and liability insurance. C) Stockholders bear any losses and share in any profits. D) They are owned by their policyholders.

C) Stockholders bear any losses and share in any profits.

14) All of the following statements about the settlement of a claim are true EXCEPT A) The insurance policy usually has a provision specifying how a notice of loss is to be made to the insurance company. B) One step in the investigation of a claim is to determine whether the policy was in force when the loss occurred. C) The adjustor must file the proof of loss, which is a sworn statement supporting his or her decision regarding a claim. D) A policy provision may determine how disputes over claim settlements are resolved.

C) The adjustor must file the proof of loss, which is a sworn statement supporting his or her decision regarding a claim.

25) All of the following statements about class rating are true EXCEPT A) Exposures with similar characteristics are placed in the same underwriting class. B) The rate charged for each class reflects the average loss experience for that class. C) The complexity of class rating makes it inappropriate for personal lines coverages. D) It is based on the assumption that future losses to insureds will be determined by the same classification factors currently in use.

C) The complexity of class rating makes it inappropriate for personal lines coverages.

15) Which of the following statements about subrogation is true? A) It is used primarily for losses paid under life insurance policies. B) It allows the insurer to sue its own insured who is negligent. C) The insured's right to collect benefits may be forfeited if the insured interferes with the insurer's subrogation rights after a loss occurs. D) The insurer is required to exercise its subrogation rights.

C) The insured's right to collect benefits may be forfeited if the insured interferes with the insurer's subrogation rights after a loss occurs.

*Rob purchased a PAP with collision and other-than-collision coverage. All of the following losses are excluded under this physical damage coverage EXCEPT*

C) a flash flood washed Rob's car off the road and damaged it

32) Renee is risk manager of XYZ Company. She purchased a Commercial Package Policy for her company and added one of the optional coverages. This option suspends the coinsurance clause and substitutes a new agreement covering any loss in the same proportion that the limit of insurance purchased bears to a value specified in the declarations. This provision is known as A) waiver of inventory coverage. B) inflation guard coverage. C) agreed value coverage. D) replacement cost coverage.

C) agreed value coverage.

28) Why can an insurer refuse to pay a claim if an insured fails to abide by the policy provisions? A) because insurance contracts are aleatory B) because insurance contracts are unilateral C) because insurance contracts are conditional D) because insurance contracts are contracts of adhesion

C) because insurance contracts are conditional

18) Which of the following statements about problems arising from the use of a coinsurance clause is (are) true? I. The amount of insurance should be periodically evaluated to avoid a coinsurance penalty because of inflation. II. An agreed value coverage option is one method used to solve the problem of values that fluctuate throughout the policy term. A) I only B) II only C) both I and II D) neither I nor II

C) both I and II

20) Which of the following statements about the equipment breakdown protection coverage form is (are) true? I. The covered cause of loss is a breakdown of covered equipment, including boilers, machinery, and electrical and mechanical equipment. II. It provides coverage for the reasonable cost of expediting permanent repair or replacement of damaged property. A) I only B) II only C) both I and II D) neither I nor II

C) both I and II

21) Which of the following statements about preferred provider organization (PPO) health plans is (are) true? I. A PPO plan contracts with health care providers to provide medical services to members at reduced fees. II. Plan members are given a financial incentive to use PPO providers rather than other providers. A) I only B) II only C) both I and II D) neither I nor II

C) both I and II

49) Which of the following statements about managed care plans is (are) true? I. Even though managed care plans emphasize cost control, total health benefit costs continue to increase. II. Many physicians are critical of managed care plans because of restrictions placed upon their freedom to treat patients. A) I only B) II only C) both I and II D) neither I nor II

C) both I and II

6) Reasons why a peril may be considered uninsurable and therefore excluded from insurance contracts include which of the following? I. The losses from the occurrence of the peril may be due to a predictable decline in value. II. The losses from the occurrence of the peril may be incalculable and catastrophic. A) I only B) II only C) both I and II D) neither I nor II

C) both I and II

38) Pac-Coast Insurance (PCI) concentrates its underwriting activities in California. The company is concerned that if a catastrophic earthquake occurs, it might threaten the solvency of the company. To address this risk, PCI issued some debt securities. If a catastrophic earthquake occurs, PCI does not have to repay the full amount borrowed or pay interest. The securities PCI issued are called A) catastrophe futures contracts. B) interest rate swaps. C) catastrophe bonds. D) contingent options contracts.

C) catastrophe bonds.

9) Which of the following items would not appear in the income section of an insurance company's income and expense statement? A) gain on sale of securities B) common stock dividends received C) commissions D) premiums

C) commissions

*A car damaged in an auto accident may have reduced market value or resale value after it is repaired. Some insureds have sought to recover this reduction in market or resale value. This loss in value is called:*

C) diminution

22) Huge Insurance Company is a property insurer that is interested in protecting itself against cumulative losses that exceed $200 million during the year. This protection can best be obtained using a(n) A) quota-share reinsurance treaty. B) surplus-share reinsurance treaty. C) excess-of-loss reinsurance treaty. D) reinsurance pool.

C) excess-of-loss reinsurance treaty.

42) Med Profs is a group of 18 doctors. These doctors work out of their own offices and treat patients on a fee-for-service basis. In addition, Med Profs doctors also agree to treat HMO members at reduced fees. The type of HMO that uses organizations like Med Profs is called a(n) A) group model plan. B) closed panel plan. C) individual practice association plan. D) network model plan.

C) individual practice association plan.

*All of the following are exclusions under the medical payments coverage of the PAP EXCEPT*

C) injuries incurred in an auto while it is used in a share-the-expense car pool

41) Trisha was injured when the delivery truck for a local furniture store struck her. The delivery driver claimed the brakes of the delivery truck failed, causing the accident. Trisha filed suit, and in her lawsuit named the delivery driver, the furniture store, the service station responsible for vehicle maintenance, and the manufacturer of the vehicle. Even though the manufacturer of the vehicle may be only 1 percent responsible for the accident, it may be required to pay a large percentage of the damages under the A) collateral source rule. B) assumption of risk rule. C) joint and several liability rule. D) last clear chance rule.

C) joint and several liability rule.

44) In determining insurance limits and deductibles, an important concept is that insurance should be used to pay big losses rather than small losses. The objective is to insure big losses that could cause financial ruin and to exclude small losses that can be budgeted out of current income. This concept is called the A) law of large numbers. B) efficient loss-cost concept. C) large-loss principle. D) retention-transfer tradeoff.

C) large-loss principle.

*Ken purchased a PAP with liability limits of 100/300/50, medical payments coverage, and collision coverage. Ken fell asleep while driving late at night. He crossed the center line and hit a car approaching from the other direction. The following losses occurred: -The driver of the other car suffered $30,000 in bodily injuries. -Ken's car sustained $5,000 in damages -Ken incurred $5,000 in medical expenses -The car that Ken hit was a total loss.* Which of Ken's PAP coverages will cover Ken's medical expenses?

C) medical payments coverage

24) ABC Insurance has always used the exclusive agency system to market coverages. ABC, however, cannot afford full-time agents in sparsely-populated areas. To reach customers in these areas, ABC enters into agreements with local independent agents. Using more than one marketing system is called employing a A) direct response system. B) general agency system. C) multiple distribution system. D) branch office system.

C) multiple distribution system.

37) An Econodeath Insurance Company actuary calculated the present value of the expected death claim the company will pay if it sells whole life insurance to a 30-year-old woman. This value is called the A) net level premium. B) gross premium. C) net single premium. D) life insurance policy reserve.

C) net single premium.

45) An insurance policy provision that specifies how a property loss will be settled if more than one property insurance policy covers the loss is the A) insuring agreement provision. B) loss settlement provision. C) other insurance provision. D) coinsurance provision.

C) other insurance provision.

45) The price per unit of insurance is called the A) premium. B) loss adjustment expense. C) rate. D) loss reserve.

C) rate.

Jan needed health insurance. She met with an agent who described the provisions of a health insurance policy. Jan purchased the policy. When she received the policy, she noted that several provisions were different from the provisions the agent described. She was not satisfied with the policy and immediately sent it back to the agent with a note stating her reasons for returning the policy. Jan is guaranteed a premium refund because of which policy provision? A) waiting period B) time limit on certain defenses C) ten-day right to examine the policy D) waiver-of-premium provision

C) ten-day right to examine the policy

*Which of the following is considered to be a collision loss under Part D (coverage for damage to your auto) of the PAP?*

C) the covered auto is damaged when it slid off an icy road and hit a fence.

*All of the following are insured persons under the liability coverage of the PAP EXCEPT:*

C) the former spouse of the named insured who moved out of the home 2 years ago when the divorce was finalized

1) All of the following are contained in the common policy declarations page of a commercial package policy EXCEPT A) a description of the insured property B) the policy period C) the premium paid D) the policy cancellation provisions

D

16) Jessica is an agent for LMN Life Insurance Company. She met with Brad, who was interested in purchasing life insurance. Jessica explained the various uses of life insurance, including income for Brad's wife during the 1- or 2-year period following Brad's death. This period is known as the A) dependency period. B) estate clearance period. C) blackout period. D) readjustment period.

D

16) XYZ, Inc. would suffer serious financial consequences if either of its two major customers were shut down and could not purchase XYZ products. Which of the following types of consequential loss protection would provide protection against this exposure? A) leasehold interest B) extra expense C) marine insurance D) business income from dependent properties

D

17) Which of the following statements about premium taxes is (are) true? I. They are levied by the federal government as a result of the McCarran-Ferguson Act. II. Their primary purpose is to provide funds for insurance regulation. A) I only B) II only C) both I and II D) neither I nor II

D

22) Which of the following statements about yearly renewable term insurance is (are) true? I. It requires evidence of insurability for renewal. II. It is most appropriate when an insured needs lifetime protection. A) I only B) II only C) both I and II D) neither I nor II

D

22) Which of the following types of water damage is covered under an unendorsed Homeowners 3 policy? A) damage from flood B) damage from water backing up through a drain C) damage from water below the surface of the ground that seeps into a basement D) damage from water that overflows from a malfunctioning washing machine

D

23) Which of the following statements about the personal property replacement cost endorsement used with the homeowners policy is (are) true? I. The damaged or destroyed property must be repaired or replaced, no matter the size of the loss. II. It is designed primarily for antiques and fine art. A) I only B) II only C) both I and II D) neither I nor II

D

24) A few states have dual automobile insurance systems. A motorist can pay a higher premium and retain the right to sue under the tort system, or pay a lower premium and be covered under the state's no-fault law. This dual system is called a(n) A) pure no-fault plan. B) modified no-fault plan. C) "no pay, no play" law. D) choice no-fault plan.

D

28) A legal reserve in life insurance is a result of A) premium taxes payable by life insurance companies being postponed during the early policy years. B) dividends being paid to policyowners. C) inadequate premiums in the early policy years being subsidized by investment earnings. D) excess premiums in the early policy years being invested at compound interest.

D

28) Gail lives in southern Illinois, near the New Madrid Fault Line. She added an earthquake endorsement to her Homeowners 3 policy. All of the following statements about the endorsement are true EXCEPT A) The deductible is expressed as a percentage of the coverage limit rather than as a dollar amount. B) The endorsement also covers landslides and volcanic eruptions. C) Aftershocks occurring within 72 hours of an earthquake are considered part of the same occurrence. D) The endorsement also covers flood losses.

D

28) Under the Homeowners 3 policy, all of the following are options of the insurer for settling claims EXCEPT A) paying the claim in cash. B) replacing the property. C) repairing the property. D) paying the claim with insurance company stock.

D

28) Which of the following statements about the waiver-of-premium provision in life insurance is true? A) Because the probability of becoming disabled exceeds the probability of premature death, the cost to include this provision is usually prohibitive at younger ages. B) Premiums are usually waived if the insured becomes partially disabled. C) Life insurance protection continues in force during a period of disability, but dividends cease and cash values are reduced. D) The disability must occur before a stated age, such as 65, for premiums to be waived.

D

33) Shauna purchased a condominium unit. Which homeowners policy is specially designed for condominium unit owners? A) HO-2 B) HO-3 C) HO-4 D) HO-6

D

41) A strip-mall includes eight identical-sized retail units. All of the units were built at the same time and each has an identical sprinkler system. Unit number two is a dry cleaning business. Unit number three is a bar and grill. Unit number four is a dress shop. The owners of these three units are all insured by the same insurance company, but the property insurance premiums vary significantly. Which of the following rating factors best explains the difference in premiums? A) exposure B) protection C) construction D) occupancy

D

41) Ted purchased a home. To fund the purchase, he borrowed $140,000 from ABC Bank, pledging the home as collateral. Shortly after purchasing the home, Ted lost his job. He could not find another job and could not pay the mortgage each month. Ted set fire to the home. The claims adjuster suspected arson, and an investigation proved that Ted intentionally caused the loss. Under the mortgage clause of the Homeowners 3 policy, how will this loss be settled? A) The insurer has no liability because the loss was intentional. B) The insurer will pay Ted the actual cash value of the loss as intentional loss is not excluded. C) The insurer will pay ABC the value of its insurable interest and pay Ted the value of his insurable interest. D) The insurer will pay ABC the value of its insurable interest and then attempt to recoup the loss payment from Ted.

D

43) Jose and Maria would like ʺall-risksʺ coverage on their home and their personal property. Which unendorsed homeowners form provides this coverage? A) Homeowners 2 B) Homeowners 3 C) Homeowners 4 D) Homeowners 5

D

45) One liability on a property and liability insurance company's balance sheet is for the costs associated with settling and paying reserved claims. This liability is the A) pre-paid expense reserve. B) loss reserve. C) unearned premium reserve. D) loss adjustment expense reserve.

D

51) A life insurance policyholder may no longer need life insurance. Such a policyholder may sell the policy to a third party for more than its cash value. The purchaser becomes the new beneficiary and is responsible for subsequent premium payments. Such a financial transaction is called a(n) A) collateral assignment. B) accelerated death benefits rider. C) absolute assignment. D) life settlement.

D

56) Which of the following statements regarding the accidental death benefit rider (also known as double indemnity) is true? A) Adding the accidental death benefit rider doubles the premium for the policy. B) Financial planners agree that adding the accidental death benefit rider is a wise purchase. C) The economic value of a human life is doubled or tripled if death is caused by an accident, justifying the purchase of the rider. D) The death benefit is doubled only if an accidental injury is the direct cause of death and death occurs prior to a specified age.

D

6) Which of the following statements about the Homeowners 3 policy is (are) true? I. Losses to the dwelling are always settled on an actual cash value basis. II. Coverage on personal property is limited to $5,000. A) I only B) II only C) both I and II D) neither I nor II

D

6) Which of the following statements about the provisions of the building and personal property coverage form is (are) true? I. A separate deductible must be satisfied for each building damaged in the same occurrence. II. Under the replacement cost option, there is an automatic 10 percent deduction for depreciation. A) I only B) II only C) Both I and II D) Neither I nor II

D

26) Eric's property was damaged in an accident. He phoned his agent to see if the loss was covered under his property insurance policy. The agent said, "As long as the cause of loss is not specifically excluded in the policy, the loss is covered." Based on the agent's answer, what type of insuring agreement appears in the policy? A) unconditional coverage B) named-perils coverage C) extended-perils coverage D) "open-perils" coverage

D) "open-perils" coverage

35) XYZ Insurance Company uses class rating to determine the rate to charge for insurance. For one type of insurance, the pure premium XYZ actuaries calculated is $75 per unit. If XYZ's expense ratio is 25 percent, what is the gross rate for this coverage? A) $37.50 B) $55.25 C) $75.00 D) $100.00

D) $100.00

Dirk suffered a heart attack and was rushed to the hospital where heart surgery was performed. His total bill for medical services was $50,000. Dirk has a major medical policy with a $1,000 calendar-year deductible and a $5,000 out-of-pocket limit. His coinsurance percentage is 20 percent. The out-of-pocket limit applies to coinsurance only. Assuming this hospitalization was the first medical care that Dirk received during the year and that all of the hospital services were eligible for coverage under the policy, how much of the $50,000 bill will the insurer pay? A) $39,000 B) $39,200 C) $40,000 D) $44,000

D) $44,000

34) XYZ Insurance Company expects $500,000 in claims and loss adjustment expenses for each 1,000 properties that it insures in a certain category of business insurance. What pure premium should XYZ charge for each property insured? A) $69.99 B) $166.67 C) $350.00 D) $500.00

D) $500.00

10) All of the following statements about group universal life insurance are true EXCEPT A) Interest rates credited to a policy vary over time but are subject to a minimum guarantee. B) Premiums can be varied as long as the cash value is sufficient to pay current mortality and expense charges. C) Coverage is issued on a guaranteed basis up to certain limits with no evidence of insurability. D) At retirement, an employee must begin liquidating his or her cash value in the form of an annuity.

D) At retirement, an employee must begin liquidating his or her cash value in the form of an annuity.

A controversial provision of the Affordable Care Act is the expansion of a public assistance program designed to make health coverage available to low-income individuals. By increasing the maximum income level that can be earned and still qualify for benefits, millions of individuals will be eligible for coverage under this public assistance program. This public assistance program is called A) Medicare. B) Health Maintenance Organization. C) Affordable Health Exchange. D) Medicaid.

D) Medicaid

ll of the following statements about individual disability income policies are true EXCEPT A) Premiums are often waived while a person is disabled but must be resumed if the insured recovers. B) At the time of purchase, the insured can choose the length of the benefit period from among several available options. C) In order to encourage rehabilitation, benefits may be continued during periods of vocational training. D) Most disability income insurance policies contain an elimination period of 10 or fewer days.

D) Most disability income insurance policies contain an elimination period of 10 or fewer days.

3) Which of the following statements about group insurance underwriting principles is true? A) Employees should be required to remit premiums directly to the insurance company. B) The average age of the group should ideally increase over time. C) A group should be formed for the specific purpose of obtaining insurance. D) The employer should ideally share in the cost of a group insurance plan.

D) The employer should ideally share in the cost of a group insurance plan.

*Joyce was injured by an uninsured drunk driver while she was riding in a friend's car. Joyce and her friend each have a PAP with an uninsured motorist's limit of $50,000. How much will be paid by each policy if it is determined that Joyce has $70,000 of bodily injuries?*

D) The friend's policy will pay $50,000, and Joyce's policy will pay $20,000.

48) All of the following statements about subrogation are true EXCEPT A) The general rule allows the insurer to recover up to the amount paid to its insured under the policy. B) Subrogation does not apply in life insurance. C) Interfering with the insurer's subrogation rights can jeopardize indemnification of the insured. D) The insurer reserves the right to subrogate against its own insureds.

D) The insurer reserves the right to subrogate against its own insureds.

15) Which of the following statements about reinsurance is true? A) A reinsurer may not purchase reinsurance. B) The reinsurer is responsible for providing claims services to the insured after a loss occurs. C) The amount of insurance transferred to a reinsurer is called the net retention. D) The insurer transferring business to a reinsurer is called the ceding company.

D) The insurer transferring business to a reinsurer is called the ceding company.

23) All of the following statements about life insurance company investments are true EXCEPT A) Funds for these investments are derived primarily from premium income, investment earnings, and maturing investments that must be reinvested. B) Income from these investments reduces the cost of insurance. C) A primary objective in making these investments is safety of principal. D) The majority of these investments are short-term investments.

D) The majority of these investments are short-term investments.

53) Bob purchased insurance on his home with an insurer that was not licensed to do business in the state. In this case, which requirement to form a binding insurance contract is lacking? A) exchange of consideration B) offer and acceptance C) legal purpose D) competent parties

D) competent parties

27) Janet hit a wall causing a large dent in the fender of her car. She was busy at work and delayed reporting the damage to her insurer for 9 months. The insurer denied the claim, stating, "Although such a loss is usually covered, you are required under the terms of the contract to provide prompt notification in case of loss." The prompt notification requirement is an example of a(n) A) declaration. B) definition. C) insuring agreement. D) condition.

D) condition.

4) The exclusion of flood in a homeowners policy is an example of an A) excluded activity. B) excluded condition. C) excluded property. D) excluded peril.

D) excluded peril.

40) In schedule rating, each building is individually evaluated based on several rating factors. One factor refers to the possibility that the building will be damaged or destroyed by a fire that starts at an adjacent property and spreads to the building. This rating factor is known as A) occupancy. B) protection. C) maintenance. D) exposure.

D) exposure.

38) Tom opened a store in a mall. His store is located between a theater and a department store. Tom counts on the theater and department store to generate walk-in business at his store. While his store has been successful, Tom knows that if either or both of the other businesses closed, his store would suffer an economic loss. What type of dependent property situation is illustrated in this scenario? A) contributing location B) recipient location C) manufacturing location D) leader location

D) leader location

45) One liability on a property and liability insurance company's balance sheet is for the costs associated with settling and paying reserved claims. This liability is the A) pre-paid expense reserve. B) loss reserve. C) unearned premium reserve. D) loss adjustment expense reserve.

D) loss adjustment expense reserve.

19) A false material statement made by an applicant for insurance is an example of A) concealment. B) breach of warranty. C) lack of offer and acceptance. D) misrepresentation.

D) misrepresentation.

4) Failure to exercise the degree of care required by law to protect others from harm is called A) premeditated liability. B) vicarious liability. C) punitive damages. D) negligence.

D) negligence.

11) Which of the following statements about claim settlement is (are) true? I. The fair payment of claims requires an insurer to adopt a very liberal claims policy. II. To prevent lawsuits, an insurer should provide no personal assistance to a claimant other than that which is required by contractual obligations. A) I only B) II only C) both I and II D) neither I nor II

D) neither I nor II

13) Which of the following statements about a calendar-year deductible is (are) true? I. It requires the insured to pay a specified amount of each claim regardless of when the claim occurs during the year and regardless of any previous claims during the year. II. It is used only in policies which cover direct property losses. A) I only B) II only C) both I and II D) neither I nor II

D) neither I nor II

*Which of the following statements is (are) true with respect to the collision damage waiver on rented cars?* I. It is inexpensive and is provided at no charge by most rental car companies. II. It waives the renter's liability for bodily injury liability arising out of use of the rented auto.

D) neither I or II

*Which statement concerning towing and labor coverage under the PAP is (are) true?* I. If the insured purchases collision coverage, towing and labor coverage is provided automatically. II. Towing and labor coverage pays for repairs at a service station or garage.

D) neither I or II

*All of the following statements about Part D (coverage for damage to your auto) of the PAP are true EXCEPT*

D) no coverage is provided for newly-acquired vehicles

44) Marv is covered by a group health insurance plan at work. His employer funds the entire cost of the group health insurance. Because of this characteristic, the group health insurance plan can be described as A) defined benefit. B) contributory. C) defined contribution. D) noncontributory.

D) noncontributory.

41) A strip-mall includes eight identical-sized retail units. All of the units were built at the same time and each has an identical sprinkler system. Unit number two is a dry cleaning business. Unit number three is a bar and grill. Unit number four is a dress shop. The owners of these three units are all insured by the same insurance company, but the property insurance premiums vary significantly. Which of the following rating factors best explains the difference in premiums? A) exposure B) protection C) construction D) occupancy

D) occupancy

47) A legal wrong for which the law allows a remedy in the form of money damages is a A) crime. B) breach. C) misdemeanor. D) tort.

D) tort.

45) Some states have a law that requires payment of the face amount of insurance to the insured if a total loss to real property occurs from a peril specified in the law. These laws are called A) agreed amount laws. B) replacement cost laws. C) homestead laws. D) valued policy laws.

D) valued policy laws.

33) The voluntary relinquishment of a legal right is called A) subrogation. B) adhesion. C) estoppel. D) waiver.

D) waiver.

Purpose of Deductible

Eliminate small claims that are expensive to handle and process Reduce premiums paid by the insured Reduce moral hazard and attitudinal (morale) hazard

Which of the following statements about self-insurance is (are) true? I. It is a form of planned retention. II. State law usually prohibits its use for workers compensation.

I only

Which of the following statements about the Medicare prescription drug benefit is (are) true? I. Beneficiaries select a prescription drug plan and pay monthly premiums, with the premium reduced or waived for low-income beneficiaries. II. The benefit provides first-dollar coverage of prescription drugs up to a maximum of $10,000 per year, but no coverage after $10,000.

I only

Which of the following statements about the financial status of Medicare Part A is (are) true? I. Part A has serious financial problems. II. Reforms instituted by Congress have reduced total Part A expenditures in recent years.

I only

Which of the following statements about a personal risk management program is (are) true? I. Insurance and retention are the only techniques used to handle potential losses. II. The steps in a personal risk management process are the same steps used by businesses.

II only

Which of the following statements about captive insurance companies is (are) true? I. A captive insurance company established by a U.S. company must be domiciled in the United States. II. A captive insurance company may be owned by several parents.

II only

Which of the following situations would be covered by the liability section of an unendorsed PAP if the insured is legally liable?

The insured backs into and damages the garage door of his rented house.

All of the following statements about captive insurers are true EXCEPT

They increase the volatility of the parent companyȇs earnings.

Methods by which insurers may minimize or avoid catastrophic losses include which of the following 1. The use of reinsurance 2. Concentrating coverage written in one geographic region a. 1 only b. 2 only c. both 1 and 2 d. neither 1 or 2

a

The appraisal provision in the Personal Auto Policy is used to determine the

amount paid for a physical damage loss to the insured's auto if the insured and insurer disagree.

A risk manager is concerned with I. Identifying potential losses. II. Selecting the appropriate techniques for treating losses.

both I and II

Factors a risk manager must consider in selecting an insurer include which of the following? I. The availability of risk management services. II. The financial strength of the insurer

both I and II

Preloss objectives of risk management include which of the following? I. Preparing for potential losses in the most economical way. II. Reduction of anxiety.

both I and II

Which of the following is (are) covered under an unendorsed Commercial General Liability Policy? I. False arrest of a customer wrongly accused of shoplifting II. Publication of material that violates an individual's right to privacy

both I and II

Endorsement

is a written provision that adds to, deletes from, or modifies the provisions in the original contract

1) Which of the following types of families is likely to have the least need for a large amount of life insurance? A) a blended family B) a traditional family C) a single person family D) a sandwiched family

c

Ken purchased a PAP with liability limits of 100/300/50, medical payments coverage, and collision coverage. Ken fell asleep while driving late at night. He crossed the center line and hit a car approaching from the other direction. The following losses occurred. —The driver of the other car suffered $30,000 in bodily injuries. —Ken's car sustained $5,000 in damages. —Ken incurred $5,000 in medical expenses. —The car that Ken hit was a total loss. Which of Ken's Personal Auto Policy (PAP) coverages will cover the damage to Ken's car?

collision coverage

One general liability loss exposure develops as a result of a written or oral agreement to assume the legal liability of another party. A lease that specifies that the building owner is held harmless for liability arising out of use of the building is an example. This liability loss exposure is

contractual liability

Carelessness or indifference to a loss is an example of A) physical hazard. B) objective probability. C) moral hazard. D) attitudinal hazard.

d

The purpose of adding garagekeepers legal liability coverage to the Garage Coverage Form is to

eliminate the care, custody, and control exclusion under the policy

Some directors and officers (D&O) liability policies include an insuring agreement that covers the legal liability of a corporation arising out of the acts of directors and officers. This coverage for the organization is called

entity coverage

David Florrell started a public accounting firm. David is concerned that accountants on his staff may give incorrect accounting advice, and the accounting firm may be sued. What type of liability insurance should David purchase to protect against such claims?

errors and omissions insurance

ABC Insurance retains the first $1 million of each property damage loss and purchases insurance for that part of any property loss that exceeds $1 million. The insurance for property losses above $1 million is called

excess insurance.

Medical malpractice insurance written on a claims-made basis can leave a surgeon vulnerable to malpractice claims if the surgeon retires, changes insurers, or drops coverage. A provision can be added to the claims-made form that protects the surgeon for future claims arising from incidents that occurred while the claims-made policy was in force. This provision is called a(n)

extended reporting period endorsement

A provision included in a claims-made commercial general liability policy provides coverage for claims filed after the policy has expired. This provision is called a(n)

extended reporting period provision

Purpose of Coinsurance in Property

in a property insurance contract encourages the insured to insure the property to a stated percentage of its insurable value; equity in rating

Coordination of benefits in group health insurance

in group health insurance is designed to prevent overinsurance and the duplication of benefits if one person is covered under more than one group health insurance plan

Which of the following statements is true concerning the ISO employment-related practices liability form?

legal defense costs are included as part of the policy limit

Which of the following losses is generally covered under a commercial umbrella policy?

liability arising out of use of a business auto

All of the following statements about aviation insurance are true EXCEPT

liability coverage applies to liability arising out of a workers compensation law

Kelly was hit by a car while she was walking to the park. She incurred $750 in medical costs at a hospital emergency room. Kelly has coverage for this charge under which of her Personal Auto Policy (PAP) coverages?

medical payments

A situation or circumstance in which a loss is possible, regardless of whether a loss occurs, is called a

loss exposure

Ken purchased a PAP with liability limits of 100/300/50, medical payments coverage, and collision coverage. Ken fell asleep while driving late at night. He crossed the center line and hit a car approaching from the other direction. The following losses occurred. —The driver of the other car suffered $30,000 in bodily injuries. —Ken's car sustained $5,000 in damages. —Ken incurred $5,000 in medical expenses. —The car that Ken hit was a total loss. Which of Ken's Personal Auto Policy (PAP) coverages will cover Ken's medical expenses?

medical payments coverage

The general liability policy covers bodily injury or property damage caused by "an accident, including continuous repeated exposure to substantially the same general harmful conditions." This cause is loss is called a(n)

occurrence

The purpose of gap insurance is to

pay the difference between the amount the insurer pays if a car is a total loss and the amount owed on a lease or car loan

Laura Evans is risk manager of LMN Company. Laura decided to retain certain property losses. All of the following are methods which Laura can use to fund retained property losses EXCEPT

private insurance.

All of the following are coverage options under the physical damage coverage in the Business Auto Coverage Form EXCEPT

property damage liability coverage

All of the following are potential advantages of retention EXCEPT

protection from catastrophic losses.

The purpose of the collision damage waiver when renting an auto is to

relieve the renter of financial responsibility if the rented car is damaged or stolen.

In reviewing his companyȇs operations, a risk manager noticed that all of the companyȇs finished goods were stored in a single warehouse. The risk manager recommended that the finished goods be divided among three warehouses to prevent all of the finished goods from being destroyed by the same peril. Dividing the finished goods among three warehouses illustrates

risk control.

Purchasing health insurance illustrates the use of which personal risk management technique?

risk transfer

Insuring Agreement

summarizes the major promises of the insurer

Holiday DeCor Company is insured by a commercial general liability policy (CGL). Holiday DeCor rented a storage facility for one month to store some excess inventory. While visiting the storage facility, a Holiday DeCor employee discarded a cigarette. The resulting fire severely damaged the storage facility. Which statement is true regarding coverage of this damage under the CGL?

the damage is covered under the fire legal liability coverage

All of the following losses are excluded under Part D (coverage for damage to your auto) of an unendorsed PAP EXCEPT

theft of a compact disc player which was permanently installed in the auto

An employee of Nelson Manufacturing was injured by a defective machine Nelson purchased from Clark Corporation. The employee's tort action against Clark was successful. Clark, in turn, sued Nelson, alleging that Nelson failed to provide proper operating instructions to the employee. This claim (Clark vs. Nelson) is covered under Part Two of Nelson's worker compensation and employer liability policy. Such claims are called

third-party over cases

Angie was injured when her car was struck by a driver who ran a red light. The other driver carried the minimum liability coverage necessary to be considered financially responsible. Angie's injuries were $15,000 above the minimum bodily injury limit. There is a coverage that can be added to the PAP that applies when a negligent driver carries the minimum liability insurance required by the state, but is less than the insured's actual damages for bodily injury.

underinsured motorists coverage.

Acme Company has three identical manufacturing plants, one on the Texas Gulf Coast, one in southern Alabama, and one in Florida. Each plant is valued at $50 million. Acmeȇs risk manager is concerned about the damage which could be caused by a single hurricane. The risk manager believes there is an extremely low probability that a single hurricane could destroy two or all three plants because they are located so far apart. What is the maximum possible loss associated with a single hurricane?

$150 million

Jenkins Company purchased a commercial umbrella policy with a $10 million limit and a $100,000 self-insured retention. The insurer required Jenkins Company to carry a $1 million per-occurrence limit on its general liability policy and a $1 million per-occurrence limit on its business auto policy. Jenkins purchased these required limits. A Jenkins driver was negligent while operating a company vehicle and killed another motorist. The court ruled that Jenkins must pay $5 million. How much of this amount will be paid by the umbrella policy?

$4.0 million

14) Life insurance policyholders may borrow the cash value from their life insurance policies. Where are life insurance policy loans shown on a life insurance company's financial statements? A) as an asset B) as a liability C) as income D) as an expense

A

13) Which of the following statements about state insurance guaranty funds is (are) true? I. They limit the amount that policyholders can collect if an insurer becomes insolvent. II. They are usually funded by general revenues of the states. A) I only B) II only C) both I and II D) neither I nor II

A

14) Disadvantages of automobile insurance plans include which of the following? I. High premiums may cause many high-risk drivers to go uninsured. II. Large underwriting profits have resulted in high-risk drivers subsidizing the cost of insurance for good drivers in the voluntary markets. A) I only B) II only C) both I and II D) neither I nor II

A

1) Under financial responsibility laws, proof of financial responsibility is generally required under all of the following circumstances EXCEPT A) the use of an auto by any driver with fewer than 3 years of driving experience. B) an auto accident involving property damage over a specified amount. C) failure to pay a final judgment resulting from an auto accident. D) conviction for certain serious traffic offenses, such as driving while intoxicated.

A

10) Arguments often used against no-fault automobile insurance laws include all of the following EXCEPT A) It is often difficult to determine which driver was negligent when a multiple-vehicle accident occurs. B) Many injured persons will not be compensated for their full losses because payments for pain and suffering will be eliminated. C) The defects of the negligence system are exaggerated, and the system needs only to be reformed. D) Claims of efficiency and premium saving are exaggerated, and automobile insurance premiums might actually increase.

A

12) When the capital retention approach is used to determine how much life insurance to purchase, all of the following are subtracted from total assets to calculate the capital available to produce income EXCEPT A) investments in stocks and bonds. B) non-income producing capital such as autos and the value of the home. C) the amount of money needed to payoff the mortgage. D) auto loans and credit card debt.

A

13) Under which type of automobile insurance arrangement are all automobile insurers in a state assigned their proportionate share of high-risk drivers based on the total volume of automobile business written in the state? A) automobile insurance plan B) unsatisfied judgment fund C) reinsurance facility D) specialty automobile plan

A

13) Which of the following statements about property and casualty insurance company operating results is (are) true? I. An insurance company can have a combined ratio greater than 1 (or 100 percent) and still be required to pay income taxes. II. By all measures, the property and casualty insurance industry is highly profitable when compared to other industries. A) I only B) II only C) both I and II D) neither I nor II

A

46) The assets of a property and liability insurance company are primarily A) investments such as stocks and bonds. B) loss reserves. C) plant and equipment. D) premiums paid by policyholders.

A

5) A property and casualty insurer's loss reserve includes estimates for all of the following EXCEPT A) claims anticipated but not yet incurred. B) claims reported and adjusted but not yet paid. C) claims reported and filed but not yet adjusted. D) claims incurred but not yet reported to the company.

A

5) Which of the following statements about the grace period in a whole life insurance contract is (are) true? I. The purpose of the grace period is to prevent the policy from lapsing by giving the policy owner additional time to pay an overdue premium. II. If the insured dies during the grace period, the death benefit is reduced by 50 percent. A) I only B) II only C) both I and II D) neither I nor II

A

7) The major difference between the Homeowners 2 policy and the Homeowners 3 policy is the A) insured perils for the dwelling. B) insured perils for personal property. C) type of liability coverage provided. D) amount of coverage for other structures.

A

Rob purchased a Personal Auto Policy (PAP) with collision and other-than-collision coverage. Which of the following losses would be covered under his policy?

A flash flood washed Rob's car off the road and damaged it

46) Steve was involved in an auto accident. Both drivers were partially at fault for the accident. Steve's actual damages were $50,000. He was judged to be 60 percent at fault. If Steve's state has a 51 percent rule for comparative negligence, how much will Steve collect? A) $0 B) $20,000 C) $30,000 D) $50,000

A) $0

*Sarah purchased a PAP with liability limits of 50/100/25. Sarah ran a stop sign and hit a van. The van sustained $15,000 in damages. The following bodily injuries were suffered by passengers in the van: Passenger #1, $15,000; Passenger #2, $60,000; and Passenger #3, $10,000. Sarah sustained $5,000 in medical expenses, and Sarah's car sustained $10,000 in damages. How much will Sarah's insurer pay under Part A: Liability Coverage?*

A) $90,000

35) Janice purchased a living room set for $1,000 and insured this furniture on an actual cash value basis. Two years later the living room set was destroyed by a covered peril. At the time of loss, the property had depreciated in value by 25 percent. The replacement cost of the furniture at the time of loss was $1,200. Assuming no deductible, how much will Janice receive from her insurer? A) $900 B) $950 C) $1,000 D) $1,200

A) $900

30) Which of the following statements about the continuation of group health insurance under COBRA is true? A) A continuation of coverage must be made available even if an employee voluntarily terminates employment. B) The length of the continuation of coverage is 90 days. C) The option to continue coverage applies to minor children only, not to adults. D) The employer must pay the entire cost of coverage during the continuation period.

A) A continuation of coverage must be made available even if an employee voluntarily terminates employment.

19) All of the following statements about business income insurance are true EXCEPT A) Business income is defined as total sales that would have been made if the loss had not occurred. B) Payroll is considered a continuing normal operating expense. C) Business income insurance does not cover the physical damage caused by a peril which created the interruption in business. D) The business income coverage form can be purchased with a coinsurance requirement.

A) Business income is defined as total sales that would have been made if the loss had not occurred.

7) Which of the following statements about Lloyd's of London is true? A) Coverage is actually written by syndicates who belong to Lloyd's of London. B) New individual members or Names who belong to the various syndicates have unlimited legal liability. C) It operates as an admitted insurer throughout the United States. D) It allows underwriters to write coverage without meeting stringent financial requirements.

A) Coverage is actually written by syndicates who belong to Lloyd's of London.

*Duties of an insured after a collision loss covered under the PAP include which of the following?* I. Take reasonable steps to protect the vehicle from further damage. II. Admit fault if the insured believes he or she caused the collision.

A) I only

*Tony has a PAP which provides medical payments coverage. Under which of the following circumstances would the injured person be eligible for benefits under Tony's policy?* I. A friend in the car is injured while Tony is driving a covered auto II. A passenger on a motorcycle driven by Tony is injured when Tony hit another vehicle.

A) I only

*Which of the following statements about the payment of defense costs by the PAP is (are) true?* I. They are paid in addition to the policy limits. II. They are payable even after the limit of liability is exhausted.

A) I only

Which of the following statements regarding health care expenditures in the United States is (are) true? I. As a nation, the U.S. spends significantly more per-person on health care than most other industrialized nations. II. Health care expenditures in the U.S. are high because everyone is covered by a health insurance plan. A) I only B) II only C) both I and II D) neither I nor II

A) I only

35) Jane purchased a $50,000 liability insurance policy from Insurer A. Fearing that she did not have enough liability insurance, she purchased an additional $100,000 of liability coverage from Insurer B. As a result of a negligent act, Jane was ordered to pay $75,000 in damages. Assuming the coverage from Insurer A is primary and the coverage from Insurer B is excess, how will this claim be settled? A) Insurer A will pay $50,000 and Insurer B will pay $25,000. B) Insurer A will pay $37,500 and Insurer B will pay $37,500. C) Insurer A will pay $25,000 and Insurer B will pay $50,000. D) Insurer A will pay nothing and Insurer B will pay $75,000.

A) Insurer A will pay $50,000 and Insurer B will pay $25,000.

25) Kate is covered under her employer's group health plan. She is also covered as a dependent under her husband's group health plan. Under the usual coordination-of-benefits provision, how will each company respond to a claim filed by Kate? A) Kate's plan is primary, and her husband's plan is excess. B) Her husband's plan is primary, and Kate's plan is excess. C) The plan of the person with the birthday earliest in the year pays first, and the other plan is excess. D) Each plan will pay 50 percent of the claim.

A) Kate's plan is primary, and her husband's plan is excess.

28) Maria is covered under a group medical expense plan as an employee. She is also covered under her husbandʹs plan as a dependent. If Maria is hospitalized, how will each plan respond to her medical bills if both plans have the typical coordination-of-benefits provision? A) Mariaʹs plan is primary, and her husbandʹs plan is excess. B) Her husbandʹs plan is primary, and Mariaʹs plan is excess. C) Her husbandʹs plan will pay its benefits, and Mariaʹs plan will deny coverage. D) Both plans will pay benefits on a pro rata basis.

A) Mariaʹs plan is primary, and her husbandʹs plan is excess.

All of the following statements about long-term care insurance are true EXCEPT A) Premiums can be reduced by electing shorter elimination periods. B) A common benefits trigger is the inability to perform a certain number of activities of daily living. C) Protection against inflation is usually made available as an optional benefit. D) Policies currently sold are guaranteed renewable.

A) Premiums can be reduced by electing shorter elimination periods.

35) All of the following statements about group long-term disability income plans are true EXCEPT A) The definition of disability becomes less restrictive after a worker has been disabled for 2 years. B) Coverage is provided for both occupational and nonoccupational disabilities. C) Benefits are reduced if a worker is eligible for Social Security or workers compensation benefits. D) Maximum monthly benefits under long-term disability income plans are higher than the benefits paid under short-term disability income plans.

A) The definition of disability becomes less restrictive after a worker has been disabled for 2 years.

34) Sue double-majored in mathematics and statistics in college. She also enrolled in a number of finance courses. After graduation, she was hired by Econodeath Insurance Company. Her job is to calculate premium rates for life insurance coverages. Sue is a(n) A) actuary. B) underwriter. C) claims adjustor. D) producer.

A) actuary.

14) Life insurance policyholders may borrow the cash value from their life insurance policies. Where are life insurance policy loans shown on a life insurance company's financial statements? A) as an asset B) as a liability C) as income D) as an expense

A) as an asset

One provision of the Affordable Care Act is designed to benefit young adults up to age 26. This provision allows these young adults to A) remain covered under their parents' health insurance policies. B) receive a tax credit for their health insurance premium if they are unemployed. C) receive low-interest government loans to finance their health insurance. D) receive coverage under Medicare if they are not covered by a private health insurance plan.

A) remain covered under their parents' health insurance policies.

36) A manufacturing company just hired a new risk manager, and she has instituted several employee safety programs. She has persuaded the insurer writing the company's workers compensation insurance to base the premium on the company's actual loss experience during the current period rather than on the company's historical performance. This type of plan is called a(n) A) retrospectively rated plan. B) class rated plan. C) experience rated plan. D) judgment rated plan.

A) retrospectively rated plan.

25) There are a number of implied warranties in ocean marine insurance. One implied warranty is that the vessel is properly constructed and maintained, and that it properly equipped for the voyage to be undertaken. This implied warranty is the warranty of A) seaworthiness. B) barratry. C) no deviation from planned course. D) legal purpose.

A) seaworthiness.

37) Advantages of cafeteria plans include all of the following EXCEPT A) simplicity of benefit administration. B) employees can select benefits that best match their needs. C) reduced taxes for employees. D) greater employer control over increasing benefit costs.

A) simplicity of benefit administration.

16) All of the following are reasons for a primary insurer to use reinsurance EXCEPT A) to increase the unearned premium reserve. B) to increase underwriting capacity. C) to protect against catastrophic losses. D) to stabilize profits.

A) to increase the unearned premium reserve.

2) Which of the following is a fundamental purpose of the principle of indemnity? A) to reduce moral hazard B) to minimize physical hazards C) to settle property insurance losses on a replacement cost basis D) to require deductibles in all property insurance policies

A) to reduce moral hazard

27) All of the following are considered to be instrumentalities of transportation and communication for purposes of inland marine insurance EXCEPT A) trains. B) tunnels. C) transmission lines. D) television towers.

A) trains.

26) Jan is employed by an insurance company. She reviews applications to determine whether her company should insure the applicant. If insurable, Jan assigns the applicant to a rating category based on the applicant's degree of risk. Jan is a(n) A) underwriter. B) actuary. C) loss control engineer. D) claims adjustor.

A) underwriter.

1) Which of the following statements about the ownership of a life insurance policy is (are) true? I. Under the ownership clause, the policyholder and beneficiary equally share all contractual rights in the policy while the insured is living. II. The policyholder can designate a new owner by filing an appropriate form with the insurance company. A) I only B) II only C) both I and II D) neither I nor II

B

10) All of the following are covered under the dwelling coverage (Coverage A) of the Homeowners 3 policy EXCEPT A) any structure attached to the dwelling. B) the land on which the insured dwelling is located. C) materials and supplies intended for alteration or repair of the dwelling. D) materials and supplies intended for construction of a detached garage.

B

10) JKL Insurance Company reported the following information on its accounting statements last year: Premiums Written $90,000,000 Loss Adjustment Expenses $5,000,000 Underwriting Expenses $30,000,000 Premiums Earned $100,000,000 Incurred Losses $70,000,000 What was JKL's loss ratio last year? A) 70.0 percent B) 75.0 percent C) 83.3 percent D) 90.0 percent

B

10) Which of the following situations is covered under Section II of an unendorsed homeowners policy? A) injuries which occur at a park resulting from a motorcycle owned by the insured B) injuries resulting from an auto not subject to registration because it is in dead storage C) injuries resulting from an auto rented by the insured while on a vacation D) injuries resulting from a trailer being pulled by the insured's auto

B

11) Which of the following statements about the capital retention approach for determining life insurance needs is (are) true? I. It assumes that life insurance proceeds will be liquidated to provide income to survivors. II. It requires the preparation of a personal balance sheet. A) I only B) II only C) both I and II D) neither I nor II

B

12) Which of the following types of forms is used to insure fluctuations in business personal property? A) a value blanket form B) a value reporting form C) an inventory floater form D) an inventory business income form

B

14) All of the following are exclusions in the homeowners policy that apply to medical payments to others (Coverage F) EXCEPT A) bodily injury resulting from nuclear radiation. B) bodily injury to persons who are social guests at the insured location. C) bodily injury to persons eligible to receive workers compensation benefits. D) bodily injury to residents of the household who are in the care of an insured.

B

16) All of the following statements about the coverage for loss of use (Coverage D) under the Homeowners 3 policy are true EXCEPT A) If a covered loss makes the home unfit for living, the insurer pays additional living expenses the insured may incur as a result of the loss. B) The duration of payments for additional living expenses is limited to a maximum of 15 days. C) There is coverage for the fair rental value of any part of the premises rented to others. D) There is coverage if civil authorities prohibit the insured from using the premises because of a loss to a neighboring premises from an insured peril.

B

16) Which of the following statements about the additional coverage for damage to property of others in Section II of the homeowners policy is true? A) The maximum amount of coverage is $25,000 per occurrence. B) Payments are made on the basis of the replacement cost of the damaged property. C) There is coverage for property damage arising out of a business engaged in on a full-time basis by an insured. D) There is coverage for intentional property damage by any insured who is a teenager.

B

20) When using the needs approach, several "special needs" should be considered. One special need is money to cover unexpected events, such as major car repairs, dental bills, or home repairs. Money set aside for this purpose is called a(n) A) estate clearance fund. B) emergency fund. C) readjustment period fund. D) mortgage redemption fund.

B

23) Sharon lives in a state that has a no-fault automobile insurance law. Under the law, an injured person has the right to sue the negligent driver only if the bodily injury claim exceeds a dollar or verbal threshold. The no-fault law in Sharon's state is a(n) A) pure no-fault plan. B) modified no-fault plan. C) add-on plan. D) choice no-fault plan

B

36) The practice of buying the life insurance policy of a terminally ill insured at a discount is referred to as a A) collateral assignment. B) viatical settlement. C) catastrophic illness conversion. D) grace period transaction.

B

36) Which of the following statements about universal life insurance is (are) true? I. The interest rate credited to the cash value at the time the policy is issued remains fixed for the life of the policy. II. A monthly deduction is made from the policy's cash value for the cost of insurance protection. A) I only B) II only C) both I and II D) neither I nor II

B

36) Which of the following statements concerning the proposed optional federal charter for life insurers is (are) true? I. Large insurers operating in many states would more likely prefer a state charter while smaller, regional, insurers would more likely choose a federal charter. II. Proponents of the federal charter argue that it would speed the development and approval of new products. A) I only B) II only C) both I and II D) neither I nor II

B

37) An Inter-Ocean Transfer cargo ship was forced to jettison some cargo in heavy seas. If $400,000 worth of iron ore was jettisoned, for how much of this amount would Inter-Ocean Transfer be responsible under general average? A) nothing B) $160,000 C) $200,000 D) $400,000

B

39) The exterior walls, the roof, and the plumbing, heating and air conditioning systems of a residential condominium can be insured through the purchase of A) builders risk insurance B) condominium association coverage form C) dwelling property broad form D) condominium commercial unit-owners insurance

B

39) Which of the following statements about the combined ratio is true? A) It is equal to the loss ratio minus the expense ratio. B) A combined ratio greater than 1 (or 100 percent) means an underwriting loss has occurred. C) The combined ratio considers the company's investment income. D) A combined ratio less than 1 (or 100 percent) indicates that an underwriting loss has occurred.

B

4) All of the following statements about the methods of regulating insurance are true EXCEPT A) All states have insurance laws that regulate the operations of insurers. B) Insurers are totally exempt from regulation by federal agencies and laws. C) The courts regulate insurance in many ways, including the interpretation of policy clauses and provisions. D) State insurance commissioners, through administrative rulings, have considerable power over insurers doing business in their states.

B

4) Amy purchased a life insurance policy with the intent of committing suicide to pay all the debts that were burdening her family. If she commits suicide 9 months after the policy is purchased, and the insurer is able to prove that her death was a suicide, how much will be paid by the insurance company? A) nothing, because the policy is void B) the premiums paid for the policy C) the policy's cash value D) the face value of the policy

B

42) A property and liability insurance company's loss reserve and unearned premium reserve are A) assets. B) liabilities. C) income. D) expenses.

B

44) One provision of the Dodd-Frank Act was creation of the Financial Stability Oversight Council. This council is charged with identifying nonbank financial companies that could increase the risk of collapse of the entire financial system. This risk is called A) market risk. B) systemic risk. C) diversifiable risk. D) enterprise risk.

B

47) ABC Insurance Company's investment income ratio last year was 4.2 percent. The company's combined ratio last year was 102.6 percent. What was ABC's overall operating ratio? A) 96.8 percent B) 98.4 percent C) 103.2 percent D) 106.8 percent

B

5) Which of the following statements about the coverage for medical payments to others (Coverage F) under the homeowners policy is true? A) Medical payments may be made for any resident of the named insured's household. B) Medical expenses incurred within 3 years of an accident are covered. C) Medical payments are made only if the insured is not legally liable. D) Covered medical expenses are limited to emergency room charges.

B

53) Which of the following statements about variable universal life insurance is (are) true? I Variable universal life insurance has fixed premium payments. II. Variable universal life insurance allows the policyowner to decide where the premiums are invested. A) I only B) II only C) both I and II D) neither I nor II

B

18) Which of the following is an expense for a life insurance company? A) loss reserves B) death benefits paid to a beneficiary C) unrealized capital gains D) realized capital gains

B) death benefits paid to a beneficiary

26) Ratemakers at ABC Insurance Company calculated the pure premium to be $280 for a risk they were considering insuring. What is the gross rate for this risk, assuming a 30 percent expense ratio? A) $364 B) $400 C) $430 D) $520

B) $400

16) David owns a commercial building with a replacement cost of $4 million. The building is insured on a replacement cost basis for $2.4 million under a fire insurance policy that has an 80 percent coinsurance clause. How much will David collect if the building sustains a covered fire loss with a replacement cost of $80,000? A) $50,000 B) $60,000 C) $66,667 D) $80,000

B) $60,000

19) Connie has a major medical policy with a $200 deductible. She is required to pay 25 percent of covered expenses in excess of the deductible. The insurer will pay 75 percent of expenses in excess of the deductible. If Connie has eligible medical expenses of $10,000, how much will be paid by her insurer? A) $7,300 B) $7,350 C) $7,500 D) $9,800

B) $7,350

21) Delta Insurance Company has a surplus-share treaty with Eversafe Reinsurance. Delta has a retention limit of $200,000, and nine lines of insurance are ceded to Eversafe. How much will Eversafe pay if a $1,600,000 building insured by Delta suffers an $800,000 loss? A) $600,000 B) $700,000 C) $720,000 D) $800,000

B) $700,000

38) Mark owns a building that he insured for $90,000. The replacement cost of the building is $100,000. Mark's property insurance policy has an 80 percent coinsurance clause and no deductible. If Mark's building is destroyed by a covered peril, how much will Mark receive from his insurer? A) $80,000 B) $90,000 C) $101,250 D) $112,500

B) $90,000

3) Sam's furniture was destroyed by a fire. The furniture cost $1200 when it was purchased, but similar new furniture now costs $1800. Assuming the furniture was 50 percent depreciated, what is the actual cash value of Sam's loss? A) $600 B) $900 C) $1200 D) $1800

B) $900

15) MedProf Insurance markets medical malpractice insurance. The company's combined ratio in 2012 was 95.4. Its expense ratio was 25.4. What was the company's loss ratio? A) 60.4 B) 70.0 C) 88.2 D) 120.8

B) 70.0

10) JKL Insurance Company reported the following information on its accounting statements last year: Premiums Written $90,000,000 Loss Adjustment Expenses $5,000,000 Underwriting Expenses $30,000,000 Premiums Earned $100,000,000 Incurred Losses $70,000,000 What was JKL's loss ratio last year? A) 70.0 percent B) 75.0 percent C) 83.3 percent D) 90.0 percent

B) 75.0 percent

18) Which of the following statements about the condominium commercial-unit owners coverage form is (are) true? I. It provides coverage for the unit owner's proportionate financial interest in the condominium building. II. It provides coverage for the business personal property of the unit owner. A) I only B) II only C) both I and II D) neither I nor II

B) II only

20) Which of the following statements about the immunity of governmental entities is (are) true? I. Governmental entities are more likely to be immune from liability when performing proprietary functions than when performing governmental functions. II. Many courts have eliminated the immunity of government entities. A) I only B) II only C) both I and II D) neither I nor II

B) II only

24) Which of the following statements about consideration in an insurance contract is (are) true? I. The insured's total consideration is submission of a completed application. II. The insurer's consideration is the promise to do those things specified in the policy. A) I only B) II only C) both I and II D) neither I nor II

B) II only

45) Which of the following statements about the ISO businessowners policy is true? A) The current form is written on a named-perils basis, but an "open-perils" (all risks) endorsement is available. B) It provides business income and extra expense coverage as an additional coverage. C) The policy covers business personal property but does not cover buildings. D) The policy is designed to meet the special needs of large businesses.

B) It provides business income and extra expense coverage as an additional coverage.

7) Which of the following statements describes how losses will be settled if a property insurance policy is written on a replacement cost basis? A) Losses are settled without the applicable deductible. B) Losses are settled without a deduction for depreciation. C) The insurer must replace the damaged or destroyed property in lieu of a cash settlement. D) The policy is converted to a valued policy.

B) Losses are settled without a deduction for depreciation.

8) One source of life and health insurance underwriting information is an organization that life and health insurance companies can join. As a member, life and health insurance companies report health impairments of applicants, and this information is shared with member companies. Although the information is shared, the underwriting decision of the member company is not disclosed. What is this organization called? A) Fair Isaac Corporation (FICO) B) Medical Information Bureau (MIB) C) National Association of Insurance Commissioners (NAIC) D) National Association of Mutual Insurance Companies (NAMIC)

B) Medical Information Bureau (MIB)

40) A navigation error caused a cargo ship to run aground on a reef. The ship, valued at $1.5 million, was carrying $1.5 million in grain and $1.5 in electronic devices. The ship sustained $150,000 in physical damage. If this loss is a "particular average" loss, how will the loss be settled? A) The owners of each of the three interests are responsible for $50,000. B) The owner of the ship is responsible for the entire $150,000 loss. C) The two cargo owners are each responsible for $75,000 of the loss. D) The owner of the ship is responsible for the first $100,000 of the loss, and the two cargo owners are each responsible for $25,000 of the loss.

B) The owner of the ship is responsible for the entire $150,000 loss.

30) Which of the following statements about retrospective rating is true? A) The premium for the current period is determined by the loss experience in prior periods. B) The premium for the current period is determined by the loss experience during the current period. C) The premium for future periods is determined by the average loss experience for the current and previous periods. D) The premium for future periods is determined by the loss experience for the current period.

B) The premium for the current period is determined by the loss experience during the current period.

19) All of the following statements about business objectives in designing a rating system are true EXCEPT A) The rating system should encourage loss control activities. B) The rating system should be independent of long-run changes in economic conditions. C) The rating system should be simple to understand. D) The rating system should be stable over short periods so that consumer satisfaction can be maintained.

B) The rating system should be independent of long-run changes in economic conditions.

5) Which of the following statements is true about fraternal insurers? A) They are a form of stock insurer. B) They specialize in writing life and health insurance. C) They are taxed more heavily than other types of insurers because of discriminatory marketing practices. D) They account for the majority of life insurance in force in the United States.

B) They specialize in writing life and health insurance.

39) Bruce believes a local manufacturer is responsible for contaminating some land he owns. He filed suit against the company. Rather than have the case go to court, the manufacturing company's legal team suggested mediation or arbitration to settle the case. Methods that are employed to resolve legal disputes without litigation, such as mediation, are called A) collateral source rules. B) alternative dispute resolution techniques. C) joint and several liability techniques. D) comparative negligence rules.

B) alternative dispute resolution techniques.

50) If a third party is led to reasonably believe that an agent is acting with the scope of his/her authority, even though the agent is exceeding his/her authority, the principal may still be bound by the agent's actions. In this case, the agent has bound the principal by A) implied authority. B) apparent authority. C) incidental authority. D) express authority.

B) apparent authority.

23) Under one doctrine, a person who understands the danger inherent in an activity cannot recover damages in the event of injury from the activity. This doctrine is called the A) contributory negligence doctrine. B) assumption of risk doctrine. C) comparative negligence doctrine. D) fellow servant doctrine.

B) assumption of risk doctrine.

47) Dave and Meagan Philips borrowed $150,000 from Fifth National Bank to help fund the purchase of a new home. The home serves as collateral for the loan. Fifth National has an insurable interest in the home based on A) potential responsibility for legal liability. B) being a secured creditor. C) expectation of ownership. D) having a contractual right.

B) being a secured creditor.

*If the value of a vehicle is increased after repairs, such as repainting an entire auto when only one fender or door is damaged, the insurer will not pay for the increase in value. Another name for the increase in value is:*

B) betterment

27) Some employers offer employees a choice of health care plans which are designed to make employees more sensitive to health care costs, to provide an incentive to avoid unneeded care, and to seek low-cost health care providers. Such plans are called A) employee assistance plans. B) consumer-directed health plans. C) cafeteria plans. D) preferred provider organization (PPO) plans.

B) consumer-directed health plans.

*The 2005 PAP states that the insurer has no duty to provide coverage if the insured fails to comply with certain listed duties. In practice, however, the insurer is only relieved of its duty to provide coverage if:*

B) failure to comply with the duties is prejudicial to the insurer

34) The financial services field is currently experiencing consolidation and convergence. If both of these trends continue, in the future we should observe A) fewer financial institutions offering a narrower range of financial services products. B) fewer financial institutions offering a wider range of financial services products. C) more financial institutions offering a narrower range of financial services products. D) more financial institutions offering a wider range of financial services products.

B) fewer financial institutions offering a wider range of financial services products.

40) Marcy advises her clients on investments, taxes, wealth management, estate issues, budgeting, and insurance. Marcy is also a licensed life insurance agent. When Marcy sells life insurance to a client, the distribution channel used is a(n) A) stock broker. B) financial planner. C) financial institution. D) independent agent.

B) financial planner.

Ellen purchased a health insurance policy. Under the provisions of the Affordable Care Act, which of the following renewal provisions must the insurer use in the policy? A) cancellable B) guaranteed issue C) renewable at the insurer's option D) conditionally renewable

B) guaranteed issue

Because of the Affordable Care Act, beginning in 2014, all new medical expense plans that offer individual and group coverage must accept all individuals and employers in the state who apply for coverage. These insurers are required to continue to renew the coverage at the option of the individual or plan sponsor. Thus, under the Affordable Care Act, the renewal provision is A) conditionally renewable. B) guaranteed issue. C) nonrenewable. D) renewable at the insurer's option.

B) guaranteed issue.

42) A property and liability insurance company's loss reserve and unearned premium reserve are A) assets. B) liabilities. C) income. D) expenses.

B) liabilities.

32) Ross studied engineering in college. After graduation, he went to work for an insurance company. Ross visits properties insured by his company. He conducts inspections and makes recommendations about alarm systems, sprinkler systems, and building construction. In what functional area does Ross work? A) underwriting B) loss control C) information systems D) claims adjusting

B) loss control

20) An HMO that contracts with two or more independent group practices to provide medical services to covered members is called a(n) A) group model HMO. B) network model HMO. C) staff model HMO. D) independent practice association HMO.

B) network model HMO.

33) Amy heads the legal staff of a large property and liability insurance company. Amy's staff is likely involved in which of the following activities? A) reviewing investment options for the insurer's assets B) reviewing language and policy provisions in insurance contracts C) calculating premiums to be charged for the insurer's products D) reviewing applications to determine if the company should insure the risk

B) reviewing language and policy provisions in insurance contracts

4) The corporate structure of mutual insurers is changing rapidly. All of the following are current trends EXCEPT A) demutualization of some insurers. B) sharp increase in the number of mutual insurance companies. C) increase in company mergers. D) formation of mutual holding companies.

B) sharp increase in the number of mutual insurance companies.

33) James was injured in an auto accident caused by another motorist's negligence. To reimburse him for his hospital bills and lost earnings, items which can be specifically itemized, James will receive A) punitive damages. B) special damages. C) imputed damages. D) general damages.

B) special damages.

29) Under one type of HMO, the physicians are employees of the HMO and are paid a salary and sometimes an incentive bonus to hold down costs. This type of HMO is called a(n) A) individual practice association (IPA). B) staff model. C) group model. D) network model.

B) staff model.

24) One tort reform permits manufacturers to assert that as long as the product conformed to the prevailing technology and production methods at the time it was produced, it cannot be considered a defective product today. This defense is called the A) collateral source rule. B) state of the art defense. C) strict liability defense. D) privity of contract rule.

B) state of the art defense.

All the following are common exclusions in a major medical insurance policy EXCEPT A) routine dental care. B) surgeons' fees. C) expenses covered by workers compensation laws. D) cost of eyeglasses.

B) surgeons' fees.

12) What is the purpose of stop-loss insurance that is used with self-insured group medical expense plans? A) to require employees to buy insurance for losses in excess of some specified amount B) to have a commercial insurer pay claims that exceed a specified limit C) to obtain administrative services from a commercial insurer D) to exempt self-insured plans from state insurance laws that require mandated benefits

B) to have a commercial insurer pay claims that exceed a specified limit

30) New Liability Insurance Company began operations last year and has been very successful. The company's ability to grow is being restricted by an accounting rule that requires insurers to realize acquisition expenses immediately, while not realizing premiums received as income until some time has passed. Reinsurance is often used in such cases for which of the following purposes? A) to stabilize profitability B) to reduce the unearned premium reserve C) to provide protection against catastrophic losses D) to withdraw from a line of business or territory

B) to reduce the unearned premium reserve

29) A firm wishing to insure a single shipment of merchandise sent by a common carrier would purchase a(n) A) annual transit policy. B) trip transit policy. C) bailee's liability policy. D) equipment floater.

B) trip transit policy.

1) All of the following persons are insured for personal liability under the homeowners policy EXCEPT A) children of the named insured under age 24 who are attending college full time and temporarily residing elsewhere. B) foster children under the age of 21 who reside with the named insured. C) nonresident employees of the named insured. D) the spouse of the named insured if a resident of the same household

C

1) Reasons for regulation of insurance include which of the following? I. Maintaining insurer solvency II. Ensuring reasonable rates A) I only B) II only C) both I and II D) neither I nor II

C

10) Which of the following statements about the use of risk-based capital requirements is (are) true? I. Insurers must have a certain amount of capital depending on the riskiness of their investments and insurance operations. II. Insurers may be required to take certain actions depending on how much capital they have relative to their risk-based capital requirements. A) I only B) II only C) both I and II D) neither I nor II

C

19) All of the following are major rating factors for determining private passenger automobile insurance premiums EXCEPT A) gender. B) age. C) race. D) marital status.

C

2) The human life value is defined as the A) present value of a deceased breadwinner's future gross income. B) future value of a deceased breadwinner's past earnings. C) present value of the family's share of a deceased breadwinner's future earnings. D) future value of the family's share of a deceased breadwinner's future earnings.

C

20) A shortcoming of state regulation of insurance found by Congressional committees and the General Accounting Office is that state regulation A) leads to decentralized governmental power. B) provides opportunities for innovation. C) provides inadequate consumer protection. D) is more responsive to local needs.

C

20) All of the following statements about regulatory objectives of insurance rate making are true EXCEPT A) One purpose of rate adequacy is to maintain the solvency of insurers. B) Rates unfairly discriminate if loss exposures that are similar with respect to losses and expenses are charged substantially different rates. C) Insurers know in advance if the coverages marketed will be profitable, so rate regulation is not needed. D) Rates are excessive if policyholders are paying substantially more than the actual value of their protection.

C

25) Which of the following statements about life insurance settlement options is true? A) Under the fixed period option, the beneficiary normally has the right to make partial withdrawals in case of emergency. B) Under the fixed period option, any remaining proceeds revert to the insurer if the beneficiary dies before the end of the fixed period. C) Under the fixed amount option, the beneficiary can be given the right to increase or decrease the fixed amount. D) Under the fixed amount option, any interest credited in excess of the guaranteed rate increases the amount of each periodic payment.

C

26) David has a Homeowners 3 policy that provides $280,000 of insurance on his dwelling, which has a current replacement value of $400,000. Ignoring any deductible, how much will David collect if a kitchen with a replacement value of $24,000 but an actual cash value of $18,000 is destroyed in a fire? A) $18,000 B) $20,000 C) $21,000 D) $24,000

C

31) Which of the following statements is (are) true with respect to the use of credit-based insurance scores as an auto insurance rating factor? I. Insurers claim that drivers who have poor insurance scores are expected to have relatively more accidents. II. The use of insurance scores in personal lines underwriting is controversial. A) I only B) II only C) both I and II D) neither I nor II

C

32) Renee is risk manager of XYZ company. She purchased a commercial Package Policy for her company and added one of the optional coverages. This option suspends the coinsurance clause and substitutes a new agreement covering any loss int eh same proportion that the limit of insurance bears to a value specified in the declarations. This provision is known as A) waiver of inventory coverage B) inflation guard coverage C) agreed value coverage D) replacement cost coverage

C

35) All of the following statements about universal life insurance are true EXCEPT A) Interest is credited to the policy's cash value each month. B) Any withdrawal of a policy's cash value reduces the amount of the death benefit. C) Interest credited to a policy's cash value is taxable for the policyowner in the year credited. D) The policyowner can add to a policy's cash value at any time subject to policy guidelines.

C

44) A property and liability insurance company's loss ratio was 74 percent in 2011, 68 percent in 2012, and 66 percent in 2013. The same insurer's expense ratio was 31 percent in 2011, 33 percent in 2012, and 30 percent in 2013. Which of the following statements is true about the company's underwriting results? A) The insurer made money from its underwriting activities each year. B) The insurer's profitability from underwriting has been deteriorating each year. C) The insurer's profitability from underwriting has been improving each year. D) The insurer lost money from its underwriting activities each year.

C

45) Jane purchased a life insurance policy on her own life and named her daughter, Cheryl, as beneficiary. Cheryl has a history of not managing money well. Jane wants the death benefit paid to Cheryl in monthly installments over 20 years. Which settlement option should Jane pre-select for Cheryl? A) lump sum B) fixed amount C) fixed period D) interest option

C

47) Carl would like to purchase life insurance. He would also like to invest in a mutual fund. An agent told Carl about a form of life insurance in which Carl could select where the saving component is invested. This form of life insurance has fixed premiums and the cash value is not guaranteed. This type of life insurance is called A) universal life insurance. B) current assumption whole life. C) variable life insurance. D) indeterminate-premium whole life.

C

47) One method of ensuring the solvency of insurers is a periodic review, every three to five years, of insurers that operate on a multistate basis. This review is coordinated by the NAIC. This review is called a(n) A) annual report. B) early warning system. C) field examination. D) inspection report.

C

52) Which of the following $100,000 whole life insurance policies, issued by the same company to a man age 32, would require the highest first-year premium? A) continuous premium (ordinary) life B) whole life paid-up at 65 C) 10-payment whole life D) 20-payment whole life

C

58) Which statement about the incontestable clause is true? I. It protects the beneficiary if the insurer tries to deny a claim, years after the policy is issued. II. It protects the insurer from having to pay a claim during the first two years if the insured made a material misrepresentation or concealed material information in the application. A) I only B) II only C) both I and II D) neither I nor II

C

44) In the context of medical malpractice, what is a "never event"? A) A medical condition that despite the best screening and technology cannot be detected. B) A pandemic that quickly spreads and infects many people. C) A medical error that should never occur. D) An event for which doctors and medical facilities cannot be held legally responsible.

C) A medical error that should never occur.

40) Dave is an agent for Easy Pay Insurance. Easy Pay insures only the highest-quality applicants. Dave wanted to earn more commissions, so he sold some policies to applicants he knew were below-average risks. When these policyowners started filing claims, Easy Pay tried to deny the claims stating that Dave had not acted appropriately. Which general rule of agency makes Easy Pay responsible for the claims of the higher-than-average risk policyowners? A) There is no presumption of an agency relationship. B) Agents should be compensated based on the quality of the business they generate. C) A principal is responsible for the acts of its agents who are acting within the scope of their authority. D) An agent must have authority to represent the principal.

C) A principal is responsible for the acts of its agents who are acting within the scope of their authority.

22) Which of the following statements about offer and acceptance for insurance contracts is true? A) In property and liability insurance, agents typically do not have the authority to bind coverage. B) In life insurance, the agent can usually accept an offer by immediately binding coverage. C) In property insurance, the offer and acceptance are usually in writing but may be oral. D) In life insurance, the offer is merely the promise to pay the first premium.

C) In property insurance, the offer and acceptance are usually in writing but may be oral.

25) Which of the following statements regarding the Health Insurance Portability and Accountability Act is true? A) It guarantees the availability of group health insurance to small employers. B) It requires employers to provide at least a minimum level of health benefits prescribed by law. C) It requires giving credit for previous coverage with respect to any preexisting conditions exclusion found in a new health plan. D) It limits preexisting conditions exclusions to 90 days.

C) It requires giving credit for previous coverage with respect to any preexisting conditions exclusion found in a new health plan.

) What is the effect of the optional agreed value provision in the building and personal property coverage form? A) It provides replacement cost coverage. B) It increases insured values along with inflation. C) It suspends the coinsurance clause. D) It reduces the deductible to $50.

C) It suspends the coinsurance clause.

7) Which of the following statements about group term life insurance is true? A) It usually is written in the form of 5-year level term insurance. B) An employee who leaves the group can usually convert the coverage to an individual term life insurance policy. C) Many employers provide a reduced amount of coverage on retired employees. D) It represents only about 5 percent of the group life insurance in force.

C) Many employers provide a reduced amount of coverage on retired employees.

33) Which of the following statements about group short-term disability income plans is true? A) Most plans pay benefits for a period of 3 to 5 years. B) Most plans cover occupational disabilities only. C) Most plans provide benefits for total disabilities only. D) Most plans have a 90-day elimination (waiting) period.

C) Most plans provide benefits for total disabilities only.

13) Which of the following statements about the business income coverage form is true? A) Business income is defined as gross earnings before taxes. B) Payroll is excluded unless it is specifically added. C) The form covers loss of business income and extra expenses incurred during restoration. D) The form can be used by a manufacturing operation only.

C) The form covers loss of business income and extra expenses incurred during restoration.

15) All of the following statements about the independent agency system are true EXCEPT A) Agents are often authorized to adjust small claims. B) Agents are compensated on the basis of commissions. C) The insurer rather than the agent owns the renewal rights to the business. D) The agent is an independent business person who represents several insurers.

C) The insurer rather than the agent owns the renewal rights to the business.

44) A property and liability insurance company's loss ratio was 74 percent in 2011, 68 percent in 2012, and 66 percent in 2013. The same insurer's expense ratio was 31 percent in 2011, 33 percent in 2012, and 30 percent in 2013. Which of the following statements is true about the company's underwriting results? A) The insurer made money from its underwriting activities each year. B) The insurer's profitability from underwriting has been deteriorating each year. C) The insurer's profitability from underwriting has been improving each year. D) The insurer lost money from its underwriting activities each year.

C) The insurer's profitability from underwriting has been improving each year.

32) All of the following statements about the rules governing agency relationships are true EXCEPT A) An agent must be authorized to act on behalf of a principal. B) An agency agreement may grant certain powers to the agent as well as denying the agent other powers. C) The principal is responsible for the acts of agents only if the acts are criminal. D) Knowledge of the agent is presumed to be knowledge of the principal with respect to matters within the scope of the agency relationship.

C) The principal is responsible for the acts of agents only if the acts are criminal.

*All of the following are considered to be uninsured vehicles for purposes of the uninsured motorists coverage of the PAP EXCEPT*

C) a vehicle owned by an individual who purchased enough liability insurance to satisfy the state's financial responsibility law.

43) In the optional replacement cost coverage provision is not selected, how are losses settled under the building and personal property (BPP) coverage form? A) market value B) original purchase price C) actual cash value D) book value

C) actual cash value

7) One of the additional coverages under the building and personal property coverage form is "increased cost of construction." The coverage is payable if A) a skilled artisan is needed to duplicate the construction of the damaged property. B) a labor strike or materials shortage increases construction costs. C) an ordinance or building code increases the cost of construction. D) the materials necessary to rebuild the damaged structure are more expensive than ordinary building materials.

C) an ordinance or building code increases the cost of construction.

31) HBM Company offers three health plans to its employees. The ʺGold Planʺ costs $500 per month and provides generous benefits. The ʺSilver Planʺ costs $350 per month and provides more limited benefits. The ʺBronze Planʺ is quite limited and costs $250 per month. HBM contributes $250 per month for each employeeʹs health care benefits. Employees who desire broader coverage are free to contribute additional money to fund the coverage. Based on the description, HBMʹs health plan is a A) preferred provider health plan. B) point-of-service health plan. C) defined contribution health plan. D) high-deductible health plan.

C) defined contribution health plan.

35) One type of commercial property insurance excludes perils which are covered by the basic coverages. Some businesses buy this coverage to fill coverage gaps, including flood and earthquake, and to cover property in other countries. This type of insurance is called A) protection and indemnity insurance. B) building and personal property coverage form. C) difference in conditions insurance. D) builders risk coverage form.

C) difference in conditions insurance.

27) ABC Term Life Insurance Company uses an interesting marketing system—it has no agents. Instead, the company markets its coverages through television and radio ads, newspaper inserts, and the Internet. The type of marketing system that ABC Term Life Insurance Company uses is called the A) mass merchandising system. B) mixed marketing system. C) direct response system. D) worksite marketing system.

C) direct response system.

24) Under one type of merit rating, the class or manual rate is adjusted upward or downward based on past loss history. This type of merit rating is called A) schedule rating. B) judgment rating. C) experience rating. D) retrospective rating.

C) experience rating.

21) The unit of measurement used in property and casualty insurance pricing is called the A) unit rate. B) premium. C) exposure unit. D) experience unit.

C) exposure unit.

44) Some courts have ruled that an alternative to "replacement cost less depreciation" should be used to determine the actual cash value of a property loss. Under this alternative, the value of property lost is determined by the price a willing buyer would pay a willing seller for the property in a free market. This method of determining actual cash value is called the A) intrinsic value method. B) valued policy method. C) fair market value method. D) reconstruction cost method.

C) fair market value method.

22) The leaders of a religious group decided to start a life insurance organization to insure members of the religious group. The insurer will operate as a nonprofit organization, thus receiving favorable tax treatment. The insurer formed will be a A) health maintenance organization. B) stock insurer. C) fraternal insurer. D) reciprocal exchange.

C) fraternal insurer.

42) Mark owns a bar. The bar has a back room where Mark has some slot machines. Mark lets some of his patrons play the machines, and Mark keeps any profits. This type of gambling is illegal where Mark lives. Mark wanted to purchase insurance in case his slot machines were confiscated by the police. Such an insurance contract would not be enforceable. Which requirement needed to form a valid insurance contract is missing? A) consideration B) offer and acceptance C) legal purpose D) competent parties

C) legal purpose

8) High deductible group health insurance plans have all of the following characteristics EXCEPT A) health savings accounts or health reimbursement accounts B) high dollar deductibles C) low coverage limits D) major medical insurance

C) low coverage limits

23) Chris applied for life insurance and paid the first premium on Monday. She was given an insurability premium receipt which specified that coverage was effective on the date of the application or the date of the medical exam, whichever is later. She took the medical exam the following Thursday. She was found to be in perfect health. On which day was her coverage effective? A) on Monday, when she completed the application and paid the first premium B) on Wednesday, two days after completing the application and paying the first premium C) on Thursday when she passed the medical exam D) on Saturday, two days after passing the medical exam

C) on Thursday when she passed the medical exam

50) One tort reform proposal is capping noneconomic damages. Noneconomic damages include A) payment for time missed from work. B) payment for the cost of a hospital stay. C) payment for pain and suffering. D) payment for the cost of care provided by a physical therapist.

C) payment for pain and suffering.

54) A pharmaceutical company employs a young chemist who is responsible for three new patents last year and for the development of the company's two best-selling drugs. The company purchased a large life insurance policy on the chemist. In this case, the insurable interest requirement was met because of a(n) A) ownership interest. B) close family relationship. C) pecuniary interest. D) economic family relationship.

C) pecuniary interest.

22) Lisa has three fire insurance policies on her office building. The policy from company A is for $400,000, and the policies from companies B and C are for $100,000 each. If Lisa has a $360,000 loss, how much of the loss will be covered by each policy if the loss is settled on a pro rata basis by the insurers? A) each policy: $120,000 B) policy A: $160,000; policies B and C: $100,000 each C) policy A: $240,000; policies B and C: $60,000 each D) policy A: $360,000; policies B and C: nothing

C) policy A: $240,000; policies B and C: $60,000 each

*The Miscellaneous-Type Vehicle Endorsement to the PAP can be used to insure all of the following vehicles EXCEPT*

C) snowmobiles

29) Jim would like to start a business raising thoroughbred racehorses. The business would be the first of its kind in the state where he lives. Obtaining insurance on the horses is a key concern, and he was dismayed to learn that none of the insurers authorized to operate in his state offer this specialty insurance. What is the name of the intermediary which Jim can use to place this coverage with an insurer not admitted to his state? A) alien insurer B) general agent C) surplus lines broker D) direct writer

C) surplus lines broker

*All of the following statements about the termination provisions of the PAP are true EXCEPT*

C) the insurer can cancel the policy after it has been in force for 60 days only if the insured has three or more traffic violations

52) ABC Life Insurance Company insures both smokers and nonsmokers. Beth lied on her life insurance application, checking the box for "no" in response to the question of whether she smokes cigarettes or uses other tobacco products. Even though Beth smokes 10 to 15 cigarettes each day, the policy was issued at the "preferred nonsmoker rate." Beth's lie is materiel in this case because A) it was in writing on the application. B) it was given with the intent to deceive. C) the policy would have been issued on different terms if the insurer knew the true facts. D) the policy would have been issued for a lower face value if the insurer knew the true facts.

C) the policy would have been issued on different terms if the insurer knew the true facts.

37) Melody's car was damaged when another driver ran a stop sign and hit her car. Melody decided to collect from her own insurer and to let her insurer recoup the loss payment from the negligent driver who hit her. What fundamental legal principle is illustrated in this scenario? A) the principle of utmost good faith B) the principle of insurable interest C) the principle of subrogation D) the principle of reasonable expectations

C) the principle of subrogation

All of the following are historical reasons for the increase in health care expenditures in the U.S. EXCEPT A) cost insulation because of third-party payers. B) employment-based health insurance. C) universal health insurance coverage. D) technological advances in health care.

C) universal health insurance coverage.

12) JKL Insurance Company reported the following information on its accounting statements last year: Premiums Written $90,000,000 Loss Adjustment Expenses $5,000,000 Underwriting Expenses 30,000,000 Premiums Earned $100,000,000 Incurred Losses $70,000,000 What was JKL's combined ratio last year? A) 100.0 B) 103.3 C) 105.0 D) 108.3

D

12) Which of the following situations would be covered under Section II of a homeowners policy? I. The insured is sued by his girlfriend because he infected her with the AIDS virus. II. The insured's son is sued after a friend suffered serious injury as a result of using illegal drugs sold to him by the son. A) I only B) II only C) both I and II D) neither I nor II

D

1) Which of the following statements is (are) true regarding renters insurance? I. Renters insurance is needed if you rent a house, but is not needed if you rent an apartment. II. The ISO renterʹs policy provides ʺall-risksʺ coverage on the insuredʹs personal property. A) I only B) II only C) both I and II D) neither I nor II

D

10) The period during which a surviving spouse is ineligible for Social Security benefits is referred to as the A) emergency period. B) readjustment period. C) dependency period. D) blackout period.

D

12) All of the following losses are subject to special limits of liability under the Homeowners 3 policy EXCEPT the A) theft of firearms. B) destruction by fire of property used for business purposes. C) storm damage to a boat at a marina. D) destruction of jewelry by fire.

D

13) Disadvantages of the capital retention approach include which of the following? I. Assets are often liquidated too quickly. II. It underestimates the amount of life insurance needed. A) I only B) II only C) both I and II D) neither I nor II

D

13) Which of the following statements about the personal property coverage (Coverage C) of the Homeowners 3 policy is (are) true? I. The coverage applies only in the United States and Canada. II. The full amount of coverage applies only if the property is permanently located at any residence of the insured. A) I only B) II only C) both I and II D) neither I nor II

D

14) Tom and Nancy Boyle provide financial support for their two children. In addition, they provide financial support for Tom's aged father and Nancy's aged mother. The Boyle family can be described as a A) blended family. B) single-parent family. C) two-income earner family. D) sandwiched family.

D

14) Under one type of rate regulation, insurers do not have to register their rates with state regulatory authorities. However, insurers may be required to furnish rate schedules and supporting data to state officials. A fundamental assumption underlying this type of rating law is that market forces will determine the price and availability of insurance, rather than discretionary acts of regulators. This type of rate regulation is called A) a flex-rating law. B) a prior-approval law. C) a file-and-use law. D) no filing required

D

21) Most family heads need substantial amounts of life insurance. However, with limited income, money spent on life insurance reduces the amount of discretionary income available for other high-priority needs. What an insured person gives up when he or she purchases life insurance instead of using the premium dollars for other purposes is called the A) estimated cost of life insurance. B) net cost of life insurance. C) real (inflation-adjusted) cost of life insurance. D) opportunity cost of buying life insurance.

D

24) All of the following statements about term insurance are true EXCEPT A) The insurance provides protection for a specified period of time. B) Most policies can be renewed for additional periods without evidence of insurability. C) Most policies can be converted to a permanent life insurance policy. D) Most policies have a cash value that is refunded when coverage ceases.

D

24) Which of the following statements about marine insurance coverages is true? A) Hull insurance limits coverage to the breakdown of a ship's machinery and equipment. B) Cargo insurance protects the shipowner for the loss of the earnings if a ship is lost of damaged and goods are not delivered. C) Freight insurance covers the owner of cargo if the cargo is damaged or destroyed. D) Protection and indemnity insurance provides liability insurance to the ship owner for bodily injury and property damage to third parties.

D

25) A life insurance company based in Canada was licensed to operate in Massachusetts. When operating in Massachusetts, the Canadian insurer would be considered a(n) A) domestic insurer. B) captive insurer. C) foreign insurer. D) alien insurer.

D

25) A number of benefits are payable under no-fault plans. Under one provision, benefits are paid for tasks normally performed by the insured, including such things as lawn care, housework, and home repairs. These tasks are called A) home health care services. B) hospice services. C) activities of daily living. D) essential services expenses.

D

25) All of the following statements about the conversion of a term policy are true EXCEPT A) Under an attained age conversion, the premium is based on the insured's attained age at the time of conversion. B) Under an original age conversion, the policyowner must pay a financial adjustment in addition to the premium for the new policy. C) Most insurers require original age conversion to take place within a specified period (5 years, for example) of the issue of the term policy. D) Evidence of insurability is required before a conversion is permitted.

D

27) Which of the following statements about the appraisal clause in the Homeowners 3 policy is true? A) The appraiser for each party is selected by a judge. B) A binding settlement can be reached only if the umpire agrees with both appraisers. C) Both the insured and the insurer must agree that the appraisal process be used. D) Appraisal expenses are shared by the insured and the insurer.

D

3) All of the following statements about the personal liability coverage (Coverage E) of an unendorsed homeowners policy are true EXCEPT A) Coverage is written on an occurrence basis. B) Coverage is provided for bodily injury liability. C) Coverage is provided for property damage liability. D) Coverage is provided for personal injury liability.

D

30) State X's premium tax rate is 2 percent. State Y's premium tax rate is 3 percent. State X insurers are required to pay the 3 percent rate on business written in State Y. State X requires insurers from State Y to pay a 3 percent premium tax on business written in State X, even though the premium tax rate is only 2 percent in State X. This practice is known as a A) tax tariff. B) guaranty fund assessment. C) risk-based capital requirement. D) retaliatory tax law.

D

32) Which of the following statements about limited-payment life insurance is true? A) It is a form of term insurance. B) It matures at the end of the premium-payment period. C) The premium decreases each year during the premium-payment period. D) Its use may be appropriate if a person wants paid-up life insurance during retirement.

D

35) Which of the following statements is (are) true regarding the quality of insurance regulation? I. The quality of insurance regulation is uniform from state to state. II. All evidence suggests federal regulation of insurance would improve the quality of regulation. A) I only B) II only C) both I and II D) neither I nor II

D

35) XYZ Insurance Company uses class rating to determine the rate to charge for insurance. For one type of insurance, the pure premium XYZ actuaries calculated is $75 per unit. If XYZ's expense ratio is 25 percent, what is the gross rate for this coverage? A) $37.50 B) $55.25 C) $75.00 D) $100.00

D

38) Tom opened a store in a mall. His store is located between a theater and a department store. Tom counts on the theater and department store to generate walk-in business at his store. While his store has been successful, Tom knows that if either or both of the other business closed, his store would suffer an economic loss. What type of dependent property situation is illustrated in this scenario? A) contributing location B) recipient location C) manufacturing location D) leader location

D

5) All of the following are extensions of coverage under the building and personal property coverage form EXCEPT A) newly acquired or constructed property B) valuable papers and records C) property off the premises D) currency and securities

D

6) Which of the following statements is true with respect to a pure no-fault auto insurance plan? A) Under such a plan, you would collect from your own insurer and retain the right to sue the other party if your injuries surpass a dollar or verbal threshold. B) Most no-fault plans that have been adopted by states are pure no-fault plans. C) Under such a plan, you would collect from your own insurer and retain the right to sue the other party without having to satisfy a threshold. D) Under such a plan, an accident victim cannot sue the other party, and no payments are made for pain and suffering.

D

7) All of the following are considered insured locations for medical payments to others (Coverage F) under a homeowners policy EXCEPT A) a new vacation home purchased by an insured during the policy period. B) a motel room where an insured is temporarily residing. C) a hall rented by the insured for her daughter's graduation party. D) farmland owned by the insured.

D

7) Which of the following statements about modified no-fault laws is (are) true? I. Claims less than a certain dollar threshold must be assumed by the injured accident victim, but the injured person has the right to sue a negligent driver. II. Claims above a certain dollar threshold are paid in full by an injured accident victim's insurer, and the right to sue a negligent driver is eliminated. A) I only B) II only C) both I and II D) neither I nor II

D

8) A loss reserve established for each individual claim when it is reported to a property and casualty insurance company is call a(n) A) admitted asset. B) incurred-but-not-reported (IBNR) reserve. C) unearned premium reserve. D) case reserve.

D

8) Which of the following is considered a nonadmitted asset for an insurer? A) cash B) preferred stocks C) real estate D) office furniture

D

8) Which of the following statements about beneficiary designations is (are) true? I. The primary beneficiary is entitled to the death proceeds of a life insurance policy only if the contingent beneficiary dies before the insured. II. If a revocable beneficiary designation is used, the insured must obtain the beneficiary's permission of exercise most policy rights. A) I only B) II only C) both I and II D) neither I nor II

D

9) All of the following are arguments for no-fault automobile insurance laws EXCEPT A) No-fault is unnecessary as it's easy to determine which driver was negligent when a multiple-vehicle accident occurs. B) A large portion of each premium dollar is used for purposes other than compensating accident victims for their losses. C) There are delays in compensating accident victims under the tort system. D) Seriously injured accident victims tend to be overcompensated, while those with small economic losses are inadequately indemnified.

D

One provision of the Affordable Care Act provides the creation in each state of a transparent and competitive insurance marketplace where individuals and small firms with fewer than 100 employees can purchase affordable and qualified health coverage. This marketplace is called a(n) A) Medicaid plan. B) Medicare plan. C) Health Maintenance Organization (HMO). D) Affordable Health Insurance Exchange.

D) Affordable Health Insurance Exchange.

1) All of the following statements about the characteristics of group insurance are true EXCEPT A) Many people are covered under a single contract. B) The cost of group insurance may be lower than the cost of individual insurance. C) The actual experience of a large group is a factor in determining the premium that is charged. D) Individual evidence of insurability is usually required.

D) Individual evidence of insurability is usually required.

36) Jacob sold his house to Shelia for $140,000 in cash. Jacob "threw in" insurance on the house as part of the deal and did not bother telling the insurer that there was a new owner. Four months after Shelia purchased the home, a windstorm damaged the roof. Which of the following legal characteristics of insurance contracts could the insurer use to legally deny payment for the damage to the roof? A) Insurance contracts are unilateral contracts. B) Insurance contacts are contracts of adhesion. C) Insurance contracts are aleatory contracts. D) Insurance contracts are personal contracts.

D) Insurance contracts are personal contracts.

*John has an auto which is covered for collision losses subject to a $250 deductible. Kate's auto also has collision coverage but her deductible is $500. Which of the following statements describes how a $2,000 collision loss will be paid if it occurs when John borrows Kate's car because his car is in the shop for repairs?*

D) Kate's policy will pay $1,500, and John's policy will pay $250.

14) Which of the following statements about the exclusive agency system for marketing property and liability insurance is true? A) Exclusive agents typically have complete ownership of policy expirations. B) A higher commission rate is usually paid on exclusive agents' renewal business than on new business. C) Exclusive agents represent several different insurance companies. D) New exclusive agents may start as employees and after a training period become independent contractors.

D) New exclusive agents may start as employees and after a training period become independent contractors.

Which of the following statements about individual disability income policies is true? A) Benefits are typically paid only for disabilities resulting from sickness. B) Benefits paid for partial disabilities are usually greater than benefits paid by the same policy for total disabilities. C) Most policies pay a benefit equal to 100 percent of the disabled person's lost income. D) Newer policies often provide or make available a residual disability benefit for persons who are able to work but at a reduced income.

D) Newer policies often provide or make available a residual disability benefit for persons who are able to work but at a reduced income.

23) Kevin has three liability policies which provide for contribution by equal shares if other insurance applies to a loss. How much will each policy pay for a $3,000,000 liability judgment if policy A provides $500,000 of coverage, policy B provides $1,000,000 of coverage, and policy C provides $3,000,000 of coverage? A) Each policy will pay $500,000, and Kevin must pay the remaining $1,500,000. B) Policy A will pay $500,000, policies B and C will each pay $1,000,000, and Kevin must pay the remaining $500,000. C) Policy A will pay nothing, policy B will pay $1,000,000, and policy C will pay $2,000,000. D) Policy A will pay $500,000, policy B will pay $1,000,000, and policy C will pay $1,500,000.

D) Policy A will pay $500,000, policy B will pay $1,000,000, and policy C will pay $1,500,000.

18) A reinsurance contract that is entered into on a case-by-case basis after an application for insurance is received by a primary insurer is called A) a reinsurance pool. B) automatic treaty reinsurance. C) retrocession. D) facultative reinsurance.

D) facultative reinsurance.

9) Which of the following perils is not included in the causes-of-loss basic form of the ISO commercial package policy? A) fire B) lightning C) explosion D) flood

D) flood

19) An HMO physician who determines if medical care from a specialist is necessary is called a(n) A) capitator. B) internist. C) network facilitator. D) gatekeeper.

D) gatekeeper.

34) James was injured in an auto accident caused by another motorist's negligence. He received severe facial lacerations and injured his back in the accident. In payment for his pain, suffering, and disfigurement, losses which cannot be specifically itemized, James will receive A) punitive damages. B) special damages. C) imputed damages. D) general damages.

D) general damages.

23) Scott works in property and liability insurance marketing. He legally represents insurance purchasers, rather than insurance companies. Scott is paid a commission on the insurance placed with insurers. Scott is a(n) A) exclusive agent. B) direct writer. C) branch manager. D) insurance broker.

D) insurance broker.

10) Which of the following is a function of the marketing department of an insurance company? A) to settle claims after a loss has been reported B) to determine the cost of products the insurer sells C) to make final underwriting decisions D) to identify production goals

D) to identify production goals

*What is the purpose of the extended non owned liability coverage endorsement to the PAP?*

D) to provide liability coverage for an insured person who operates a non owned auto on a regular basis

49) State insurance regulators require LMN Life Insurance Company to maintain a separate account. The assets in the separate account would support the liabilities for which of the following products? A) term life insurance B) whole life insurance C) fixed annuity D) variable life insurance

D) variable life insurance

Which of the following statements about Medigap policies is (are) true? I. Insurers are required to have an open enrollment period of 6 months from the date an applicant first enrolls in Medicare Part B and is age 65 or older. II. Most policies are of limited value because they largely duplicate benefits already provided by Medicare.

I only

The Commercial General Liability (CGL) Policy covers liability arising out of which of the following loss exposures? I. Bodily injury of an employee of the insured arising out of and in the course of employment II. Bodily injury of a customer injured by a defective product manufactured by the insured

II only

Which of the following conditions is (are) appropriate for using retention? I. Losses are difficult to predict. II. The worst possible loss is not serious.

II only

Which of the following is (are) covered under a claims-made policy? I. Claims arising out of occurrences which take place during the policy period, regardless of when the claim is made II. Claims first reported during the policy period, provided the event occurred after a retroactive date stated in the policy

II only

Which of the following statements about hospice benefits under the hospital insurance (Part A) portion of Medicare is (are) true? I. Hospice benefits are payable only while the beneficiary is hospitalized. II. Hospice benefits are provided to persons who are terminally ill.

II only

Which of the following statements about taxation of Social Security retirement benefits under federal law is (are) true? I. Social Security retirement benefits are never considered taxable income. II. Up to 85 percent of Social Security retirement benefits may be considered taxable income, depending on the amount of other income received by the beneficiary.

II only

Which of the following statements about the Miscellaneous-Type Vehicle Endorsement to the PAP is (are) true? I. It provides bodily injury liability coverage for any vehicle rented by the insured. II. To lower premiums when a motorcycle is insured, bodily injury to passengers can be excluded.

II only

Which of the following statements about the Social Security cost-of-living adjustment is (are) true? I. The amount of the adjustment is limited to a maximum of 2.5 percent annually. II. Increases are based on changes in the consumer price index.

II only

Which of the following statements about the typical commercial umbrella policy is (are) true? I. The policy is designed to cover only those loss exposures specifically excluded by the underlying insurance contracts. II. The policy is excess over the required underlying coverages.

II only

Which of the following statements is (are) true with respect to the Medicare Advantage Plans? I. These plans replace the original Medicare program, Parts A and B, which are no longer available to new retirees. II. These plans provide an opportunity for retirees to receive Medicare coverage through private health care plans.

II only

Which of the following statements is (are) true with respect to the treatment of legal defense costs under the ISO Commercial General Liability policy? I. Legal defense costs usually are counted against the policy limit. II. Once the insurer has paid out the applicable limit of liability, the insurer's duty to defend ends.

II only

Which of the following statements regarding the use of retention is (are) true? I. Retention is best used for loss exposures that have a low frequency and a high severity. II. A financially strong firm can have a higher retention level than a firm whose financial position is weak.

II only

Which of the following types of loss exposures may be appropriately handled through the purchase of insurance? I. High-frequency, low-severity II. Low-frequency, high-severity

II only

All of the following statements about the administration of a risk management program are true EXCEPT

If a risk management program is properly designed, periodic review of the program is unnecessary

When the insurer has to meet coinsurance requirement

If the insured wants to collect in full for a partial loss, the coinsurance requirement must be satisfied

All of the following statements about avoidance are true EXCEPT

It can be used for any loss exposure facing a firm.

All of the following are disadvantages of using insurance in a risk management program EXCEPT

It results in considerable fluctuations in earnings after losses occur.

John has an auto which is covered for collision losses subject to a $250 deductible. Kate's auto also has collision coverage but her deductible is $500. Which of the following statements describes how a $2,000 collision loss will be paid if it occurs when John borrows Kate's car because his car is in the shop for repairs?

Kate's policy will pay $1,500, and John's policy will pay $250

Type of Policy that doesn't contain a deductible

Life Insurance Policy

All of the following statements about Part D (coverage for damage to your auto) of the PAP are true EXCEPT

No coverage is provided for newly-acquired vehicles

John occasionally borrows the car of his friend, Sophie. Sophie has a PAP with liability limits of 100/300/50. John also has a PAP, and his liability limits 250/500/50. John had an accident while using Sophie's car and was found to be legally liable for $300,000 in bodily injury liability for injuries suffered by one person. How much will be paid by each policy?

Sophie's policy will pay $100,000, John's policy will pay $200,000

Joyce was injured by an uninsured drunk driver while she was riding in a friend's car. Joyce and her friend each have a PAP with an uninsured motorists limit of $50,000. How much will be paid by each policy if it is determined that Joyce has $70,000 of bodily injuries?

The friend's policy will pay $50,000, and Joyce's policy will pay $20,000.

Which of the following statements is true about uninsured motorists coverage under the Personal Auto Policy?

The insured collects from his or her own insurer, and the insurer can recoup the loss payment from the other driver.

Which of the following statements about an excess insurance plan is true?

The insurer does not participate in a loss until it exceeds the amount the firm has decided to retain.

Primary vs. Secondary on Auto Insurance

The normal rule is that liability insurance on the borrowed car is primary and any other insurance is considered excess(secondary)

All of the following are disadvantages of noninsurance transfers EXCEPT

The only potential losses that can be transferred are those that are not commercially insurable

A group of farmers agreed that if any farmer suffered a property loss, the loss would be spread over the entire group. In this way, each farmer is responsible for the average loss of the group rather than the actual loss that the farmer sustained. Which characteristic of insurance is embodied in this agreement? A) pooling of losses B) fortuitous losses C) risk avoidance D) indemnification

a

A group of farmers agreed that if any farmer suffered a property loss, the loss would be spread over the entire group. In this way, each farmer is responsible for the average loss of the group rather than the actual loss that the farmer sustained. Which characteristic of insurance is embodied in this agreement? a. pooling of losses b. fortuitous losses c. risk avoidance d. indemnification

a

A pure risk is defined as a situation in which there is A) only the possibility of loss or no loss. B) only the possibility of profit. C) a possibility of neither profit nor loss. D) a possibility of either profit or loss

a

Williams Company installed smoke detectors, a sprinkler system, and fire extinguishers in its new manufacturing facility. These devices are all examples of A) loss control. B) noninsurance transfer. C) risk avoidance. D) risk retention.

a

Williams Company installed smoke detectors, a sprinkler system, and fire extinguishers in its new manufacturing facility. These devices are all examples of a. loss control b. non-insurance transfer c. risk avoidance d. risk retention

a

Which of the following is a covered person under the medical payments coverage of the PAP?

a family member of the named insured if struck by an auto while crossing the street

All of the following are covered autos under the liability section of the PAP EXCEPT

a nonowned van which is driven by the insured on a regular basis.

All of the following are considered to be uninsured vehicles for purposes of the uninsured motorists coverage of the PAP EXCEPT

a vehicle owned by an individual who purchased enough liability insurance to satisfy the state's financial responsibility law

All of the following are programs to insure fundamental risks EXCEPT A) federally subsidized flood insurance. B) auto physical damage insurance. C) Social Security. D) unemployment insurance.

b

Bay Gallery owns a building a Wharf Street. Bay Gallery purchased a Commercial General Liability Policy with no endorsements. All of the following claims would be covered under the policy EXCEPT

an employee of the gallery accidentally started a fire that caused extensive damage to the building

Dennis was involved in an accident. He believes the damage to his auto is $6,000. His insurer believes the damage is only $3,500. Which PAP provision is designed to handle disputes between the insurer and the insured over the amount of the loss?

appraisal provision

Abandoning an existing loss exposure is an example of

avoidance

Bev lives in the suburbs and works downtown. She drives to work, and her most direct route to work would require her to pass through an area where carjackings and drive-by-shootings are common. Bev does not drive through this area. Instead, she uses a route which adds 10 minutes to her commute. Which risk management technique is Bev using with respect to the risk of injury while driving through the dangerous area?

avoidance

All of the following are programs to insure fundamental risks EXCEPT a. federally subsidized flood insurance b. auto physical damage insurance c. social security d. unemployment insurance

b

Which of the following is a form of casualty insurance a. fire insurance b. general liability insurance c. inland marine insurance d. ocean marine insurance

b

Which of the following types of loss exposures may be appropriately handled through the purchase of insurance? 1. high-frequency, low-severity 2. low frequency, high-severity a. 1 only b. 2 only c. both 1 and 2 d. neither 1 or 2

b

Which of the following types of risks best meets the requirements for being insurable by private insurers? A) market risks B) property risks C) financial risks D) political risks

b

Which of the following types of risks best meets the requirements for being insurable by private insurers? a. market risks b. property risks c. financial risks d. political risks

b

XYZ Insurance Company writes coverage for most perils which can damage property. XYZ, however, does not write flood insurance on property located in flood plains. Which requirement of an ideally insurable risk might be violated if XYZ wrote flood insurance on property located in flood plains? A) There must be a large number of similar exposure units. B) The loss should not be catastrophic. C) The chance of loss must be calculable. D) The losses must be determinable and measurable

b

XYZ Insurance Company writes coverage for most perils which can damage property. XYZ, however, does not write flood insurance on property located in flood plains. Which requirement of an ideally insurable risk might be violated if XYZ wrote flood insurance on property located in flood plains a. there must be a large number of similar exposure units b. the loss should not be catastrophic c. the chance of loss must be calculated d. the losses must be determinable and measurable

b

A phrase that encompasses all of the major risks faced by a business firm is A) financial risk. B) speculative risk. C) enterprise risk. D) pure risk.

c

A phrase that encompasses all the major risks faced by a business firm is a. financial risk b. speculative risk c. enterprise risk d. pure risk

c

A risk manager is concerned with 1. identifying potential losses 2. selecting the appropriate techniques for treating losses a. 1 only b. 2 only c. both 1 and 2 d. neither 1 or 2

c

A useful measure of an organization is the total of the organizations expenditures for treating loss exposures including retained losses, loss control expenses, insurance premiums, and other related expenses. This measure is called the organization's a. cost of capital b. cost of goods sold c. cost of risk d. cost of equity

c

ABC Insurance Company calculated the amount that it expected to pay in claims under each policy sold. Rather than selling the insurance for the amount it expected to pay in claims, ABC added an allowance to cover the cost of doing business, including commissions, taxes, and acquisition expenses. This allowance is called a(n) A) policyowner dividend. B) premium. C) expense loading. D) rate credit

c

ABC Insurance Company plans to sell homeowners insurance in five Western states. ABC expects that 8 homeowners out of every 100, on average, will report claims each year. The variation between the rate of loss that ABC expects to occur and the rate of loss that actually does occur is called A) objective probability. B) subjective probability. C) objective risk. D) subjective risk.

c

ABC Insurance Company plans to sell homeowners insurance in five Western states. ABC expects that 8 homeowners out of every 100, on average, will report claims each year. The variation between the rate of loss that ABC expects to occur and the rate of loss that actually does occur is called a. objective probability b. subjective probability c. objective risk d. subjective risk

c

ABC insurance company calculated the amount that it expected to pay in claims under each policy sold. Rather than selling the insurance for the amount it expected to pay in claims, ABC added an allowance to cover the cost of doing business, including commissions, faces, and acquisition expenses. This allowance is called a a. policy dividend b. premium c. expense loading d. rate credit

c

Which statement about a company's cost of risk is (are) true? 1. cost of risk includes insurance premiums and retained losses 2. reducing the cost of risk increases profitability a. 1 only b. 2 only c. both 1 and 2 d. neither 1 or 2

c

according to the law of large numbers, what should happen as an insurer increases the number of units insured? a. the amount the insurer expects to pay in claims should decrease b. underwriting expenses should decrease c. actual results will more closely approach expected results d. the insurer's profitability should become more variable

c

*Larry has $25,000 of bodily injury liability coverage under his PAP. This limit is the maximum amount required by his state to be considered financially responsible. While on a vacation, Larry visited a neighboring state which has a minimum financial responsibility limit of $50,000 for bodily injury. Which of the following statements describes the situation for Larry while he was in the neighboring state?*

c) Larry's policy automatically provided $50,000 of liability coverage.

Ex. of Conditions Policy Provisions

cancellation, subrogation, grace period, misstatement of age, lapsed policy

ABC Company purchased an ISO Commercial General Liability Policy. The agent who sold ABC the coverage noted that the coverage trigger in this policy was different from the trigger in the previous policy. The new policy covers only those claims which are first reported during the policy period, provided the event occurred after a retroactive date. This type of policy is known as

claims-made coverage

All of the following are potential advantages for retention EXCEPT a. lower expenses b. increased cash flow c. encouragement of loss prevention d. protection from catastrophic losses

d

A customer was injured when a furnace exploded following its faulty installation by a heating and cooling business. For the heating and cooling business, this is an example of

completed operations liability

A-1 Electrical Service is an electrical contractor that employs 14 electricians. A-1 faces many loss exposures. One general liability exposure arises out of faulty work that A-1 electricians may perform for customers at their homes or businesses. This liability exposure is known as

completed operations liability

Liability arising out of work performed by independent contractors is referred to as

contingent liability

All of the following are risk management objectives prior to the occurrence of loss EXCEPT

continuing operations after a loss.

A useful measure for an organization is the total of the organizationȇs expenditures for treating loss exposures including retained losses, loss control expenses, insurance premiums, and other related expenses. This measure is called the organizationȇs

cost of risk.

Sources of information that can be used by a risk manager to identify pure loss exposures include all of the following EXCEPT

currency exchange rates.

) Jim and Paula Franklin started a dry cleaning business. The business may be successful or it may fail. The type of risk that is present when either a profit or loss could occur is called A) pure risk. B) subjective risk. C) nondiversifiable risk. D) speculative risk.

d

49) Alex, age 26, purchased a 20-payment whole life insurance policy. After Alex has made 20 premium payments, his life insurance policy is considered A) matured. B) reduced. C) expired. D) paid-up.

d

A restaurant owner leased a meeting room at the restaurant to a second party. The lease specified that the second party, not the restaurant owner, would be responsible for any liability arising out oft the use of the meeting room, and that the restaurant owner would be "held harmless" for any damages. The restaurant owner's use of the hold-harmless agreement is an example of a. retention b. self insurance c. insurance d. non-insurance transfer

d

Which of the following statements about financial is (are) true? 1. Enterprise risk does not include financial risk 2. Financial risk is easily addressed through the purchase of insurance a. 1 only b. 2 only c. both 1 and 2 d. neither 1 or 2

d

A car damaged in an auto accident may have reduced market or resale value after it is repaired. Some insureds have sought to recover this reduction in market or resale value. This loss in value is called

diminution

Executives of a Fortune 500 firm blocked an acquisition attempt by a larger company. When the stockholders learned they could have made large profits had the acquisition not been blocked, they filed a lawsuit against the executives of the firm. What type of liability insurance protects the executives of the organization against such suits?

directors and officers liability insurance

Jennifer was just asked to serve on the Board of an insurance company. Jennifer is concerned that if the company is mismanaged, the policyowners, stockholders, and employees might sue the management team. What type of liability insurance will protect Jennifer from such claims if she accepts a position on the insurance company's Board?

directors and officers liability insurance

Ted Thomas is risk manager of XYZ Company. He is concerned his company might be sued by current or former employees alleging wrongful termination, failure to promote, racial or gender discrimination, or sexual harassment. What type of liability insurance can Ted purchase to cover these types of claims?

employment-related practices liability insurance

Which of the following types of loss exposures are best handled by the use of avoidance?

high-frequency, high-severity

Which of the following is least likely to occur during a hard insurance market period?

higher insurer profits

Cal was just hired as XYZ Companyȇs first risk manager. Cal would like to employ the risk management process. The first step in the process Cal should follow is to

identify potential losses faced by XYZ Company.

Ryan decided to review his personal risk management program. His car is 10 years old, and he would receive little money from his insurer if the car was damaged or destroyed. Ryan decided to drop the physical damage insurance on the car. From a risk management perspective, dropping the physical damage insurance on the car is best described as

increasing the use of retention in the risk management program

In which of the following situations would medical payments be paid under an unendorsed PAP?

injuries incurred in an auto while it is used in a share-the-expense car pool

Elimination Period

is a stated period of time at the beginning of a loss during which no insurance benefits are paid

Parker Department Stores has been hurt in recent months by a large increase in shoplifting losses. Parkerȇs risk manager concluded that while the frequency of shoplifting losses was high, the severity is still relatively low. What is (are) the appropriate risk management technique(s) to apply to this problem?

loss control and retention

An insurance policy specifically written and designed to meet the needs of an insurance purchaser is called a(n)

manuscript policy.

The worst loss that could ever happen to a firm is referred to as the

maximum possible loss.

A restaurant owner leased a meeting room at the restaurant to a second party. The lease specified that the second party, not the restaurant owner, would be responsible for any liability arising out of the use of the meeting room, and that the restaurant owner would beheld harmless for any damages. The restaurant owner's use of the hold-harmless agreement is an example of

noninsurance transfer.

Owen rents an apartment and parks his car in the lot behind the apartment building. A hailstorm occurred one evening, and his car was severely damaged. Which PAP coverage, if Owen purchased it, would cover this damage to his auto?

other-than-collision

Barb, who is self-employed, is the main breadwinner for her family. Barb does not have disability income insurance because she has never stopped to consider the impact of a long-term disability upon her family. Barbȇs treatment of the risk of disability is best described as

passive retention.

HRC Company purchased an unendorsed ISO Commercial General Liability (CGL) policy. Which of the following claims would be covered under the CGL?

personal and advertising injury liability

Which of the following exposures is covered under a Commercial General Liability Policy with no endorsements?

personal injury liability

Brenda identified all of the pure loss exposures her family faces. Then she analyzed these loss exposures, developed a plan to treat these risks, and implemented the plan. The process Brenda conducted is called

personal risk management.

Mark owns a 1998 sedan. The last time Mark renewed his auto insurance, he decided to drop the physical damage insurance on this vehicle. How is Mark dealing with the auto physical damage exposure in his personal risk management program?

planned retention

The worst loss that is likely to happen is referred to as the

probable maximum loss.

Loss frequency is defined as the

probable number of losses that may occur during some period.

Loss severity is defined as the

probable size of the losses which may occur during some period

Ken purchased a PAP with liability limits of 100/300/50, medical payments coverage, and collision coverage. Ken fell asleep while driving late at night. He crossed the center line and hit a car approaching from the other direction. The following losses occurred. —The driver of the other car suffered $30,000 in bodily injuries. —Ken's car sustained $5,000 in damages. —Ken incurred $5,000 in medical expenses. —The car that Ken hit was a total loss. Which of Ken's Personal Auto Policy (PAP) coverages will cover the damage to the car that Ken hit?

property damage liability

Primary and Excess Insurance

provision, the primary insurer pays first, and the excess insurer pays only after the policy limits under the primary policy are exhausted

Members of Mid-South Petroleum Distributors, a trade group, had trouble obtaining affordable pollution liability insurance. The members formed a group captive that is exempt from many state laws that apply to other insurers. This group captive is called a(n)

risk retention group.

One physical damage coverage alternative under the Business Auto Policy provides coverage for losses caused by certain named perils, such as fire, lightning, or explosion; theft; windstorm, hail or earthquake; flood; vandalism or mischief; or the sinking, burning, collision or derailment of any conveyance transporting the covered auto. This physical damage coverage alternative is

specified causes-of-loss coverage

All of the following are covered as supplementary payments under the liability section of the PAP EXCEPT

the cost of a bail bond for a traffic violation when no accident is involved

All of the following are insured persons under the liability coverage of the PAP EXCEPT

the former spouse of the named insured who moved out of the home 2 years ago when the divorce was finalized.

Pen perils or Special Coverage

where all losses are covered except those losses specifically excluded

Named Perils Coverage

where only those perils specifically named in the policy are covered

Risk management is concerned with

the identification and treatment of loss exposures.

Straight Deductible

the insured must pay a certain number of dollars of loss before the insurer is required to make a payment ex. Auto insuarnce

Which of the following statements about physicians, surgeons, and dentists liability insurance is true?

the policy covers the physician for the negligent acts of an employee

What is the purpose of the extended nonowned liability coverage endorsement to the PAP?

to provide liability coverage for an insured who operates a nonowned auto on a regular basis

Patricia purchased a Personal Auto Policy (PAP). Her car was rear-ended by a driver who fled the scene. Patricia suffered whiplash, migraine headaches, and she was unable to work. Which of the following coverages will cover her lost work earnings?

uninsured motorists

15) Under what type of rate regulation are insurers required to obtain approval of rates before using them if the rate change exceeds a specified predetermined range? A) flex-rating law B) prior-approval law C) file-and-use law D) use-and-file law

A

15) Which of the following statements about joint underwriting associations for insuring high-risk drivers is (are) true? I. Underwriting losses are proportionately shared by all auto insurers based on premiums written in the state. II. Each company participating in a joint underwriting association sets it own rates. A) I only B) II only C) both I and II D) neither I nor II

A

17) Liability arising out of all of the following are covered under the personal injury endorsement to the homeowners policy EXCEPT A) bodily injury. B) malicious prosecution. C) slander. D) invasion of privacy.

A

17) Which of the following forms is used to insure building that are under construction? A) builders risk coverage form B) floor plan coverage form C) new construction coverage form D) labor and materials coverage form

A

17) Which of the following statements about the additional coverages under Section I of the Homeowners 3 policy is true? A) There is coverage for the reasonable cost incurred to protect property from further damage after a covered loss occurs. B) There is unlimited coverage for furnishings of the landlord in an apartment on the premises that is rented to others. C) Property removed from the premises because it is endangered by an insured peril is covered on a named-perils basis while it is outside the insured premises. D) Damage caused by the settling of the insured structure is covered as a collapse.

A

18) Liability arising out of all of the following are covered under the personal injury endorsement to the homeowners policy EXCEPT A) property damage. B) false arrest. C) libel. D) slander.

A

19) All of the following are covered perils under the personal property coverage (Coverage C) of an unendorsed Homeowners 3 policy EXCEPT A) earthquake. B) theft. C) windstorm. D) explosion.

A

19) All of the following statements about business income insurance are true EXCEPT A) Business income is defined as total sales that would have been made plus continuing normal operating expenses. B) Payroll is considered a continuing normal operating expense. C) Business income insurance does not cover the physical damage caused by a peril which created the interruption in business D) The business income coverage form can be purchased with a coinsurance requirement

A

19) Bill is attempting to determine how much life insurance to purchase. He has two dependent children and his wife does not work outside of the home. An advisor suggested that Bill should consider Social Security benefits when doing his life insurance planning. One concern in this regard is the period after Social Security benefits to a widow terminate until they resume again. This period is called the A) blackout period. B) dependency period. C) emergency period. D) readjustment period.

A

2) Defects of financial responsibility laws include which of the following? I. Accident victims may not be fully compensated for their injuries. II. They apply only to property damage losses, not to bodily injury claims. A) I only B) II only C) both I and II D) neither I nor II

A

2) Which of the following would not appear in the asset section of an insurance company's balance sheet? A) loss reserves B) bonds C) common stock D) real estate

A

20) Which of the following statements about endorsements to the homeowners policy is (are) true? I. The purpose of the inflation guard endorsement is to help prevent home owners from being under-insured. II. The purpose of the watercraft endorsement is to provide coverage for watercraft when they are used outside the United States. A) I only B) II only C) both I and II D) neither I nor II

A

21) A state law that requires individuals who have been involved in an auto accident or who have been convicted for certain vehicle-related offenses to demonstrate the ability to pay liability claims up to a specified dollar amount is called a(n) A) financial responsibility law. B) compulsory insurance law. C) unsatisfied judgment fund law. D) "no pay, no play" law.

A

21) Spoilage because of loss of refrigeration at a frozen foods processor would be covered under A) equipment breakdown insurance B) extra expense insurance C) business income insurance D) inland marine insurance

A

21) Which of the following statements about covered perils under the personal property coverage (Coverage C) of the Homeowners 3 policy is true? A) Theft losses to boats and watercraft are covered only if they occur at the insuredʹs residence. B) Damage from an explosion is covered only if the explosion is the result of fire or lightning. C) Property damage from vehicles is covered only if it is caused by the insured. D)Smoke damage is covered only if it arises from agricultural or industrial operations.

A

22) The portion of an insurance premium allocated to expenses, profit, and a margin for contingencies is called the A) loading. B) pure premium. C) gross premium. D) experience rate.

A

22) Which of the following is a principal method of ensuring the solvency of insurers? A) requiring submission of annual financial statements to state regulators B) tracking and investigating market conduct complaints against insurers C) disciplining agents of the insurer for illegal sales practices D) regulating the forms (applications and policies) employed by the insurer

A

22) Which of the following statements about an earthquake endorsement for a homeowners policy is (are) true? I. The endorsement also covers landslides and volcanic eruptions. II. The coverage for earthquakes is written without a deductible. A) I only B) II only C) both I and II D) neither I nor II

A

23) What happens to the premiums for yearly renewable term insurance as an insured gets older? A) They increase at an increasing rate. B) They increase at a decreasing rate. C) They decrease at a constant rate. D) They remain level.

A

24) All of the following statements about the interest settlement option are true EXCEPT A) The minimum guaranteed interest rate is usually equal to the prime rate. B) The interest can be paid monthly, quarterly, semiannually, or annually. C) The beneficiary may be allowed to withdraw part or all of the proceeds. D) The beneficiary may be allowed to change to another settlement option.

A

27) All of the following are considered to be means of transportation and communication for purposes of inland marine insurance EXCEPT A) trains B) tunnels C) transmission lines D) television towers

A

27) Janet was unable to obtain auto insurance in the voluntary insurance market. She was contacted by the state insurance department and notified that XYZ Insurance would be her insurer. Eric was also unable to obtain auto insurance, and he received a similar notice. ABC Insurance would be his insurer. The mechanism used in Janet's and Eric's state to provide auto insurance to high-risk drivers is a(n) A) automobile insurance plan. B) joint underwriting association. C) specialty automobile insurer. D) reinsurance facility.

A

27) Mutual Property Insurance Company has a surplus of $2 million. According to a conservative rule, how much new net premiums can Mutual Property Insurance Company safely write? A) $2 million B) $8 million C) $10 million D) $20 million

A

28) Fly-By-Night Insurance Company had much larger losses than forecast. The company did not charge adequate premiums nor did the company purchase reinsurance. If Fly-By-Night becomes insolvent, which of the following will help pay the unpaid claims of the insurer? A) guaranty fund B) premium rebates C) risk-based capital D) admitted assets

A

29) All of the following are duties of the mortgagee under the standard mortgage clause EXCEPT A) to reimburse the insurer for any loss payments. B) to notify the insurer of any change in ownership of the property of which the mortgagee is aware. C) to provide a proof of loss form if the insured fails to do so. D) to pay the premium if the insured fails to do so.

A

29) Grace is a life insurance agent. She is attempting to sell a large life insurance policy, but the prospective purchaser is having second thoughts. To persuade the prospective purchaser, Grace said, "I will earn a $1,000 commission if you buy this policy. I'll give you $500 of my commission if you buy the policy." In most states, what illegal sales practice will Grace be guilty of if she splits her commission with the purchaser? A) rebating B) churning C) twisting D) backdating

A

33) Nathan was hired as an actuary with ABC Insurance. Nathan was asked to calculate the annual premium for a new product and to explain his calculations to ABC's director of ratemaking. Nathan calculated the pure premium and presented this value as the final premium. After Nathan's presentation, the director of ratemaking said, "You left out something very important. If we sell coverage at the pure premium rate, we'll be out of business soon." What did Nathan overlook in his calculations? A) loading for expenses B) the underwriting cycle C) seasonality of claims D) investment income

A

33) Which of the following statements about endowment insurance policies is (are) true? I. The face amount is paid if the insured dies during the policy period or at the end of the policy period if the insured is still alive. II. The use of endowment insurance has increased in recent years because of its favorable tax treatment. A) I only B) II only C) both I and II D) neither I nor II

A

35) Bettyʹs personal property is insured for $100,000 under her Homeowners 3 policy. If she usually keeps some personal property at a mountain cabin that she owns, how much coverage for this property is available under her homeowners policy? A) $10,000 B) $20,000 C) $40,000 D) $50,000

A

36) A manufacturing company just hired a new risk manager, and she has instituted several employee safety programs. She has persuaded the insurer writing the company's workers compensation insurance to base the premium on the company's actual loss experience during the current period rather than on the company's historical performance. This type of plan is called a(n) A) retrospectively rated plan. B) class rated plan. C) experience rated plan. D) judgment rated plan.

A

36) In Maryland, drivers who are unable to obtain auto insurance in the voluntary market are insured through a(n) A) state fund. B) reinsurance facility. C) joint underwriting association D) automobile insurance plan.

A

36) Inter-Ocean Transfer owns 12 large cargo ships which transport goods. Inter-Ocean can obtain physical damage insurance on these vessels by purchasing A) hull insurance B) cargo insurance C) protection and indemnity insurance D) freight insurance

A

37) All of the following statements describe the flexibility available to the owner of a universal life insurance policy EXCEPT A) Policy loans are permitted on an interest-free basis. B) The frequency of premium payments can be varied. C) The death benefit can be increased with evidence of insurability. D) Premium payments can be any amount provided there is sufficient cash value to keep the policy in force.

A

39) Bruce lied about his health history when he purchased a life insurance policy. He died 3 years after the policy was issued. Which life insurance policy provision will require the life insurer to pay the beneficiary even though Bruce lied on the application? A) incontestable clause B) entire contract clause C) ownership clause D) change-of-plan provision

A

4) Which of the following statements about the building and personal property coverage form is (are) true? I. A limited amount of coverage is provided for pollutant cleanup and removal at the described premises if the release or discharge of the pollutant results from a covered cause of loss. II. Fire department service charges are specifically excluded because they are a normal cost of doing business. A) I only B) II only C) Both I and II D) Neither I nor II

A

4) Which of the following statements about unsatisfied judgment funds is (are) true? I. An accident victim must obtain a judgment against the motorist who caused an accident and must show the judgment cannot be collected. II. The major disadvantage is that the negligent uninsured motorist is relieved of legal liability. A) I only B) II only C) both I and II D) neither I nor II

A

40) Which of the following statements about an indeterminate-premium whole life insurance policy is (are) true? I. It permits the insurer to adjust premiums based on anticipated future experience. II. It allows policyholder dividends to be used to lower premiums. A) I only B) II only C) both I and II D) neither I nor II

A

40) Which of the following statements concerning the scheduled personal property endorsement with agreed value loss settlement? I. The endorsement provides "all-risks" coverage on the scheduled property. II. Losses under the endorsement are settled on an actual cash value basis. A) I only B) II only C) both I and II D) neither I nor II

A

5) Which of the following statements about the licensing of insurance companies is (are) true? I. A new capital stock insurer must meet minimum capital and surplus requirements, which vary by state and line of insurance. II. The licensing requirements for insurance companies are less stringent than those imposed on most other types of firms. A) I only B) II only C) both I and II D) neither I nor II

A

5) Which of the following statements about uninsured motorists coverage is true? A) An innocent motorist must establish that the uninsured motorist is legally liable. B) The amount of coverage in most states must be at least $300,000. C) Property damage is covered in all states but bodily injury is not covered in all states. D) The coverage is very expensive and beyond the means of all but wealthy insureds.

A

51) Dave purchased a life insurance policy. The policy is nonparticipating and the cash values are based on the insurer's present mortality, investment, and expense experience. After 2 years, the insurer will recalculate the premium based on the mortality, investment, and expense experience at that time. Dave purchased A) current assumption whole life. B) variable life insurance. C) universal life insurance. D) variable universal life insurance

A

54) Which statement is true concerning the economic problem of premature death in the United States? I. The economic impact of premature death of the breadwinner varies for different types of families. II. Increased life expectancy has increased the economic problem of premature death over time. A) I only B) II only C) both I and II D) neither I nor II Answer: A

A

6) Which of the following statements about the needs approach for estimating the amount of life insurance to purchase is (are) true? I. It involves an analysis of various family needs which must be met if a family breadwinner dies. II. Its use is appropriate only if a person currently has no life insurance protection.

A

8) Section II of the homeowners policy has exclusions applying to all of the following EXCEPT A) activities of the named insured's minor children. B) business activities. C) transmission of a communicable disease. D) intentional injuries.

A

1) Which of the following is (are) included in the common declarations page of a commercial package policy? I. A description of the insured property II. A listing of the causes-of-loss that are covered by the policy A) I only B) II only C) both I and II D) neither I nor II

A) I only

11) Which of the following statements about Blue Cross Plans is (are) true? I. They typically provide service benefits rather than cash benefits to members. II. They usually provide very limited benefits for hospital charges. A) I only B) II only C) both I and II D) neither I nor II

A) I only

13) Advantages of the direct response system for marketing life insurance include which of the following? I. Advertising can be specifically directed to selected markets. II. Complex products can be easily sold. A) I only B) II only C) both I and II D) neither I nor II

A) I only

13) Which of the following statements about adjustment bureaus is (are) true? I. They are frequently used to settle claims when a large number of losses occur in a given geographic location as a result of a catastrophic event. II. Their major advantage is low cost because of the use of part-time adjustors. A) I only B) II only C) both I and II D) neither I nor II

A) I only

13) Which of the following statements about property and casualty insurance company operating results is (are) true? I. An insurance company can have a combined ratio greater than 1 (or 100 percent) and still be required to pay income taxes. II. By all measures, the property and casualty insurance industry is highly profitable when compared to other industries. A) I only B) II only C) both I and II D) neither I nor II

A) I only

15) Which of the following statements about the extra expense coverage form is (are) true? I. It provides coverage for the increased cost that must be paid to continue operations during a period of restoration. II. It provides coverage for lost business income if a key customer or key supplier experiences a loss. A) I only B) II only C) both I and II D) neither I nor II

A) I only

16) Which of the following statements about the legal obligations of a property owner is (are) true? I. A property owner must inspect the premises for the benefit of an invitee and correct any unsafe conditions. II. A property owner has the right to set a trap designed to injure a trespasser. A) I only B) II only C) both I and II D) neither I nor II

A) I only

18) Which of the following statements about HMO managed care plans is (are) true? I. There is an emphasis on controlling costs. II. They usually have high deductibles. A) I only B) II only C) both I and II D) neither I nor II

A) I only

20) Purposes of the coinsurance clause in health insurance contracts include which of the following? I. To reduce premiums. II. To exclude coverage for certain medical procedures. A) I only B) II only C) both I and II D) neither I nor II

A) I only

20) Which of the following statements about a warranty in an insurance contract is (are) true? I. It is part of the insurance contract. II. Statements made by an insurance applicant are considered warranties rather than representations. A) I only B) II only C) both I and II D) neither I nor II

A) I only

24) Which of the following statements about the investments of property and liability insurers is (are) true? I. Income from investments is important in offsetting any unfavorable underwriting experience. II. Because premium income is continually being received, the investment objective of liquidity is of little importance. A) I only B) II only C) both I and II D) neither I nor II

A) I only

28) Arguments in favor of reforming the civil justice system include which of the following? I. There is often a long delay in settling lawsuits. II. Compensation awards have decreased significantly over the past two decades. A) I only B) II only C) both I and II D) neither I nor II

A) I only

29) Which of the following statements about experience rating is (are) true? I. The insured's past loss experience is used to determine the premium for the next policy period. II. Its use is generally limited to small firms whose actual experience lacks credibility. A) I only B) II only C) both I and II D) neither I nor II

A) I only

29) Which of the following types of insurance policies can usually be assigned without the insurer's consent? I. Life insurance II. Property insurance A) I only B) II only C) both I and II D) neither I nor II

A) I only

3) Which of the following statements about "open-perils" coverage is (are) true? I. All losses are covered except those losses specifically excluded. II. The burden of proof is on the insured to prove that a loss is covered. A) I only B) II only C) both I and II D) neither I nor II

A) I only

3) Which of the following statements about underwriting policy is (are) true? I. A company must establish an underwriting policy consistent with company objectives. II. Underwriting policy is usually subjective and allows the underwriter considerable flexibility with respect to lines written and forms used. A) I only B) II only C) both I and II D) neither I nor II

A) I only

39) The exterior walls, the roof, and the plumbing, heating and air conditioning systems of a residential condominium can be insured through the purchase of which of the following forms? I. Condominium association coverage form II. Condominium commercial unit-owners coverage form A) I only B) II only C) both I and II D) neither I nor II

A) I only

4) Which of the following statements about the building and personal property coverage form is (are) true? I. A limited amount of coverage is provided for pollutant cleanup and removal at the described premises if the release or discharge of the pollutant results from a covered cause of loss. II. Fire department service charges are specifically excluded because they are a normal cost of doing business. A) I only B) II only C) both I and II D) neither I nor II

A) I only

41) Which of the following statements is (are) true regarding insurance agents and insurance brokers? I. A property and liability insurance agent has the authority to bind the insurer for certain types of coverage. II. A licensed broker who is not a licensed agent has the legal authority to bind an insurer. A) I only B) II only C) both I and II D) neither I nor II

A) I only

9) Which of the following statements about group accidental death and dismemberment (AD&D) insurance is (are) true? I. The principal sum is paid if the employee dies in an accident. II. The employer usually pays at least 50 percent of the cost of voluntary AD&D coverage. A) I only B) II only C) both I and II D) neither I nor II

A) I only

34) All of the following statements about group short-term disability income plans are true EXCEPT A) Most plans have a short elimination period for accidents but cover sickness from the first day of disability. B) Disability is usually defined in terms of the employeeʹs own occupation. C) The amount of disability income benefits typically is equal to some percentage of a workerʹs normal earnings. D) Most short-term plans cover nonoccupational disability only.

A) Most plans have a short elimination period for accidents but cover sickness from the first day of disability.

8) Temporary evidence of insurance until a policy is actually issued is provided by a(n) A) binder. B) brokerage agreement. C) pre-approval form. D) endorsement.

A) binder.

*Ken purchased a PAP with liability limits of 100/300/50, medical payments coverage, and collision coverage. Ken fell asleep while driving late at night. He crossed the center line and hit a car approaching from the other direction. The following losses occurred: -The driver of the other car suffered $30,000 in bodily injuries. -Ken's car sustained $5,000 in damages -Ken incurred $5,000 in medical expenses -The car that Ken hit was a total loss.* Which of Ken's Personal Auto Policy (PAP) coverages will cover the other driver's medical expenses?

A) bodily injury liability

17) Which of the following forms is used to insure buildings that are under construction? A) builders risk coverage form B) floor plan coverage form C) new construction coverage form D) labor and materials coverage form

A) builders risk coverage form

36) R.I.P. Company manufactures herbicide and pesticide. The company had difficulty finding affordable liability insurance. R.I.P. established its own insurance company based in Bermuda for the purpose of insuring R.I.P.'s loss exposures. The company that R.I.P. formed is called a A) captive insurer. B) reciprocal insurer. C) fraternal insurer. D) holding company.

A) captive insurer.

5) A property and casualty insurer's loss reserve includes estimates for all of the following EXCEPT A) claims anticipated but not yet incurred. B) claims reported and adjusted but not yet paid. C) claims reported and filed but not yet adjusted. D) claims incurred but not yet reported to the company.

A) claims anticipated but not yet incurred.

9) All of the following will support an insurable interest for purposes of purchasing property and liability insurance EXCEPT A) close family relationship. B) potential legal liability. C) secured creditors. D) a contract right.

A) close family relationship.

38) A personal lines insurance company initiating banking and investment services for its policyowners demonstrates which financial services industry trend? A) convergence B) demutualization C) mass merchandising D) consolidation

A) convergence

1) The portion of a property and liability insurance contract that contains information about the property or activity to be insured is called the A) declarations. B) insuring agreement. C) exclusions. D) conditions.

A) declarations.

36) The purpose of a coordination-of-benefits provision in group health insurance plans is to A) determine which plan pays first if more than one plan covers a loss. B) determine which health care provider an insured may use for his or her care. C) determine if the calendar-year deductible has been satisfied by the insured. D) determine if the employee is eligible for coverage under the group health plan.

A) determine which plan pays first if more than one plan covers a loss.

11) One of the purposes that deductible are used in insurance policies is to A) eliminate coverage for small claims. B) place restrictions or limits on the insurer's promise to perform. C) provide broader coverage by increasing the number of perils covered. D) exclude perils that are not insurable.

A) eliminate coverage for small claims.

13) Which of the following may give rise to imputed negligence? A) employer-employee relationships B) injury to a trespasser C) sole proprietorships D) attractive nuisance situations

A) employer-employee relationships

28) Mark reviewed his homeowners policy. He learned that his personal property was insured on an actual cash value basis. He would like replacement cost coverage on his personal property. He contacted his agent who said, "I'll simply add an amendment to your contract that changes the basis of recovery to replacement cost." The written provision the agent was referring to is called a(n) A) endorsement. B) coinsurance clause. C) binder. D) deductible.

A) endorsement.

7) If an underwriter suspects moral hazard, the underwriter may ask an outside firm to investigate the applicant and make a detailed report to the insurer. This report is called a(n) A) inspection report. B) application. C) M.I.B. report. D) agent's report.

A) inspection report.

21) Reimbursement for spoilage of frozen food caused by failure of a refrigeration unit at a frozen foods processor can be covered under A) equipment breakdown insurance. B) extra expense insurance. C) builders risk insurance. D) difference in conditions insurance.

A) equipment breakdown insurance.

The Affordable Care Act requires all new medical expense plans to provide a comprehensive set of coverages and services. This comprehensive set of coverages and services that must be provided are called A) essential health benefits. B) dread disease benefits. C) long-term care benefits. D) respite care benefits.

A) essential health benefits.

All of the following are typical characteristics of individual medical expense coverage EXCEPT A) first-dollar coverage. B) broad coverage. C) deductibles. D) coinsurance.

A) first-dollar coverage.

42) Compensatory damages include A) general damages and special damages. B) special damages and punitive damages. C) punitive damages and general damages. D) general damages, special damages, and punitive damages.

A) general damages and special damages.

37) As an alternative to coinsurance, rate discounts can be given as the amount of insurance to value increases. This alternative is called A) graded rates. B) agreed value coverage. C) retrospective rating. D) manual rating.

A) graded rates.

36) Inter-Ocean Transfer owns 12 large cargo ships which transport goods. Inter-Ocean Transfer can obtain physical damage insurance on these vessels by purchasing A) hull insurance. B) cargo insurance. C) protection and indemnity insurance. D) freight insurance.

A) hull insurance.

51) The authority of an agent to perform all incidental acts necessary to fulfill the purposes of the agency agreement is called A) implied authority. B) declared authority. C) apparent authority. D) express authority.

A) implied authority.

15) Under certain conditions, the wrongful acts of one person can be attributed to another person. This practice is called A) imputed negligence. B) mediation. C) comparative negligence. D) strict liability.

A) imputed negligence.

21) Sarah owns a property and liability insurance agency. She is authorized to represent several insurance companies and she is compensated by commissions. Sarah's agency owns the expiration rights to the business she sells. Sarah is a(n) A) independent agent. B) exclusive agent. C) direct writer. D) insurance broker.

A) independent agent.

*A vehicle is considered a constructive total loss when:*

B) the repair cost exceeds the actual cash value

46) The assets of a property and liability insurance company are primarily A) investments such as stocks and bonds. B) loss reserves. C) plant and equipment. D) premiums paid by policyholders.

A) investments such as stocks and bonds.

38) Louise was in a hurry and tried to cross the street in the middle of the block rather than at a street corner. A car struck her. Even though Louise placed herself in danger, she may still be able to collect for her injuries if the driver had an opportunity to avoid hitting her but failed to do so. This rule is called the A) last clear chance rule. B) collateral sources rule. C) alternative dispute resolution rule. D) joint and several liability rule.

A) last clear chance rule.

10) Deductibles are not used in which of the following type of insurance? A) life insurance B) health insurance C) property insurance D) disability income insurance

A) life insurance

33) Nathan was hired as an actuary with ABC Insurance. Nathan was asked to calculate the annual premium for a new product and to explain his calculations to ABC's director of ratemaking. Nathan calculated the pure premium and presented this value as the final premium. After Nathan's presentation, the director of ratemaking said, "You left out something very important. If we sell coverage at the pure premium rate, we'll be out of business soon." What did Nathan overlook in his calculations? A) loading for expenses B) the underwriting cycle C) seasonality of claims D) investment income

A) loading for expenses

22) The portion of an insurance premium allocated to expenses, profit, and a margin for contingencies is called the A) loading. B) pure premium. C) gross premium. D) experience rate.

A) loading.

2) Which of the following would not appear in the asset section of an insurance company's balance sheet? A) loss reserves B) bonds C) common stock D) real estate

A) loss reserves

9) Heather sued Robert for injuries suffered in an automobile accident. Based upon the facts presented, the jury concluded that Heather was 40 percent at fault in the accident and Robert was 60 percent at fault. Under the common law doctrine of contributory negligence, the jury should award Heather A) nothing. B) 40 percent of her actual damages. C) 60 percent of her actual damages. D) 100 percent of her actual damages.

A) nothing.

15) At what point in time must an insured meet the coinsurance requirement in a property insurance policy in order to avoid having to pay a portion of the loss? A) only at the time of loss B) only at the time when the policy is issued C) only at the time of policy application D) both at the time when the policy is issued and at the time of loss

A) only at the time of loss

12) When must an insurable interest legally exist in property insurance for an insured to receive payment for a loss from the insurer? A) only at the time of the loss B) only at the inception of the policy C) only at the time the loss settlement process takes place D) both at the time of the loss and at the inception of the policy

A) only at the time of the loss

*The purpose of the collision damage waiver when renting an auto is to:*

A) relieve the renter of financial responsibility if the rented car is damaged or stolen

2) Which of the following is an intentional tort? A) slander B) negligence C) strict liability D) murder

A) slander

7) Damages awarded for losses that can be determined or measured are A) special damages. B) general damages. C) punitive damages. D) comparative damages.

A) special damages.

42) John Jones is risk manager of Alpha-2-Omega Company. John wants coverage for "direct physical loss" ("open perils" coverage) under the business and personal property (BPP) coverage form. Which causes-of-loss form should he select? A) special form B) basic form C) standard form D) broad form

A) special form

47) ABC Insurance Company entered into a reinsurance agreement with XYZ Reinsurance. Under the contract, XYZ Re has no liability unless ABC's loss ratio exceeds 75 percent for the year. XYZ Re agreed to pay all losses in excess of the 75 percent loss ratio. ABC Insurance Company is using reinsurance to A) stabilize profits. B) reduce the unearned premium reserve. C) provide large risk capacity. D) retire from a line or territory.

A) stabilize profits.

30) Some investors decided to start an insurance company. Each investor contributed $50,000 to raise the capital required to charter a new company. Each investor received an ownership interest in the company. The company will raise additional capital by selling ownership rights to other investors. Under this type of organization, the customer and owner functions are separate. This type of insurer is called a A) stock company. B) reciprocal exchange. C) fraternal company. D) mutual company.

A) stock company.

10) Neil needs insurance that is unavailable in the state where he lives. To obtain insurance from a nonadmitted insurer, Neil should contact a A) surplus lines broker. B) nonadmitted agent. C) general agency broker. D) direct writer.

A) surplus lines broker.

5) All of the following are elements of negligence EXCEPT A) the ability to pay damages. B) the failure to perform a legal duty to use reasonable care. C) damages or injuries to a claimant. D) proximate cause between the negligent act and the injury or harm that occurs.

A) the ability to pay damages.

3) Which of the following is covered under the building and personal property coverage form? A) the insured's stock and inventory when located inside the insured building B) vehicles owned by the insured C) aircraft and watercraft owned by the insured D) a manufacturing building owned by a key customer or key supplier of the insured.

A) the insured's stock and inventory when located inside the insured building

49) Under a dram shop law, a business may be held liable for damages resulting from A) the sale of alcohol. B) the creation of an unsafe work place. C) the sale of prescription drugs. D) the creation of a hostile work environment.

A) the sale of alcohol.

*All of the following losses are excluded under Part D (coverage for damage to your auto) of an unendorsed PAP EXCEPT*

B) theft of a compact disc player which was permanently installed in the auto

16) The principle of utmost good faith is supported by all of the following legal doctrines EXCEPT A) representations. B) warranty. C) subrogation. D) concealment.

C) subrogation.

11) Which of the following statements is (are) true concerning the value reporting form? I. Failure to report accurately suspends coverage. II. If the insurer is able to prove that business personal property, including inventory, was under-reported before a loss, recovery is limited to the proportion that the last value reported bears to the value that should have been reported. A) I only B) II only C) Both I and II D) Neither I nor II

B

11) Which of the following statements regarding watercraft liability under Section II of an unendorsed HO-3 policy is (are) true? I. No coverage is available for watercraft liability. II. Liability arising out of the use of watercraft that are under certain length and horsepower limits is covered. A) I only B) II only C) both I and II D) neither I nor II

B

17) Sarah is using the needs approach to determine how much life insurance to buy. Her cash needs are $30,000; her income needs are $140,000; and special needs are $100,000. Sarah has the following assets: $20,000 in bank accounts, $30,000 in retirement plans, and $40,000 in investment accounts. Sarah owns no individual life insurance. She is covered by a $50,000 group life insurance policy through her employer. Based on this information, how much additional life insurance should Sarah purchase? A) $80,000 B) $130,000 C) $150,000 D) $160,000

B

17) Which of the following statements about automobile insurers that specialize in insuring high-risk motorists with poor driving records is (are) true? I. The actual premium paid by an applicant is set by the federal government and is independent of an applicant's driving record. II. The coverages available may be more limited than those written by insurers in the standard market. A) I only B) II only C) both I and II D) neither I nor II

B

18) Which of the following is an advantage of federal regulation of insurance over state regulation of insurance? A) greater opportunity for innovation B) more effective treatment of systemic risk C) greater responsiveness to local needs D) more competent regulators

B

18) Which of the following statements about the condominium commercial-unit owners coverage form is (are) true? I. It provides coverage for the unit owner's proportionate financial interest in the condominium building. II. It provides coverage for fixtures and improvements which are part of the building and owned by the unit owner. A) I only B) II only C) Both I and II D) Neither I nor II

B

19) All of the following statements about business objectives in designing a rating system are true EXCEPT A) The rating system should encourage loss control activities. B) The rating system should be independent of long-run changes in economic conditions. C) The rating system should be simple to understand. D) The rating system should be stable over short periods so that consumer satisfaction can be maintained.

B

2) The right of the states to regulate the business of insurance was first established by A) the South-Eastern Underwriters Association case. B) Paul v. Virginia. C) the Financial Modernization Act. D) the Sherman Act.

B

2) Which of the following items is (are) contained in the common policy conditions page of the commercial package policy? I. The policy period. II. A provision describing the insurer's right to audit the insured's books and records. A) I only B) II only C) Both I and II D) Neither I nor II

B

22) Bill was severely injured by an uninsured driver. Bill did not purchase uninsured motorists coverage, and the other driver, although held liable, could not pay the damages awarded. After exhausting all sources of recovery, Bill obtained some relief from a state fund designed to compensate individuals like Bill. These state funds are called A) guaranty funds. B) unsatisfied judgment funds. C) rainy-day funds. D) second injury funds.

B

23) Which of the following statements about judgment rating is true? A) It involves the manual rating of exposures. B) It is used when the loss exposures are so diverse that a class rate cannot be calculated. C) It is a form of experience rating. D) It is only used when credible loss statistics are available.

B

23) Which of the following statements describes a Difference in Conditions (DIC) Policy? A) It is a type of commercial umbrella liability policy. B) It is an all-risk policy that covers perils not insured by basic property insurance contracts. C) It is designed to cover indirect losses for which the insured has direct damage coverage. D) It is used to settle disputes when an insured has more than one insurance policy with differing policy provisions.

B

24) In which of the following did the Court decide that insurance was interstate commerce when conducted across state lines, and therefore was subject to federal regulation? A) Paul v. Virginia B) South-Eastern Underwriters Association case C) McCarran-Ferguson Act D) Financial Modernization Act

B

24) Which of the following statements about the scheduled personal property endorsement to the homeowners policy is (are) true? I. It provides named-perils coverage on scheduled items. II. It can be used to insure valuable items such as jewelry, silverware, and coin collections. A) I only B) II only C) both I and II D) neither I nor II

B

25) Which of the following statements about the replacement cost provision of the Homeowners 3 policy is true? A) It applies to personal property losses only. B) Except for small losses, the insured must repair or replace the damaged property in order to receive full replacement cost. C) The insured is required to carry an amount of insurance equal to 100 percent of the replacement value of the insured property. D) Loss settlements are equal to 50 percent of the value of the loss if the insured is carrying less than the required amount of insurance.

B

26) Alan has been involved in three accidents and has been ticketed for a number of driving violations. He tried to purchase auto insurance, but three insurers refused to sell him coverage. The generic name for state plans designed to insure drivers like Alan is the A) exchange market. B) residual market. C) voluntary market. D) excess market.

B

26) Ratemakers at ABC Insurance Company calculated the pure premium to be $280 for a risk they were considering insuring. What is the gross rate for this risk, assuming a 30 percent expense ratio? A) $364 B) $400 C) $430 D) $520

B

26) Which of the following statements about a decreasing term insurance policy is true? A) The face amount of the policy decreases during the policy period, and the premium increases. B) The face amount of the policy decreases during the policy period, but the premium remains level. C) The premium decreases during the policy period, but the face amount remains constant. D) Both the premium and the face amount of the policy decrease gradually over the policy period.

B

26) Which of the following statements about ocean marine insurance is true? A) It usually covers the peril of war B) Hull insurance includes collision liability covering the ship's owner if the ship collides with another ship. C) Protection and indemnity (P&I) insurance covers the shipper of goods for cargo losses D) A particular average loss is a loss incurred only if a ship is totally lost or destroyed.

B

26) XYZ Mutual Insurance Company has total assets of $10 million. The policyholders' surplus is $2 million. What are XYZ Mutual's total liabilities? A) $4.0 million B) $8.0 million C) $10.0 million D) $12.0 million

B

27) The purchase of term insurance is justified by which of the following circumstances? I. The insured wants to save money through the policy for a specific need. II. The insured has a temporary need for life insurance protection. A) I only B) II only C) both I and II D) neither I nor II

B

27) Which of the following statements is (are) true about the loss ratio method of class rating? I. A premium is calculated, and the pure premium is loaded to cover expenses, profit, and contingencies. II. The actual loss ratio is compared to the expected loss ratio, and the rate is adjusted accordingly. A) I only B) II only C) both I and II D) neither I nor II

B

28) In the state where Susan lives, drivers whom private insurance companies do not want to insure are placed in a common pool, and each insurer pays its pro rata share of pool losses and expenses. A common policy is used for all high-risk drivers. Susan's state handles high-risk drivers through a(n) A) automobile insurance plan. B) joint underwriting association. C) pure no-fault plan. D) specialty auto insurer.

B

29) A firm wishing to insure a single shipment of merchandise sent by a common carrier would purchase a(n) A) annual transit policy B) trip transit policy C) bailee's liability policy D) equipment floaters

B

3) All of the following information is needed to calculate a person's human life value EXCEPT A) the person's average annual earnings over his or her productive lifetime. B) the person's estimated annual Social Security benefits after retirement. C) the person's cost of self-maintenance. D) the number of years from the person's present age to the expected retirement age.

B

3) All of the following statements about the Homeowners 6 (unit-owners form) policy are true EXCEPT A) It includes personal liability coverage. B) It covers personal property on an ʺall-risksʺ basis. C) It provides limited coverage for improvements and alterations. D) It provides limited coverage for loss assessments.

B

3) Under one method of estimating a loss reserve, the reserve is based on life expectancy, duration of disability, and similar factors. This method of estimating loss reserves is called the A) judgment method. B) tabular value method. C) loss ratio method. D) average value method.

B

30) All of the following statements about the cancellation of a Homeowners 3 policy are true EXCEPT A) The insurer may cancel a new policy for any reason if it has been in force for less than 60 days and is not a renewal policy. B) At least 100 days notice of cancellation must be given if an insurer cancels a policy for nonpayment of premium. C) A policy written for longer than 1 year can be cancelled for any reason on the anniversary date by giving the insured at least 30 days notice of cancellation. D) After a policy has been in force for at least 60 days, it can be cancelled by the insurer if the risk has increased substantially since the policy was issued.

B

30) Reggie just purchased a home. A friend told him to consider insurance coverage for natural disasters when insuring the home. Losses from all of the following perils are covered under an unendorsed Homeowners 3 policy EXCEPT A) tornadoes. B) floods. C) hurricanes. D) forest fires.

B

30) Which of the following statements about retrospective rating is true? A) The premium for the current period is determined by the loss experience in prior periods. B) The premium for the current period is determined by the loss experience during the current period. C) The premium for future periods is determined by the average loss experience for the current and previous periods. D) The premium for future periods is determined by the loss experience for the current period.

B

30) Which of the following statements about the business owners policy is (are) true? I. It is designed primarily to meet the insurance needs of large manufacturing firms. II. It is a package policy designed to meet the basic property and liability needs of an insured in a single contract. A) I only B) II only C) Both I and II D) Neither I nor II

B

31) All of the following losses are excluded from coverage under Section II of an unendorsed Homeowners 3 policy EXCEPT A) Phyllis, a medical technician, misread a lab test and the patient is suing her. B) While using his canoe, Bert hit another canoe. The other canoe sank. Bert is being sued. C) Violet runs a for-profit daycare service in her home. The parents of a child injured while in Violet's care are suing Violet. D) The homeowner's son was injured when he fell off a swing in the homeowners's yard.

B

31) All of the following statements about ordinary life insurance are true EXCEPT A) Premiums are level throughout the policy period. B) The face amount of the policy is paid if the insured lives to age 65. C) There is a build-up of cash value that can be borrowed by the policyholder. D) It offers the policyholder the flexibility to meet a wide variety of financial objectives.

B

32) Some states have enacted laws which prohibit uninsured drivers from suing a negligent driver for noneconomic damages, such as pain and suffering. These laws are called A) pure no-fault laws. B) "no pay, no play" laws. C) financial responsibility laws. D) unsatisfied judgment laws.

B

33) The regulation of insurers in areas that affect consumers, which include claims handling, underwriting, complaints, advertising, sales practices, and other trade practices is called A) solvency surveillance. B) market conduct regulation. C) combined ratio analysis. D) market share regulation.

B

33) Which of the following losses would be covered under the personal liability coverage (Coverage E) of an unendorsed Homeowners 3 policy? A) liability arising out of the manufacture and sale of illegal narcotics B) liability arising out of damage to a neighbor's property that occurred over time C) liability arising out of a business operated in the home. D) liability arising out of bodily injury to an insured

B

36) Linda wants to purchase a homeowners policy, but she has some valuable personal property to internal policy limits apply. Her agent said that she could obtain coverage under her homeowners by attaching a list of this valuable property with specific amounts of insurance. Such a listing is a(n) which policy called A) binder. B) schedule. C) application. D) warranty.

B

37) Which of the following is a method used to help ensure the solvency of insurers? A) commercial lines deregulation B) risk-based capital standards C) use of credit-based insurance scores D) use of no filing required rating laws

B

38) A score derived from an individual's credit history and other factors that is used by many auto and homeowners insurers for underwriting and rating purposes is called a(n) A) CLUE score. B) insurance score. C) expense ratio score. D) combined ratio score.

B

38) Cal purchased a whole life policy 6 years ago. The policy requires annual premium payments. Cal forgot to pay the premium that was due 2 weeks ago. He wonders if his life insurance is still in force. Which life insurance policy provision is designed to keep the policy in force for a short time even if the premium payment is late? A) waiting period B) grace period C) guaranteed purchase option D) reinstatement clause

B

38) Darla purchased an unendorsed Homeowners 3 policy. While the policy was in force, a fire occurred that destroyed a living room set. The living room set cost $4,000 new, but was 25 percent depreciated when the loss occurred. Replacement furniture will cost $4,400. Assuming no deductible, how much will Darla receive from her insurer? A) $3,000 B) $3,300 C) $3,400 D) $4,400

B

39) All of the following are arguments in favor of using an applicant's credit record in personal lines underwriting EXCEPT A) Most consumers have good credit records and benefit when credit history is used as a rating factor. B) Use of credit data in underwriting and rating eliminates price discrimination against minority groups when they purchase insurance from insurers. C) Underwriting and rating may be more consistent if applicants' credit histories are considered. D) There is high correlation between an applicant's credit record and future claims experience.

B

4) To calculate a human life value, it is necessary to deduct certain costs from a person's average annual earnings. These costs include A) funeral costs. B) income taxes. C) investment income. D) pension benefits after retirement.

B

41) A systemic risk is a risk that A) can be eliminated through diversification. B) can be the cause of the collapse of an entire system. C) can be insured privately. D) can be easily contained so that it does not spread. Answer: B

B

41) A whole life insurance policy in which premiums are reduced for an initial period (e.g. 3 years) and are higher thereafter is an example of a A) level-term policy. B) modified life policy. C) limited-payment whole life policy. D) variable life policy.

B

41) Becky is considering the purchase of a whole life policy on her own life. She is concerned that if she becomes disabled, paying premiums will become a burden. Which provision can Becky attach to her life insurance policy to address this concern? A) guaranteed purchase option B) waiver-of-premium provision C) accidental death benefit rider D) accelerated death benefits rider

B

43) Which of the following statements about second-to-die life insurance is (are) true? I. The insurance is a form of endowment coverage. II. The premium is lower than the combined cost of purchasing a life insurance policy on each insured. A) I only B) II only C) both I and II D) neither I nor II

B

44) All of the following statements about covered perils and loss settlements under an unendorsed Homeowners 3 policy are true EXCEPT A) The dwelling is covered on an all-risk basis. B) Personal property losses are settled on a replacement cost basis. C) Losses to the dwelling may be settled on an actual cash value basis. D) Personal property is covered on a named-perils basis.

B

55) Which of the following statements is true regarding the results of studies by LIMRA and New York Life Insurance Company on the adequacy of life insurance owned by households in the United States? I. The average household is adequately insured against the risk of premature death. II. The average household is significantly underinsured against the risk of premature death. A) I only B) II only C) both I and II D) neither I nor II

B

6) An insurance company incorporated in another state has been licensed to operate in your state. In your state, the insurer would be considered a(n) A) nonadmitted insurer. B) foreign insurer. C) alien insurer. D) reciprocal insurer.

B

6) Which of the following statements about methods for estimating loss reserves for property and casualty insurers is (are) true? I. The judgment method involves the use of a statutory formula to estimate the loss reserve. II. The average value method is used when the number of claims is large and the claims are settled quickly. A) I only B) II only C) both I and II D) neither I nor II

B

7) Bert purchased a life insurance policy 4 years ago. He inadvertently stated that he was 1 year younger than his actual age. If Bert dies today, how much will the insurance company pay? A) nothing B) less than the policy face value C) the policy face value D) more than the policy face value

B

7) One item that appears on an insurance company's financial statements is a liability that represents an estimate of the claims reported and adjusted but not yet paid, claims reported and filed but not yet adjusted, and claims incurred but not yet reported to the company. This liability is called the insurer's A) net income. B) loss reserves. C) admitted assets. D) unearned premium reserve.

B

8) A no-fault law under which benefits are paid to an accident victim without regard to fault and the accident victim can still sue the negligent driver who caused the accident is referred to as a(n) A) pure no-fault law. B) add-on no-fault law. C) modified no-fault law. D) comparative no-fault law.

B

8) What is the length of the readjustment period which is considered when the needs approach is used to determine the amount of life insurance to own? A) 3 to 6 months B) 1 to 2 years C) until the youngest child reaches age 18 D) until the surviving spouse reaches age 65

B

9) Under the needs approach, when is the dependency period of a surviving spouse assumed to end? A) 1 or 2 years after the breadwinner's death B) when the youngest child reaches age 18 C) when the surviving spouse reaches age 65 D) when the surviving spouse dies

B

47) ABC Insurance Company's investment income ratio last year was 4.2 percent. The company's combined ratio last year was 102.6 percent. What was ABC's overall operating ratio? A) 96.8 percent B) 98.4 percent C) 103.2 percent D) 106.8 percent

B) 98.4 percent

39) Which of the following statements about the combined ratio is true? A) It is equal to the loss ratio minus the expense ratio. B) A combined ratio greater than 1 (or 100 percent) means an underwriting loss has occurred. C) The combined ratio considers the company's investment income. D) A combined ratio less than 1 (or 100 percent) indicates that an underwriting loss has occurred.

B) A combined ratio greater than 1 (or 100 percent) means an underwriting loss has occurred.

10) Which of the following statements about an insurable interest in life insurance is (are) true? I. It is required of any person named as beneficiary. II. It may result from a pecuniary (financial) interest. A) I only B) II only C) both I and II D) neither I nor II

B) II only

24) Which of the following statements about ocean marine insurance coverages is (are) true? I. Hull insurance limits coverage to the breakdown of a ship's machinery and equipment. II. Protection and indemnity insurance provides liability insurance to the ship owner for bodily injury and property damage to third parties. A) I only B) II only C) both I and II D) neither I nor II

B) II only

27) The Sarbanes-Oxley Act requires which of the following? I. Accounting firms auditing a company's books must provide other major services to the firm besides auditing services. II. The company's Chief Executive Officer (CEO) must swear to the accuracy of quarterly and annual financial reports. A) I only B) II only C) both I and II D) neither I nor II

B) II only

27) Which of the following statements is (are) true about the loss ratio method of class rating? I. A premium is calculated, and the pure premium is loaded to cover expenses, profit, and contingencies. II. The actual loss ratio is compared to the expected loss ratio, and the rate is adjusted accordingly. A) I only B) II only C) both I and II D) neither I nor II

B) II only

30) Which of the following statements about the businessowners policy is (are) true? I. It is designed to meet the insurance needs of large manufacturing firms. II. It is a package policy designed to meet the basic property and liability needs of an insured in a single contract. A) I only B) II only C) both I and II D) neither I nor II

B) II only

33) Which of the following statements is (are) true with respect to the financial services industry? I. The number of life insurers operating in the industry continues to increase at a rapid rate. II. The Financial Modernization Act of 1999 permits financial institutions to compete in other financial markets outside their core business area. A) I only B) II only C) both I and II D) neither I nor II

B) II only

47) Which of the following statements is (are) true with regard to group life insurance? I. Most group life insurance is whole life coverage. II. Most group life insurance plans allow a modest amount of life insurance on the employeeʹs spouse and dependent children. A) I only B) II only C) both I and II D) neither I nor II

B) II only

6) Which of the following statements about a reciprocal exchange is (are) true? I. It usually specializes in health insurance. II. It is an unincorporated mutual insurance company. A) I only B) II only C) both I and II D) neither I nor II

B) II only

6) Which of the following statements about methods for estimating loss reserves for property and casualty insurers is (are) true? I. The judgment method involves the use of a statutory formula to estimate the loss reserve. II. The average value method is used when the number of claims is large and the claims are settled quickly. A) I only B) II only C) both I and II D) neither I nor II

B) II only

6) Which of the following statements about the provisions of the building and personal property coverage form is (are) true? I. A separate deductible must be satisfied for each building damaged in the same occurrence. II. Under the replacement cost option, there is no deduction for depreciation. A) I only B) II only C) both I and II D) neither I nor II

B) II only

8) Which of the following statements about monetary damages awarded by a court is (are) true? I. The purpose of general damages is to provide benefits for medical expenses or loss of earnings. II. The purpose of punitive damages is to punish the tortfeasor so that others are deterred from committing the same wrongful act. A) I only B) II only C) both I and II D) neither I nor II

B) II only

8) Which of the following statements about the principle of insurable interest is (are) true? I. It makes it difficult to measure the amount of an insured's loss. II. It reduces moral hazard. A) I only B) II only C) both I and II D) neither I nor II

B) II only

Which of the following statements about disability and disability income insurance is (are) true? I. Most disability income policies replace 100 percent of gross earnings. II. The probability of being disabled before age 65 is much higher than commonly believed. A) I only B) II only C) both I and II D) neither I nor II

B) II only

Which of the following statements about eligibility requirements for qualified Health Savings Accounts (HSAs) is (are) true? I. Only individuals who are eligible for Medicare benefits can establish a qualified HSA. II. Applicants must be covered by a high deductible health plan and not be covered by any other comprehensive health plan to establish a qualified HSA. A) I only B) II only C) both I and II D) neither I nor II

B) II only

Which of the following statements about individual disability income policies that use a two-part definition of total disability is (are) true? I. During the initial period of disability, the insured must be unable to perform the duties of any gainful occupation. II. After the initial period of disability, the insured must be unable to perform the duties of any occupation for which he or she is reasonably fitted by education, training, and experience. A) I only B) II only C) both I and II D) neither I nor II

B) II only

Which of the following statements about long-term care insurance is (are) true? I. Long-term care insurance is inexpensive, especially if purchased at older ages. II. Purchasers have a choice of daily benefits and benefit periods. A) I only B) II only C) both I and II D) neither I nor II

B) II only

12) Which of the following statements about claims settlement is true? A) Agents are never authorized to settle claims. B) Independent adjustors may be used in a geographic area where the volume of business is too low for an insurer to have its own adjustors. C) A public adjustor is a salaried employee who works for one insurer. D) A company adjustor is hired by a policyholder to represent him or her if the policyholder does not agree with the claim settlement offered by the insurer.

B) Independent adjustors may be used in a geographic area where the volume of business is too low for an insurer to have its own adjustors.

23) Which of the following statements describes a Difference in Conditions (DIC) Policy? A) It is a type of commercial umbrella liability policy. B) It is an all-risk policy that covers perils not insured by basic property insurance contracts. C) It is designed to cover indirect losses for which the insured has direct damage coverage. D) It is used to settle disputes when an insured has more than one insurance policy with differing policy provisions.

B) It is an all-risk policy that covers perils not insured by basic property insurance contracts.

23) Which of the following statements about judgment rating is true? A) It involves the manual rating of exposures. B) It is used when the loss exposures are so diverse that a class rate cannot be calculated. C) It is a form of experience rating. D) It is only used when credible loss statistics are available.

B) It is used when the loss exposures are so diverse that a class rate cannot be calculated.

24) All of the following are criticisms of managed care plans EXCEPT A) The quality of care may be reduced because of the emphasis on cost control. B) Preventive care is unlikely to be provided. C) Restrictions may be placed on physiciansʹ abilities to treat patients. D) Network physicians may have a financial conflict of interest between providing high-quality medical care and holding down costs.

B) Preventive care is unlikely to be provided.

38) Metro City has six different zip codes. XYZ Insurance Company markets coverages in Metro City. Any applicant who lists one particular zip code is automatically quoted a premium that is twenty percent more than the average premium for applicants from the other five zip codes, even if the loss exposure is identical. Which regulatory objective is not being met given XYZ's premium structure? A) Rates must be adequate. B) Rates must not unfairly discriminate. C) Rates must be responsive. D) Rate must not be excessive.

B) Rates must not unfairly discriminate.

36) All of the following are methods that a property and liability insurance company can use to protect against catastrophic losses EXCEPT A) sale of catastrophe bonds. B) purchase of common stock. C) purchase of excess-of-loss reinsurance. D) quota share reinsurance with a low retention percentage.

B) purchase of common stock.

11) Which of the following statements about life insurance marketing systems is true? A) Insurance brokers are not permitted to sell live insurance. B) Some exclusive agents who market property and liability insurance also sell life insurance. C) Independent insurance agents are not permitted to market life insurance. D) Financial institutions such as banks are not permitted to market life insurance.

B) Some exclusive agents who market property and liability insurance also sell life insurance.

14) Which of the following statements about subrogation is true? A) Subrogation eliminates adverse selection. B) Subrogation helps to hold down the cost of insurance. C) Subrogation results in violation of the principle of indemnity. D) Subrogation permits a party who caused a loss to avoid responsibility for the loss.

B) Subrogation helps to hold down the cost of insurance.

17) What is the legal significance of a material concealment by an insurance applicant? A) The contract is automatically voided from its inception. B) The contract is voidable at the insurer's option. C) Loss payments are reduced by the degree of the concealment. D) The insurer is immediately entitled to a higher premium.

B) The contract is voidable at the insurer's option.

18) What is the legal significance of a material misrepresentation in an insurance application? A) The contract is automatically voided from its inception. B) The contract is voidable at the insurer's option. C) Loss payments are reduced by the degree of the misrepresentation. D) The insurer is immediately entitled to a higher premium.

B) The contract is voidable at the insurer's option.

19) Which of the following statements about treaty reinsurance is true? A) The reinsurer is required to underwrite each individual applicant that is reinsured. B) The reinsurer must accept all business that falls within the scope of the treaty. C) The ceding insurer can choose which business falling within the scope of the treaty it wishes to reinsure. D) It protects the reinsurer by requiring the ceding insurer to charge adequate premiums.

B) The reinsurer must accept all business that falls within the scope of the treaty.

12) Which of the following types of forms is used to insure fluctuations in business personal property, such as inventory and finished goods? A) accounts receivable coverage form B) a value reporting form C) difference in conditions insurance D) business income insurance

B) a value reporting form

51) Under state workers compensation programs, employers may not use common law defenses to defend against claims of workers who are injured on the job. In such cases, proof of a worker's injury is proof of responsibility of the employer. Because of this characteristic, workers compensation is an example of A) personal injury. B) absolute liability. C) comparative negligence. D) tort liability.

B) absolute liability.

17) The primary purpose of coinsurance in property insurance is to A) reduce moral hazard. B) achieve equity in rating. C) minimize problems in settling claims. D) eliminate small losses.

B) achieve equity in rating.

41) Turner Company self-insures its group life and group health insurance plans. Turner entered into an agreement with ABC Insurance through which ABC handles the plan design, claims processing, and record keeping for Turner. The agreement between Turner and ABC is called a(n) A) preferred provider agreement. B) administrative services only contract. C) exclusive provider agreement. D) point-of-service contract.

B) administrative services only contract.

42) Most insurance companies require their marketing representatives to submit an evaluation of the prospective insured. This important source of underwriting information is called the A) application. B) agent's report. C) inspection report. D) physical inspection.

B) agent's report.

*Ken purchased a PAP with liability limits of 100/300/50, medical payments coverage, and collision coverage. Ken fell asleep while driving late at night. He crossed the center line and hit a car approaching from the other direction. The following losses occurred: -The driver of the other car suffered $30,000 in bodily injuries. -Ken's car sustained $5,000 in damages -Ken incurred $5,000 in medical expenses -The car that Ken hit was a total loss.* Which of Ken's Personal Auto Policy (PAP) coverages will cover the damage to Ken's car?

B) collision coverage

55) Hank bought a farm. There was an old barn on the land. One day Hank decided to take a look at the barn. It was windy, and Hank could see the roof swaying in the wind. Hank ran out of the barn fearing the roof might collapse. While driving home, Hank stopped to see his insurance agent. He insured the barn for $20,000. The agent didn't ask if the roof might collapse, and Hank didn't say anything about it. One week later there was a strong wind and the roof collapsed. Assuming the insurer can prove it, under what legal grounds could the insurer deny payment of the claim? A) estoppel B) concealment C) warranty D) misrepresentation

B) concealment

43) Congress passed the Sarbanes-Oxley Act to address which of the following problems? A) deficiencies in the tort liability system B) corporate fraud and lax corporate governance C) inefficiencies in state regulation of the insurance industry D) inadequate availability of certain insurance coverages

B) corporate fraud and lax corporate governance

46) ABC Company provides a group health insurance plan for its employees. The plan is a basic medical expense plan with supplemental major medical. While the basic plan provides first-dollar coverage, employees must pay a deductible once the basic insurance limit is met and before the supplemental major medical plan begins to provide benefits. This deductible is called a(n) A) franchise deductible. B) corridor deductible. C) straight deductible. D) calendar-year deductible.

B) corridor deductible.

41) An elimination (waiting) period is an example of a(n) A) exclusion. B) deductible. C) other-insurance provision. D) coinsurance provision.

B) deductible.

1) The function of an actuary is to A) adjust claims. B) determine premium rates. C) negotiate reinsurance treaties. D) invest insurance company assets.

B) determine premium rates.

Kevin has an individual major medical policy that his insurer agrees to keep in force until age 65. However, the company has the right to increase the premium each year for the underwriting class in which Kevin has been placed. Which renewal provision is found in Kevin's policy? A) noncancellable B) guaranteed renewable C) conditionally renewable D) nonrenewable.

B) guaranteed renewable

*In addition to providing coverage in the United States, its territories and possessions, and Puerto Rico, where else does the PAP provide coverage?*

B) in Canada

Greta purchased a long-term care policy. Greta's eligibility for benefits under the policy may be triggered by A) how long premiums have been paid. B) inability to perform activities of daily living. C) continuous hospitalization for at least 60 days. D) eligibility for Medicare benefits.

B) inability to perform activities of daily living.

Under the Affordable Care Act, if a health insurer does not meet the minimum loss ratio requirement, the insurer must A) pay a fine to the federal government. B) issue rebates to the people the insurer covered. C) not sell any health insurance for a period of one year. D) reduce the premium on the policies it sells the following year.

B) issue rebates to the people the insurer covered.

7) One item that appears on an insurance company's financial statements is a liability that represents an estimate of the claims reported and adjusted but not yet paid, claims reported and filed but not yet adjusted, and claims incurred but not yet reported to the company. This liability is called the insurer's A) net income. B) loss reserves. C) admitted assets. D) unearned premium reserve.

B) loss reserves.

25) Cathy just started a job with XYZ Manufacturing Company. She attended an orientation and was given a packet providing information about the various employee benefits XYZ offers. One item in the packet was a booklet and application form from an auto insurer. The insurer offers lower premiums to XYZ employees. The insurer's plan for selling individually-underwritten auto insurance to employees of XYZ Manufacturing Company is called A) direct response. B) mass merchandising. C) personal selling. D) multiple marketing.

B) mass merchandising.

38) When Ben applied for life insurance, he was asked on the application if he smoked or used tobacco products. Ben answered "No." In reality, Ben smokes two packs of cigarettes a day. The policy was issued at the "preferred, nonsmoker rate." If Ben dies 6 months after the policy is issued, upon what grounds will the insurer be able to legally deny the claim? A) warranty B) misrepresentation C) waiver D) concealment

B) misrepresentation

Individual major medical insurance is characterized by which of the following? A) narrow range of benefits B) no lifetime benefit limits C) no exclusions D) first-dollar coverage

B) no lifetime benefit limits

11) When must an insurable interest legally exist in life insurance? A) only at the time of the insured's death B) only at the inception of the policy C) only at the time the beneficiary is paid D) both at the time of the insured's death and at the inception of the policy

B) only at the inception of the policy

*The purpose of gap insurance is to:*

B) pay the difference between the amount the insurer pays if a car is a total loss and the amount owed on a lease or car loan

20) David is a successful independent insurance agent. Recently, one of the life insurance companies with whom he places business offered him a special financial arrangement. If David meets sales targets, he will receive large bonuses. He will also be able to recruit and train sub-agents and receive an over-riding commission based on the amount of life insurance the sub-agents sell. Based on this description, David is a(n) A) insurance broker. B) personal-producing general agent. C) direct writer. D) multiple life exclusive agent.

B) personal-producing general agent.

Prior to passage of the Affordable Care Act, insurance policies typically contained a provision excluding coverage for impairments that were present or were treated during a specified period prior to the effective date of the policy. This provision is a(n) A) time limit on certain defenses. B) preexisting-conditions clause. C) benefit period provision. D) incontestable clause.

B) preexisting-conditions clause.

35) Easy Pay Insurance Company may require insureds who suffer a loss to submit a sworn statement to substantiate that a loss occurred and to describe the conditions under which the loss occurred. This sworn statement is called a(n) A) binder. B) proof of loss. C) inspection report. D) notice of loss.

B) proof of loss.

35) Michelle had major abdominal surgery. Months after the surgery, she still did not feel well. When she was operated on again, the surgeon discovered two sponges that were not removed at the conclusion of the first operation. Michelle should be able to collect damages without having to prove negligence under the doctrine of A) joint and several liability. B) res ipsa loquitor. C) contributory negligence. D) uberrimae fidei.

B) res ipsa loquitor.

26) Brian buys and sells investment securities for his clients. Brian also decided to become a licensed life insurance agent to better serve his customers. While Brian's primary focus is buying and selling financial securities for his clients in exchange for commissions, he also earns commissions on his life insurance sales. Brian is a(n) A) financial planner. B) stock broker. C) insurance broker. D) personal-producing general agent.

B) stock broker.

*Which of the following situations would be covered by the liability section of an unendorsed PAP if the insured is legally liable?*

B) the insured backs into and damages the garage door of his rented house

11) Nancy was injured when she drove her car through a stop sign and was struck by Philip's car. Philip saw Nancy and could have stopped. However, he failed to do so since he had the right-of-way. Nancy can recover damages from Philip under which of the following legal doctrines? A) vicarious liability B) the last clear chance rule C) contributory negligence D) the assumption of risk doctrine

B) the last clear chance rule

46) The general rule that ambiguity in insurance contracts is construed against the insurer is reinforced by an important legal principle. This principle states the insured is entitled to coverage under a policy that he or she would assume the policy would provide, and exclusions must be conspicuous, plain, and clear. This principle is known as A) the principle of utmost good faith. B) the principle of reasonable expectations . C) the principle of subrogation. D) the principle of indemnity.

B) the principle of reasonable expectations .

*Angie was injured when her car was struck by a driver who ran a red light. The other driver carried the minimum liability coverage necessary to be considered financially responsible. Angie's injuries were $15,000 above the minimum bodily injury limit. There is a coverage that can be added to the PAP that applies when a negligent drive carries the minimum liability insurance required by the state, but is less than the insured's actual damages for bodily injury. This coverage is called:*

B) underinsured motorists coverage

43) Which distinct legal characteristic of insurance contracts states that only the insurer's promise to perform is legally enforceable? A) contracts of adhesion B) unilateral contracts C) aleatory contracts D) personal contracts

B) unilateral contracts

*Patricia purchased a PAP. Her car was rear-ended by a driver who fled the scene. Patricia suffered whiplash, migraine headaches, and she was unable to work. Which of the following coverages will cover her lost work earnings?*

B) uninsured motorists

39) Robin plans to open a bar in a high-crime area. She had difficulty obtaining insurance for the business. She found an insurer willing to write the coverage, but only if Robin agreed to have a security alarm system in operation at all times when the business is closed. Robin's promise to have a security alarm system operational as a condition of having the insurance coverage in force is a A) binder. B) warranty. C) waiver. D) deductible.

B) warranty.

18) Under one life insurance marketing system, individual producers conduct sales interviews at the workplace with the approval of the management of the business. There are few direct costs to the employer, and this marketing system is especially appropriate for the low-income and middle-income workers. This life insurance marketing system is called the A) direct response system. B) worksite marketing. C) independent agency system. D) personal producing general agency system.

B) worksite marketing.

17) Which of the following statements is (are) true concerning investments of property and casualty insurers and life insurers? I. Property and casualty insurance companies place greater emphasis on liquidity than do life insurers. II. Life insurance company investments are, on average, of longer duration than property and casualty insurance company investments. A) I only B) II only C) both I and II D) neither I nor II

C

18) All of the following are additional coverages under Section I of the Homeowners 3 policy EXCEPT A) the reasonable expenses of removing debris of covered property after an insured loss. B) fire department service charges for which the insured is liable by contract or agreement. C) damage to trees and shrubs caused by a windstorm. D) losses incurred from the unauthorized use of a stolen credit card.

C

19) Factors that affect the cost of homeowners insurance include which of the following? I. Construction material II. Deductible amount A) I only B) II only C) both I and II D) neither I nor II

C

10)Which of the following statements about the cause-of-loss special form of the ISO commercial package policy is (are) true? I. The cause-of-loss special form insures against risk of direct physical loss. II. Collapse is included as a additional coverage. A) I only B) II only C) Both I and II D) Neither I nor II

C

11) JKL Insurance Company reported the following information on its accounting statements last year: Premiums Written $90,000,000 Loss Adjustment Expenses $5,000,000 Underwriting Expenses $30,000,000 Premiums Earned $100,000,000 Incurred Losses $70,000,000 What was JKL's expense ratio last year? A) 5.0 percent B) 30.0 percent C) 33.3 percent D) 50.0 percent

C

11) Which of the following statements about the characteristics of current no-fault laws is true? A) Most laws in force today are pure no-fault laws. B) Most laws apply to both bodily injury and property damage. C) Most laws permit payment of survivor benefits to a surviving spouse and children. D) States with add-on plans restrict the right of accident victims to sue negligent drivers.

C

12) The term used to describe plans in which automobile insurers participate to make insurance available to drivers unable to obtain coverage in the standard market is the A) foreign market. B) fair market. C) residual (shared) market. D) high-premium market.

C

12) Which of the following statements about the regulation of life insurance companies is (are) true? I. The percentage of assets a life insurance company may invest in a specific type of asset (e.g., stocks or bonds) is generally limited by law. II. The purpose of limiting the accumulation of surplus is to prevent an insurer from increasing its surplus at the expense of policyowner dividends. A) I only B) II only C) both I and II D) neither I nor II

C

13) Which of the following statements about the business income coverage form is true? A) Business income is defined as gross earnings before taxes B) Payroll is excluded unless it is specifically added C) The form covers loss of business income and extra expenses incurred during restoration D) The form can be used by a manufacturing operation only

C

16) Which of the following statements about reinsurance facilities for insuring high-risk drivers is (are) true? I. Underwriting losses in the reinsurance facility are shared by all auto insurers in the state. II. An insurer must accept all applicants for insurance, but the insurer has the option of placing high-risk drivers in the reinsurance pool. A) I only B) II only C) both I and II D) neither I nor II

C

20) Which of the following losses to a dwelling would be covered under an unendorsed Homeowners 3 policy? A) smoke damage resulting from agricultural operations of a neighboring farmer B) damage to the structure caused by a flash flood C) damage to the structure caused by the weight of heavy snow D) damage to a floor caused by water backing up through a sewer pipe

C

20) Which of the following statements about the equipment breakdown protection coverage form is (are) true? I. The covered cause of loss is a breakdown of covered equipment, including boilers, machinery, and electrical and mechanical equipment. II. It provides coverage for the reasonable cost of expediting permanent repair or replacement of damaged property. A) I only B) II only C) Both I and II D) Neither I nor II

C

21) The major reasons for insurer insolvency include which of the following? I. Inadequate pricing and loss reserves II. Rapid growth and inadequate surplus A) I only B) II only C) both I and II D) neither I nor II

C

21) The unit of measurement used in property and casualty insurance pricing is called the A) unit rate. B) premium. C) exposure unit. D) experience unit.

C

21) Which of the following statements about buying homeowners insurance is true? A) There is no reason to carry insurance for more than 80 percent of a dwelling's replacement cost. B) The deduction for depreciation will be increased if a personal property replacement cost endorsement is purchased. C) Premiums often can be reduced substantially by selecting a higher deductible. D) There is little reason to compare cost since the lack of competition results in little price variation among companies.

C

22) One of the filed inland marine forms is the accounts receivable coverage form. In which of the following cases would the purchaser of this form be indemnified through the coverage? A) If the purchaser cannot pay amounts owed because the purchaser has become insolvent. B) If the purchaser cannot pay amounts owned because the purchaser has suffered an insured physical damage loss. C) If the purchaser cannot collect accounts receivable because of the destruction of records. D) If the purchaser cannot collect accounts receivable because the purchaser extended credit to a poor credit risks.

C

23) All of the following are general exclusions under Section I of the Homeowners 3 policy EXCEPT A) losses due to earthquake. B) losses due to the failure of the insured to use all reasonable means to save and preserve property after the time of a loss. C) losses resulting from a power failure caused by an insured peril on the residence premises. D) losses due to radiation from a nuclear power plant.

C

23) The number of title insurance companies operating in State Z is relatively low. Recently, the largest of these companies (50 percent market share) acquired the second largest company (30 percent market share). Immediately after the acquisition, the insurer raised premiums by 75 percent. This scenario demonstrates which of the following rationales for the regulation of insurance? A) maintain insurer solvency B) prohibit unfair sales practices by agents C) ensure reasonable rates D) make insurance available

C

24) Under one type of merit rating, the class or manual rate is adjusted upward or downward based on past loss history. This type of merit rating is called A) schedule rating. B) judgment rating. C) experience rating. D) retrospective rating.

C

26) Tom is planning to build a home. He is weighing many factors, including construction materials, location, and other considerations. Which of the following statements is true with regard to homeowners insurance and Tom's new home? A) The higher the construction costs are in the area, the lower Tom's homeowners insurance premiums will be. B) The lower the homeowners deductible selected, the lower the premium will be. C) Older homes cost more to insure than newer homes. D) A wood frame home costs less to insure than a brick home.

C

27) Disadvantages of life insurance settlement options include which of the following? I. Higher yields can often be obtained elsewhere. II. Life income options have limited usefulness at younger ages. A) I only B) II only C) both I and II D) neither I nor II

C

27) Robert purchased an unendorsed Homeowners 3 policy. He is concerned that if his personal property is destroyed, the insurer will take depreciation into consideration when determining the loss settlement and will pay him less than the amount needed to purchase new property. Which endorsement can Robert add to his Homeowners 3 policy to address this concern? A) inflation guard endorsement B) personal injury endorsement C) personal property replacement cost endorsement D) earthquake endorsement

C

28) All of the following statements about filed inland marine forms are true EXCEPT A) A mail coverage form covers securities in transit by first-class mail, registered or certified mail, or expressed mail B) A commercial articles coverage form is used to insure photogenic equipment and musical instruments C) A jewelers block coverage form is used by individuals in insure jewelry limited in coverage by the homeowners form D) The signs coverage form covers neon, mechanical, and electrical sign.

C

28) Which of the following statements about schedule rating is (are) true? I. It involves the determination of a basis rate for each exposure, which is then modified by credits or debits. II. It is based on the assumption that certain physical characteristics of the insured's operations will influence the insured's future loss experience. A) I only B) II only C) both I and II D) neither I nor II

C

29) Scott had trouble obtaining auto insurance. After three companies refused to insure him, he called the state insurance department. A representative suggested he obtain coverage through Last Chance Insurance Company because "that's all they insure-high-risk drivers." Scott contacted Last Chance. He was not refused coverage; however, the premium Scott was required to pay was three times greater than the average premium in the market. Last Chance Insurance Company is a(n) A) reinsurance facility. B) automobile insurance plan. C) specialty auto insurer. D) joint underwriting association.

C

3) Criticisms of compulsory insurance laws include which of the following? I. They provide less than complete protection since they require only a minimum amount of liability insurance. II. Even with compulsory insurance laws, a substantial number of motorists continue to operate vehicles without insurance. A) I only B) II only C) both I and II D) neither I nor II

C

30) Amber believes that her auto insurance premium is too high. All of the following will reduce Amber's auto insurance premium EXCEPT A) Amber could increase her physical damage deductible. B) Amber could move from the city where she lives to a rural area outside the city. C) Amber could increase the amount of liability insurance that she carries. D) Amber could improve her driving record.

C

31) Monopoly Insurance is the only company marketing a certain line of insurance in a state. After complaints from several consumers, the State Insurance Department investigated Monopoly's rates. The regulators determined that Monopoly was taking advantage of being the only insurer offering the line by charging more than double the actuarial cost of the coverage. Which regulatory rating objective was Monopoly violating? A) Rates must be adequate. B) Rates should encourage loss control. C) Rates must not be excessive. D) Rates must not unfairly discriminate.

C

32) Small Town used to be just that—a small town 6 miles from Large City. Over the years, the area between Small Town and Large City has developed, and now Small Town is part of the suburbs surrounding Large City. An auto insurer that operated in the area had a large increase in auto claims from Small Town insureds. The insurer did not adjust its rates, and this year will lose money because of claims attributable to higher population density. Which business rating objective did this insurer fail to meet? A) simplicity B) stability C) responsiveness D) encouragement of loss control

C

32) Under one type of rating law, insurers are free to change rates and to use modified rates immediately. However, the new rate must be filed with regulators within a specified period, such as 60 days after the modified rate is employed. This type of rating law is called A) prior approval. B) file-and-use. C) use-and-file. D) flex rating.

C

32) Which of the following losses would be covered under the medical payments coverage of the Homeowners 3 policy? A) injuries to another person arising out of the insured's negligent operation of a vehicle B) medical payments resulting from the transmission of a communicable disease C) injury to a resident employee at the insured's home D) workers compensation medical payments

C

32) Which of the following statements about a typical accidental death benefit rider is (are) true? I. Accidental injury must be the cause of death for the increased benefit to be paid. II. The accidental death must occur prior to some specified age for the increased benefit to be paid. A) I only B) II only C) both I and II D) neither I nor II

C

33) A credit-based score that insurers claim is highly predictive of future claims costs is an individual's A) combined ratio. B) loss ratio. C) insurance score. D) underwriting score.

C

33) Reasons for NOT purchasing an accidental death benefit rider include which of the following? I. Most people die as a result of a disease rather than from an accident. II. The economic value of a human life is not increased if death occurs because of an accident. A) I only B) II only C) both I and II D) neither I nor II

C

34) The National Association of Insurance Commissioners (NAIC) administers an "early warning system" to help ensure insurance company solvency. This system uses data provided in the annual statement to identify companies that may pose a solvency risk. This early warning system is called A) the risk-based capital requirements. B) an insurance guaranty fund. C) the Insurance Regulatory Information System (IRIS). D) the assessment method.

C

34) Tom and Tammy Evans were ready to purchase a home. The home was to serve as collateral for their mortgage loan. Two insurers declined to insure the home, citing "an adverse CLUE report." Why would an insurer reject a homeowners insurance application because of an adverse CLUE report? A) because the previous owner had defaulted on the mortgage loan B) because the home is located in an area where the zoning law had been changed C) because there had been previous property insurance claims filed on the home D) because the home is located in an area that does not have a certified fire department

C

35) One type of commercial property insurance excludes perils which are covered by the basic coverages. Some businesses buy this coverage to fill some gaps, including food and earthquake, and to cover property in other countries. This type of insurance is called A) protection and indemnity insurance B) building and personal property coverage form C) difference in conditions insurance D) builders risk coverage form

C

35) Which of the following statements about accelerated death benefits riders is (are) true? I. The benefit paid is usually less than the full-face amount. II. Several different medical conditions may trigger the payment of benefits. A) I only B) II only C) both I and II D) neither I nor II

C

35) Which of the following statements is (are) true concerning the use of an individual's credit history as an insurance rating factor? I. Individuals with poor credit histories, as a group, generally file more homeowners claims than do individuals with good credit histories. II. The use of credit history as an insurance rating factor is controversial. A) I only B) II only C) both I and II D) neither I nor II

C

37) All of the following are additional coverages provided under Section I of the Homeowners 3 policy EXCEPT A) reasonable repairs. B) debris removal. C) personal liability. D) property removal.

C

37) An Econodeath Insurance Company actuary calculated the present value of the expected death claim the company will pay if it sells whole life insurance to a 30-year-old woman. This value is called the A) net level premium. B) gross premium. C) net single premium. D) life insurance policy reserve.

C

39) A dwelling with a replacement cost of $150,000 was insured under a Homeowners 3 policy for $100,000 at the time the roof was destroyed by a windstorm. The actual cash value of the loss was $9,000, but it will cost $12,000 to replace the roof. Assuming no deductible, what will the insurer pay to settle this loss? A) $8,000 B) $9,000 C) $10,000 D) $12,000

C

4) Reasons for the unearned premium reserve include which of the following? I. To pay losses that occur during the policy period. II. To pay premium refunds to policyholders in the event of cancellation. A) I only B) II only C) both I and II D) neither I nor II

C

42) During the financial crisis, the U.S. federal government stepped-in to prevent the financial failure of the world's largest insurer, the American International Group (AIG). AIG's near insolvency was caused by A) catastrophic hurricane and earthquake losses that were not reinsured. B) fraudulent accounting practices that had inflated earnings for many years. C) losses on derivative loan guarantees issued by the company. D) over-investment in U.S. equity markets and the sharp drop in U.S. equity values.

C

42) Mike and Michelle built their ʺdream home.ʺ They agreed to insure the home for its full value. Their insurer offered a special homeowners policy. Under the policy, if a total loss occurs, the insurer agrees to replace the home exactly as it was before the loss even if the replacement cost exceeds the amount of insurance stated in the policy. What is this type of homeowners policy called? A) waiver of inventory policy B) fair market value policy C) guaranteed replacement cost policy D) actual cash value policy

C

42) Which of the following statements about policies sold to preferred risks is (are) true? I. Preferred risks are people whose mortality experience (deaths per thousand at a given age) is expected to be more favorable than average. II. Insurers require preferred risks to purchase at least a minimum amount of life insurance, such as $250,000. A) I only B) II only C) both I and II D) neither I nor II

C

43) In schedule rating, each building is individually rated on several factors. One factor refers to the quality of the city's water supply and fire department, and the risk control devices installed in the building. This factor is called A) exposure. B) occupancy. C) protection. D) housekeeping.

C

43) To correct abuses in the financial services industry, Congress passed an Act in 2010 that included numerous provisions to reform the financial services industry. This Act was the A) Financial Modernization Act. B) McCarran-Ferguson Act. C) Dodd-Frank Act. D) Biggert-Waters Act.

C

6) Which of the following persons is (are) eligible for benefits under medical payments to others (Coverage F) of the homeowners policy? I. A friend who slips and falls on a wet patio while visiting the named insured. II. A neighborhood child who is severely scratched by a cat in the care of the named insured. A) I only B) II only C) both I and II D) neither I nor II

C

7) An insurance company chartered in another country has been licensed to operate in your state. In your state, the insurer would be considered a(n) A) nonadmitted insurer. B) foreign insurer. C) alien insurer. D) reciprocal insurer.

C

7) One of the additional coverages under the building and personal property coverage form is "increased cost of construction." The coverage is payable if A) a skilled artisan is needed to duplicate the construction of the damaged property B) a labor strike or materials shortage increases construction costs C) an ordinance or building code increases the cost of construction D) the materials necessary to rebuild the damaged structure are more expensive than ordinary building materials

C

7) The purpose of an estate clearance fund is to pay all of the following EXCEPT A) burial expenses. B) estate administration expenses. C) education costs. D) installment debts.

C

8) Persons insureds under Section I of the Homeowners 3 policy include which of the following? I. A spouse of the named insured if living in the same household II. The named insuredʹs children who are under age 24 and who are full-time college students A) I only B) II only C) both I and II D) neither I nor II

C

14) All of the following statements about group basic medical expense insurance are true EXCEPT A) Benefits under surgical expense insurance may be based on the reasonable and customary charge for the medical procedure. B) Benefits are usually provided for diagnostic X-ray and laboratory benefits. C) Benefits are designed to cover the cost of catastrophic medical expenses. D) Benefits are provided for nonsurgical care provided by a physician.

C) Benefits are designed to cover the cost of catastrophic medical expenses.

23) All of the following statements about HMOs are true EXCEPT A) They organize and deliver health care services. B) HMOs place a heavy emphasis on controlling the cost of covered services. C) Comprehensive coverage is provided even when a subscriber is outside the area served by the HMO. D) The selection of physicians is usually limited to physicians affiliated with the HMO.

C) Comprehensive coverage is provided even when a subscriber is outside the area served by the HMO.

40) John and Jane Smith were just divorced. Jane was awarded custody of the coupleʹs child, Maggie. John was born on March 18, 1979. Jane was born on January 24, 1980. John and Jane are each covered under generous employee benefit plans at work. The plans also cover the spouse and children of the worker. Assuming there is no court decree specifying responsibility for Maggieʹs health care expenses, which of the following is true regarding coverage of Maggieʹs health care expenses? A) Coverage under Johnʹs plan is primary, and coverage under Janeʹs plan is excess. B) Each plan will pay one-half of the expenses. C) Coverage under Janeʹs plan is primary, and coverage under Johnʹs plan is excess. D) Each plan will pay its pro rata share of any claims.

C) Coverage under Janeʹs plan is primary, and coverage under Johnʹs plan is excess.

*Which of the following statements about the medical payments coverage of the PAP is true?*

C) Covered expenses include the cost of funeral services.

24) Helen and John both own automobiles on which they carry liability insurance. If Helen is negligent and has an accident while driving John's car with his permission, how will each insurer respond to any liability judgment against Helen? A) The insurers will pay the judgment on a pro rata basis. B) John's insurer will pay on an excess basis if Helen's insurance is insufficient to cover the judgment. C) Helen's insurance will pay on an excess basis if John's insurance is insufficient to cover the judgment. D) The policies will pay the judgment on the basis of contribution by equal shares.

C) Helen's insurance will pay on an excess basis if John's insurance is insufficient to cover the judgment.

25) Functions of an insurance company's legal department include which of the following? I. Lobbying for legislation favorable to the insurance industry. II. Drafting policy provisions. A) I only B) II only C) both I and II D) neither I nor II

C) both I and II

*The insurance company's options for settling a collision loss to a covered auto under the PAP include which of the following?* I. Pay the loss in money. II. Repair or replace the damaged auto

C) both I and II

*Which of the following persons is (are) covered for liability insurance under the PAP?* I. a family member who drives a covered auto II. a family member who occasionally drives a friend's auto

C) both I and II

*Which of the following statements about the liability limits of the PAP is (are) true?* I. The policy is written with split limits of liability. II. Prejudgment interest is considered part of the damage award and is subject to the policy limit of liability.

C) both I and II

1) Which of the following statements about torts is (are) true? I. The person who is injured or harmed by a tort is called a plaintiff or claimant. II. The punishment for committing a tort is damages in the form of money. A) I only B) II only C) both I and II D) neither I nor II

C) both I and II

10) Which of the following statements about the causes-of-loss special form of the ISO commercial package policy is (are) true? I. The causes-of-loss special form insures against direct physical loss. II. The causes-of-loss special form excludes loss caused by earthquake. A) I only B) II only C) both I and II D) neither I nor II

C) both I and II

12) Which of the following statements about personal producing general agents is (are) true? I. They often have the option of recruiting and training sub-agents. II. They are independent agents who produce substantial amounts of life insurance with one insurer. A) I only B) II only C) both I and II D) neither I nor II

C) both I and II

17) Which of the following statements is (are) true concerning investments of property and casualty insurers and life insurers? I. Property and casualty insurance companies place greater emphasis on liquidity than do life insurers. II. Life insurance company investments are, on average, of longer duration than property and casualty insurance company investments. A) I only B) II only C) both I and II D) neither I nor II

C) both I and II

18) Which of the following are legal obligations of a property owner with respect to an invitee? I. The property owner must warn an invitee of any unsafe conditions. II. The property owner must inspect the premises and eliminate any dangerous conditions revealed by the inspection. A) I only B) II only C) both I and II D) neither I nor II

C) both I and II

22) Situations under which parents can be held liable for the actions of a child include which of the following? I. The child uses a parent's gun to injure someone. II. The child is acting as an agent of the parent. A) I only B) II only C) both I and II D) neither I nor II

C) both I and II

28) Which of the following statements about schedule rating is (are) true? I. It involves the determination of a basis rate for each exposure, which is then modified by credits or debits. II. It is based on the assumption that certain physical characteristics of the insured's operations will influence the insured's future loss experience. A) I only B) II only C) both I and II D) neither I nor II

C) both I and II

31) Alternative techniques for resolving legal disputes without litigation include which of the following? I. Arbitration II. Mediation A) I only B) II only C) both I and II D) neither I nor II

C) both I and II

32) Which of the following statements about Blue Cross and Blue Shield plans is (are) true? I. Blue Cross and Blue Shield plans can be organized on a nonprofit basis or on a for-profit basis. II. Blue Cross provides coverage for hospital services; Blue Shield provides coverage for physicians' and surgeons' fees. A) I only B) II only C) both I and II D) neither I nor II

C) both I and II

38) Which of the following statements is (are) true concerning high deductible health plans? I. An employee can withdraw money tax-free from a health savings account or health reimbursement account to pay covered medical costs. II. There is a cap on an employeeʹs out-of-pocket expenses under the plan. A) I only B) II only C) both I and II D) neither I nor II

C) both I and II

4) Reasons for having a minimum participation requirement before a group is eligible for insurance include which of the following? I. To lower the expense rate per unit of insurance. II. To minimize the possibility of insuring a group which consists largely of unhealthy individuals. A) I only B) II only C) both I and II D) neither I nor II

C) both I and II

4) Reasons for the unearned premium reserve include which of the following? I. To pay losses that occur during the policy period. II. To pay premium refunds to policyholders in the event of cancellation. A) I only B) II only C) both I and II D) neither I nor II

C) both I and II

4) Which of the following statements about underwriting standards is (are) true? I. One purpose of underwriting standards is to reduce adverse selection against the insurer. II. Equitable rates should be charged so that each group of policyowners pays its own way in terms of losses and expenses. A) I only B) II only C) both I and II D) neither I nor II

C) both I and II

40) Which of the following statements is (are) true about life insurance company investments? I. The majority of life insurance company general account assets are invested in bonds. II. The majority of life insurance company separate account assets are invested in stocks. A) I only B) II only C) both I and II D) neither I nor II

C) both I and II

44) Which of the following statements is true regarding the information systems functional area of an insurance company? I. Computers and information systems are able to perform some tasks that previously were performed directly by employees. II. Information systems can speed the processing of policies by insurers. A) I only B) II only C) both I and II D) neither I nor II

C) both I and II

9) Factors that may result in more restrictive underwriting decisions include which of the following? I. Inadequate rates. II. The unavailability of reinsurance at favorable terms. A) I only B) II only C) both I and II D) neither I nor II

C) both I and II

After all of the provisions of the Affordable Care Act are implemented, which of the following statements will be true? I. Health insurers cannot use pre-existing conditions exclusions. II. Health insurers cannot impose annual benefit limits and lifetime benefit limits. A) I only B) II only C) both I and II D) neither I nor II

C) both I and II

Problems with the current health care system in the United States include which of the following? I. Rising health care expenditures II. Considerable waste and inefficiency in the health care system A) I only B) II only C) both I and II D) neither I nor II

C) both I and II

Which of the following statements about high deductible health insurance plans is (are) true? I. Coverage under a high deductible health plan is necessary to establish a qualified health savings account (HSA). II. High deductible health plans provide a maximum limit on annual out-of-pocket expenses. A) I only B) II only C) both I and II D) neither I nor II

C) both I and II

Which of the following statements is (are) true concerning benefit payments under long-term care insurance? I. Reimbursement policies pay for actual charges up to a specified daily limit. II. Per diem policies pay a specified daily benefit regardless of the charges incurred. A) I only B) II only C) both I and II D) neither I nor II

C) both I and II

Which of the following statements is (are) true regarding the calendar-year deductible used in most individual medical expense policies? I. Once the deductible is satisfied, no additional deductible is payable during the calendar year. II. A carryover provision helps to avoid paying two deductibles in a short period of time. A) I only B) II only C) both I and II D) neither I nor II

C) both I and II

urposes of the coinsurance provision in major medical insurance policies include which of the following? I. to reduce premiums II. to prevent overutilization of policy benefits A) I only B) II only C) both I and II D) neither I nor I

C) both I and II

48) HMOs typically pay network physicians or medical groups a fixed annual or monthly payment for each member, regardless of the frequency or type of service provided. This payment is called a(n) A) pro-rata charge. B) persistency bonus. C) capitation fee. D) corridor payment.

C) capitation fee.

16) A property and casualty insurer in which the salesperson is an employee of the insurer, not an independent contractor, is called a A) fraternal insurance company. B) risk retention group. C) direct writer. D) captive insurance company

C) direct writer.

25) Shareholders and employees harmed by the negligent acts and deceptive statements of company leaders may file lawsuits against the company leaders. Company leaders are covered for such claims under which type of insurance? A) employment practices liability insurance B) employee benefit liability insurance C) directors and officers liability insurance D) general liability insurance

C) directors and officers liability insurance

14) A provision in a disability income insurance policy that requires a person to be disabled for 60 days before receiving benefits is an example of a(n) A) calendar year deductible. B) grace period. C) elimination period. D) probationary period.

C) elimination period.

21) The doctrine of respondeat superior applies to a(n) A) parent's liability for a negligent child. B) pet owner's liability for the pet. C) employer's liability for a negligent employee. D) manufacturer's liability for a faulty product.

C) employer's liability for a negligent employee.

34) Frank asked his company's employee benefits director if his group health coverage could be converted to individual coverage. The benefits director said, "Yes, you can convert to an individual policy, and the coverage is identical to your group coverage." Frank quit his job and converted to an individual policy. Six months later he filed a claim. He was dismayed to learn the conversion policy was limited compared to the group coverage, and his claim was denied. What legal doctrine will allow Frank to bring a successful legal action against his former employer because he was financially harmed due to his reasonable reliance upon a representation of fact? A) adhesion B) waiver C) estoppel D) subrogation

C) estoppel

42) Big Life Insurance Company purchased Regional Bank. Regional Bank has 27 branches in four states. Big Life Insurance Company required one employee at each bank branch to become a licensed life insurance agent and to sell Big Life Insurance Company annuities and life insurance products. This distribution channel is an example of the A) career agent distribution system. B) direct response distribution system. C) financial institution distribution system. D) worksite marketing distribution system.

C) financial institution distribution system.

22) One inland marine coverage form is the accounts receivable coverage form. In which of the following cases would the purchaser of this form (the insured) be indemnified through the coverage? A) if the insured cannot pay amounts owed because the purchaser has become insolvent B) if the insured cannot pay amounts owed because the purchaser has suffered an insured physical damage loss C) if the insured cannot collect accounts receivable because of the destruction of records D) if the insured cannot collect accounts receivable because the purchaser extended credit to poor credit risks

C) if the insured cannot collect accounts receivable because of the destruction of records

29) Beverly lives in a sparsely populated area in northern Idaho. Some insurance companies marketing coverage in northern Idaho cannot afford to have full-time adjustors there. Several insurers hire Beverly to adjust claims for their insureds. Beverly charges the insurers a fee for each claim that she settles. Beverly is a(n) A) public adjustor. B) adjustment bureau. C) independent adjustor. D) company adjustor.

C) independent adjustor.

Beginning in 2014, the Affordable Care Act requires that most U.S. citizens and legal residents have qualifying health insurance or pay a financial penalty. This provision of the Affordable Care Act is known as the A) affordable health insurance exchange option. B) public option. C) individual mandate. D) premium subsidy option.

C) individual mandate.

39) Tracy had continuous group health insurance coverage at her previous employer for 6 years. Tracy decided to change jobs. Under federal law, if Tracy changes jobs, the new employer or group health plan must give her credit for previous and continuous health insurance coverage. This characteristic is called A) renewability. B) vesting. C) portability. D) convertibility.

C) portability.

41) The business income and extra expense coverage form covers loss of business income and continuing normal operating expenses. How is business income (revenue) defined? A) total sales B) total sales less cost of goods sold C) pre-tax net profit D) after-tax net income

C) pre-tax net profit

Beth's disability income insurance policy provides benefits for accidental death, dismemberment, and loss of sight. The maximum amount payable under this benefit is known as the A) face value. B) cash value. C) principal sum. D) monthly benefit.

C) principal sum.

43) In schedule rating, each building is individually rated on several factors. One factor refers to the quality of the city's water supply and fire department, and the risk control devices installed in the building. This factor is called A) exposure. B) occupancy. C) protection. D) housekeeping.

C) protection.

48) One requirement for proving that an act was negligent is the existence of an unbroken chain of events between the act and the injury or harm that occurred. This unbroken chain of events is called A) condition precedent. B) cause and effect. C) proximate cause. D) condition subsequent.

C) proximate cause.

12) The deductible used for automobile collision losses is an example of a(n) A) calendar year deductible. B) elimination period. C) straight deductible. D) aggregate deductible.

C) straight deductible.

31) Liability Insurance Company (LIC) was approached by a regional airline to see if LIC would write the airline's liability coverage. LIC agreed to write the coverage and entered into an agreement with a reinsurer. Under the agreement, LIC retains 25 percent of the premium and pays 25 percent of the losses, and the reinsurer receives 75 percent of the premium and pays 75 percent of the losses. This reinsurance arrangement is best described as A) excess-of-loss reinsurance. B) surplus-share reinsurance. C) quota-share reinsurance. D) pool reinsurance.

C) quota-share reinsurance.

14) The building and personal property coverage form provides several optional coverages. Under one optional coverage, no deduction is taken for depreciation when settling a covered loss. This optional coverage is called A) extra expense. B) agreed value. C) replacement cost. D) inflation guard.

C) replacement cost.

Prior to passage of the Affordable Care Act, insurers could go back to the date a health insurance policy became effective and render the policy void due to a clerical error. This practice, which is prohibited under the Affordable Care Act except in cases of fraud or intentional misrepresentation of a material fact, is called A) estoppel. B) retention. C) rescission. D) reformation.

C) rescission.

32) Small Town used to be just that—a small town 6 miles from Large City. Over the years, the area between Small Town and Large City has developed, and now Small Town is part of the suburbs surrounding Large City. An auto insurer that operated in the area had a large increase in auto claims from Small Town insureds. The insurer did not adjust its rates, and this year will lose money because of claims attributable to higher population density. Which business rating objective did this insurer fail to meet? A) simplicity B) stability C) responsiveness D) encouragement of loss control

C) responsiveness

39) The process of transferring risk to the capital markets through the use of financial instruments such as bonds, futures contracts, and options is known as A) consolidation of risk. B) avoidance of risk. C) securitization of risk. D) compartmentalization of risk.

C) securitization of risk.

16) To protect policyholders, state laws place limitations on a life insurance company's investments. The assets backing interest-sensitive products, such as variable life insurance and variable annuities, are not subject to these restrictions. Assets backing interest-sensitive products are placed in a special account called the life insurer's A) policy reserve account. B) policy loan account. C) separate account. D) policyholders surplus.

C) separate account.

29) Under the terms of Jenny's auto insurance policy, she must pay the first $500 of any physical damage loss to her vehicle before her insurer will pay anything. What type of deductible is included in Jenny's auto insurance policy? A) calendar-year deductible B) waiting period C) straight deductible D) aggregate deductible

C) straight deductible

28) Antonio is a claims adjustor for LMN Insurance Company. After the insurer is notified that there has been a loss, Antonio meets with the insured. The first step in the claims process that Antonio should follow is to A) determine the amount of the loss. B) attempt to deny the claim regardless of whether he believes the claim is covered. C) verify that a covered loss has occurred. D) delay paying the claim if the claim is covered.

C) verify that a covered loss has occurred.

13) For which of the following situations would there be personal liability coverage under the homeowners policy? I. The named insured accidentally injures her husband with a chain saw. II. The named insured breaks his wife's valuable vase while practicing his golf swing. A) I only B) II only C) both I and II D) neither I nor II

D

16) By misrepresenting the true facts, Gretchen was able to convince a client to drop a life insurance policy with another company and to purchase a policy from the company that Gretchen represents. Gretchen has engaged in an illegal sales practice called A) bait and switch. B) rebating. C) retaliating. D) twisting.

D

18) Richard is using the capital retention approach to determine how much life insurance to purchase. Richard would like to provide $35,000 per year to his family, forever, if he dies. The assets that he has today will provide $25,000 in annual income without the liquidation of these assets. If life insurance proceeds can be invested to earn a 5 percent annual return, how much life insurance should Richard purchase to fund the additional income needed to meet the $35,000 annual income goal? A) $10,000 B) $100,000 C) $150,000 D) $200,000

D

18) Which of the following statements about the factors affecting automobile insurance rates is true? A) The cost of collision coverage increases as a car gets older. B) Because of high speeds in rural areas, rural drivers tend to pay more for auto insurance than city drivers pay for auto insurance. C) Young unmarried male drivers tend to have fewer accidents than young married male drivers in the same age category. D) People who drive a car to and from work tend to be charged higher rates than those who use a car for pleasure purposes.

D

19) Which of the following is an advantage of state regulation of insurance over federal regulation of insurance? A) uniformity of laws B) greater efficiency C) more effective in negotiating international agreements pertaining to insurance D) quicker response to local insurance problems

D

2) Persons insured for personal liability under the homeowners policy include which of the following? I. Relatives while visiting the named insured. II. Overnight guests who are not relatives of the named insured. A) I only B) II only C) both I and II D) neither I nor II

D

4) The loss settlement under which of the following supports the principle of indemnity? A) life insurance B) valued policies C) replacement cost property insurance D) actual cash value property insurance

D) actual cash value property insurance

20) All of the following statements about shopping for auto insurance are true EXCEPT A) Adequate liability insurance is the most important consideration. B) Consideration should be given to dropping physical damage insurance on an older car with a low market value. C) To obtain the lowest premium, an applicant should review his or her eligibility for all discounts offered. D) Price comparisons are of little value since auto insurers tend to charge the same premiums.

D

29) Lu would like to save money on her homeowners premium. All of the following steps will help her to reduce her homeowners premium EXCEPT A) raising the deductible in the homeowners policy to a higher amount. B) shopping around a homeowners policy. C) installing burglar alarms, smoke detectors, and dead-bolt locks. D) insuring the value of the land beneath the house.

D

3) All of the following are covered under the building and personal property coverage form EXCEPT A) the insured's stock and inventory when located inside the insured building B) equipment used to service the insured building C) the insured's use interest in improvements and betterments if the insured is a tenant D) property owned by key customers and key suppliers of the insured

D

31) ABC Insurance Company would like to purchase a bank. For many years, ABC was not permitted under federal law to enter into banking operations. Which of the following legislative acts eliminated the prohibition that prevented banks, insurers, and investment firms from entering into one another's markets? A) The McCarran-Ferguson Act B) The Tax Reform Act C) The Consolidated Omnibus Budget Reconciliation Act D) The Financial Modernization Act (Gramm-Leach-Bliley Act)

D

31) All of the following statements about conditions under a homeowners policy are true EXCEPT A) If an insurer broadens coverage without an additional premium during the policy period, the broadened coverage applies immediately to the present policy. B) A waiver or change in any policy provision must be approved in writing by the insurer to be valid. C) The insurer must give written consent for an assignment of the policy to be valid. D) If the named insured dies, coverage automatically ceases with respect to any property of the deceased.

D

31) All of the following statements about the business owners policy are true EXCEPT A) Coverage for business personal property includes coverage for leased personal property which the named insured has a contractual responsibility to insure. B) The amount of insurance on the buildings increase by a stated percentage each year. C) Additional coverages include pollutant cleanup and removal D) The policy must be written with a deductible of at least $2,000 for property losses

D

34) A few states have enacted laws to make minimum amounts of liability insurance available at reduced rates to individuals who cannot afford regular insurance or who have limited financial assets to protect. The coverage made available through such a plan is called A) probationary insurance. B) uninsured motorists insurance. C) no-fault auto insurance. D) low-cost auto insurance.

D

34) All of the following statements about variable life insurance are true EXCEPT A) The premium is level and guaranteed not to increase. B) The death benefit varies according to investment experience. C) The policyowner has the option of investing the cash value in several investment accounts. D) Cash surrender values are guaranteed.

D

34) Rick is risk manager of Herald News, a daily newspaper in a competitive market. Rick wants to make sure that if Herald's printing facility is damaged or destroyed , the paper will continue to be published. What type of insurance can Rick purchase to cover the added cost of continuing to print the paper after a physical damage loss has occurred? A) contingent business income coverage B) product liability insurance C) business income from dependent properties coverage D) extra expense coverage

D

34) XYZ Insurance Company expects $500,000 in claims and loss adjustment expenses for each 1,000 properties that it insures in a certain category of business insurance. What pure premium should XYZ charge for each property insured? A) $69.99 B) $166.67 C) $350.00 D) $500.00

D

39) Which of the following statements is true regarding the Homeowners 3 policy and identity theft? A) Identity theft is covered under Coverage D -- Loss of Use. B) Identity theft is covered under Coverage C -- Personal Property. C) Identity theft is covered under Coverage E -- Personal Liability. D) Identity theft is covered by adding an endorsement.

D

40) In schedule rating, each building is individually evaluated based on several rating factors. One factor refers to the possibility that the building will be damaged or destroyed by a fire that starts at an adjacent property and spreads to the building. This rating factor is known as A) occupancy. B) protection. C) maintenance. D) exposure.

D

44) Which of the following statements about savings bank life insurance is true? A) The maximum amount that a depositor can purchase is $50,000. B) The maximum amount of insurance that a depositor can purchase is limited to the amount of money on deposit in his or her savings account with the savings bank. C) The objective of savings bank life insurance is to provide protection to the bank in case a borrower dies before a loan is repaid. D) The objective of savings bank life insurance is to provide low-cost insurance to consumers by holding down expenses.

D

46) Following catastrophic hurricane losses, South Coast Insurance Company changed its deductible provision. Rather than using a specified dollar value, $250 for example; the dollar value of the deductible increases with the size of the loss. The type of deductible that South Coast changed to is called a(n) A) aggregate deductible. B) franchise deductible. C) straight deductible. D) percentage deductible.

D

48) The major argument in favor of an optional federal charter for insurers is that A) small insurers need a national charter to be competitive with large insurer. B) a federal charter will prevent insurer insolvencies. C) a federal charter will provide greater oversight of insurer market practices. D) national insurers are at a competitive disadvantage under the present system.

D

5) All of the following are defects which limit the usefulness of the human life value approach in determining the correct amount of life insurance to purchase EXCEPT A) The effects of inflation are ignored. B) Other sources of income for survivors are ignored. C) Earnings are assumed to remain constant. D) Earnings during the individual's productive lifetime are ignored.

D

5) Which of the following statements about the Homeowners 8 policy is (are) true? I. Losses to the dwelling are settled on a replacement cost basis if the dwelling is insured for its full market value. II. It is designed primarily for expensive, newer, dwellings. A) I only B) II only C) both I and II D) neither I nor II

D

9) All of the following causes of loss are included in the causes-of-loss basic form of the ISO commercial package policy EXCEPT A) fire B) lightning C) explosion D) flood

D

9) Section I of the Homeowners 3 policy provides coverage for all of the following EXCEPT A) loss of use. B) personal property. C) other structures. D) medical payments.

D

12) JKL Insurance Company reported the following information on its accounting statements last year: Premiums Written $90,000,000 Loss Adjustment Expenses $5,000,000 Underwriting Expenses 30,000,000 Premiums Earned $100,000,000 Incurred Losses $70,000,000 What was JKL's combined ratio last year? A) 100.0 B) 103.3 C) 105.0 D) 108.3

D) 108.3

31) What is the practical effect of an insurance contract being a contract of adhesion? A) The insurer can refuse to pay claims unless the insured has complied with all policy provisions. B) The insured can assign the policy only with the insurer's consent. C) The insurer can sue the insured for failure to pay any premiums. D) The policy is interpreted in the insured's favor if the policy contains any ambiguities or uncertainties.

D) The policy is interpreted in the insured's favor if the policy contains any ambiguities or uncertainties.

31) All of the following statements about the businessowners policy are true EXCEPT A) Coverage for business personal property includes coverage for leased personal property which the named insured has a contractual responsibility to insure. B) The amount of insurance on the buildings increases by a stated percentage each year. C) Additional coverages include pollutant cleanup and removal. D) The policy must be written with a deductible of at least $2,000 for property losses.

D) The policy must be written with a deductible of at least $2,000 for property losses.

46) When a fraternal insurer began operations, it asked each member, regardless of age, to pay $20 per month to the fraternal's group life insurance plan. In exchange, each member received the same amount of life insurance. Soon younger members of the group began to drop out when they realized their premiums were subsidizing a group with a higher chance of loss. Which important underwriting principle was violated in this case? A) An underwriting profit should be attained. B) Moral hazard should be avoided. C) Insureds should be selected according to underwriting standards. D) There should be equity among policyholders.

D) There should be equity among policyholders.

9) All of the following statements about endorsements and riders are true EXCEPT A) They are usually written. B) They can be used to add or delete policy provisions. C) They normally take precedence over other conflicting policy provisions. D) They are primarily used to circumvent legislation requiring specific policy provisions.

D) They are primarily used to circumvent legislation requiring specific policy provisions.

3) Why are some mutual insurers referred to as "assessment mutuals"? A) They charge low premiums because the loss exposures of their insureds are thoroughly assessed before a policy is written. B) They are noted for being very thorough in their assessment of investment opportunities. C) They are assessed for state premium taxes only if they make a profit. D) They can assess policyholders if premiums are insufficient to pay losses and expenses.

D) They can assess policyholders if premiums are insufficient to pay losses and expenses.

2) Which of the following statements about mutual insurers is true? A) They are legally organized as partnerships. B) They have a board of directors which is selected by state insurance departments. C) They are owned by their stockholders. D) They may pay dividends to their policyholders.

D) They may pay dividends to their policyholders.

32) All of the following statements about cost controls in dental insurance plans are true EXCEPT A) The coinsurance percentage used may vary by type of dental service. B) Cosmetic dental work is usually excluded. C) The limit on benefits may be expressed as an annual limit or as a lifetime limit for certain types of dental services. D) To eliminate small claims, there is no coverage for routine oral examinations, X-rays, or cleaning teeth.

D) To eliminate small claims, there is no coverage for routine oral examinations, X-rays, or cleaning teeth.

Which of the following statements about provisions in individual health insurance policies is true? A) Insurers are not permitted to place time limits on filing claims or providing proof of loss. B) After a policy is in force for 3 months, the time limit on certain defenses provision prohibits the insurance company from denying a claim based on a fraudulent misstatement in the application. C) The usual length of the grace period is 180 days. D) Under the reinstatement provision, a health insurance policy that has lapsed can be put back in force.

D) Under the reinstatement provision, a health insurance policy that has lapsed can be put back in force.

20) Which of the following statements about treaty reinsurance is true? A) Under a surplus-share treaty, 100 percent of the ceding insurer's liability must be transferred to the reinsurer. B) Using a quota-share treaty increases the ceding insurer's unearned premium reserve. C) Under an excess-of-loss treaty, the reinsurer pays losses in full only if they are less than the ceding insurer's retention limit. D) Using a reinsurance pool provides financial capacity to write large amounts of insurance.

D) Using a reinsurance pool provides financial capacity to write large amounts of insurance.

25) A contract in which the values exchanged are not equal because chance is involved is called a(n) A) contract of adhesion. B) unilateral contract. C) conditional contract. D) aleatory contract.

D) aleatory contract.

6) A total loss under a valued policy is settled on the basis of the A) market value of the loss. B) actual cash value of the loss. C) replacement value of the loss. D) amount of insurance covering the loss.

D) amount of insurance covering the loss.

*Dennis was involved in an accident. He believes the damage to his auto is $6,000. His insurer believes the damage is only $3,500. Which PAP provision is designed to handle disputes between the insurer and the insured over the amount of loss?*

D) appraisal provision

27) Why does the insured get the benefit of the doubt if an insurance policy contains any ambiguities or uncertainties? A) because insurance contracts are aleatory B) because insurance contracts are unilateral C) because insurance contracts are conditional D) because insurance contracts are contracts of adhesion

D) because insurance contracts are contracts of adhesion

16) XYZ, Inc. would suffer serious financial consequences if either of its two major customers were shut down and could not purchase XYZ products. Which of the following types of consequential loss protection would provide protection against this exposure? A) leasehold interest B) extra expense C) marine insurance D) business income from dependent properties

D) business income from dependent properties

8) A loss reserve established for each individual claim when it is reported to a property and casualty insurance company is call a(n) A) admitted asset. B) incurred-but-not-reported (IBNR) reserve. C) unearned premium reserve. D) case reserve.

D) case reserve.

40) The section of the insurance policy that includes provisions that qualify or limit the insurer's promise to perform is the A) definitions. B) insuring agreement. C) exclusions. D) conditions.

D) conditions.

39) In 2008, Liberty Mutual Insurance Company acquired Safeco Insurance Company. This acquisition demonstrates which financial services industry trend? A) convergence B) demutualization C) mass merchandising D) consolidation

D) consolidation

41) Ted's insurance claim was denied by XYZ Insurance Company. When Ted inquired why the claim was denied, he was told to, "Read the exclusion on page 5 of the policy." Ted read the exclusion. In his opinion, the exclusion was poorly worded and vague. If a court of law agrees with Ted's assessment of the exclusion, Ted may still be able to have his claim paid by the insurer because insurance contracts are A) personal contracts. B) unilateral contracts. C) aleatory contracts. D) contracts of adhesion.

D) contracts of adhesion.

16) A deductible that must be satisfied after the benefits of a basic medical expense plan are exhausted and before supplemental major medical benefits are payable is called a(n) A) calendar-year deductible. B) prospective deductible. C) straight deductible. D) corridor deductible.

D) corridor deductible.

The effect of an annual out-of-pocket limit in an individual medical expense policy is to A) limit the lifetime benefits payable under the policy. B) put a cap on annual benefits the insurer will pay. C) prevent the insured from receiving duplicate benefits if medical expenses are also covered under workers compensation insurance. D) cover 100 percent of eligible medical expenses after an insured has incurred a specified amount of out-of-pocket expenses.

D) cover 100 percent of eligible medical expenses after an insured has incurred a specified amount of out-of-pocket expenses

5) All of the following are extensions of coverage under the building and personal property coverage form EXCEPT A) newly acquired or constructed property. B) valuable papers and records. C) personal property temporarily off the premises. D) currency and securities.

D) currency and securities.

28) Big Mutual Insurance Company would like to take advantage of financial services deregulation by acquiring a bank and a stock brokerage firm. Big Mutual, however, would have trouble raising the funds needed to make these acquisitions under the mutual form of organization. Big Mutual is planning to switch from the mutual form of organization to the stock form, and to issue shares of common stock to raise capital. This change in organizational structure is called A) mutualization. B) retrocession. C) reinsurance. D) demutualization.

D) demutualization.

6) The period of time during which an employee can sign up for group insurance coverage without furnishing evidence of insurability is called a(n) A) probationary period. B) noninsurability window. C) waiting period. D) eligibility period.

D) eligibility period.

30) Shauna hurt her back and was unable to work. She filed a claim under her disability income insurance policy. Under terms of the policy, 60 days must pass between the date of the injury and when the insurer begins to replace lost earnings. This 60-day period is called a(n) A) grace period. B) enrollment period. C) probationary period. D) elimination (waiting) period.

D) elimination (waiting) period.

49) Powers specifically conferred on an agent to act on behalf of a principal are A) incidental authority. B) apparent authority. C) implied authority. D) express authority.

D) express authority.

34) Rick is risk manager of Herald News, a daily newspaper in a competitive market. Rick wants to make sure that if Herald's printing facility is damaged or destroyed, the paper will continue to be published. What type of insurance can Rick purchase to cover the added cost of continuing to print the paper after a physical damage loss has occurred? A) contingent business income coverage B) product liability insurance C) business income from dependent properties coverage D) extra expense coverage

D) extra expense coverage

27) Mark has been an underwriter for 20 years. An application he recently reviewed looked odd to him. The building value in the application seemed far too high, and Mark suspected the applicant might be planning to destroy the property after it is insured. Mark hired an outside firm to investigate the applicant and to prepare a report about the applicant. This report is called a(n) A) agent's report. B) binder. C) physical inspection. D) inspection report.

D) inspection report.

36) Francis opened a store. She knows that customers who come to the store may be injured on the premises and hold her responsible for their injuries. Under common law, business customers in the store Francis opened are considered A) aliens. B) licensees. C) trespassers. D) invitees.

D) invitees.

17) Which of the following statements about the sale of property and liability insurance through the direct response system is (are) true? I. Selling expenses are higher because market segmentation tends to be less precise than with other marketing methods. II. It is the most appropriate system for selling complex products. A) I only B) II only C) both I and II D) neither I nor II

D) neither I nor II

2) Which of the following statements about group insurance underwriting principles is (are) true? I. If a plan is contributory, 100 percent of the eligible employees must be covered. II. Employees should be allowed to determine their own level of benefits. A) I only B) II only C) both I and II D) neither I nor II

D) neither I nor II

Barb was injured in an auto accident. She was totally disabled and collected disability income benefits for 8 months. She would like to return to work on a part-time basis to see if her recovery is complete. During this period, her insurer will pay reduced disability income benefits. This type of disability is called A) recurrent disability. B) presumptive disability. C) permanent disability. D) partial disability.

D) partial disability.

The inability of the insured to perform some but not all of the important duties of his or her occupation is called A) residual disability. B) total disability. C) recurrent disability. D) partial disability.

D) partial disability.

22) A managed care plan under which members can receive medical care from non-network providers at higher out-of-pocket costs is an example of a(n) A) group practice plan. B) individual practice plan. C) exclusive provider organization. D) point-of-service plan.

D) point-of-service plan.

21) The purpose of other-insurance provisions is to A) eliminate the need for deductibles. B) penalize those insureds who carry inadequate amounts of insurance. C) specify who will pay losses if the insurer is bankrupt. D) preserve the principle of indemnity.

D) preserve the principle of indemnity.

*Ken purchased a PAP with liability limits of 100/300/50, medical payments coverage, and collision coverage. Ken fell asleep while driving late at night. He crossed the center line and hit a car approaching from the other direction. The following losses occurred: -The driver of the other car suffered $30,000 in bodily injuries. -Ken's car sustained $5,000 in damages -Ken incurred $5,000 in medical expenses -The car that Ken hit was a total loss. Which of Ken's PAP coverages will cover the damage to the car that Ken hit?*

D) property damage liability.

44) Which ocean marine coverage provides comprehensive liability insurance for property damage caused by the ship to piers and docks, damage to the ship's cargo, and injury to the passengers or crew? A) collision liability clause B) hull insurance C) freight insurance D) protection and indemnity insurance

D) protection and indemnity insurance

41) One method through which reinsurance is provided is through an organization of insurers that underwrites insurance on a joint basis. Through the organization, financial capacity is available for large commercial risks. This reinsurance arrangement is a(n) A) quota-share treaty. B) surplus-share treaty. C) excess-of-loss treaty. D) reinsurance pool.

D) reinsurance pool.

45) Doris started a business 2 years ago. The business has been successful, and Doris is thinking about starting to offer some employee benefits for her workers. She plans to offer a group term life insurance benefit. All of the following are usual eligibility requirements for participation in a group life insurance plan EXCEPT A) full-time employment. B) be actively at work when insurance becomes effective. C) apply for insurance during the eligibility period. D) satisfy a 2-year probationary period.

D) satisfy a 2-year probationary period.

One long-term care insurance benefit trigger considers whether the insured needs supervision to protect against threats to health or safety due to memory loss or disorientation. This benefit trigger is referred to as a(n) A) activities of daily living trigger. B) medical necessity trigger. C) needs test trigger. D) severe cognitive impairment trigger.

D) severe cognitive impairment trigger.

43) Catastrophe bonds are made available to institutional investors in the capital markets through an entity that is specially created for that purpose. This is entity is called a A) risk retention group. B) fraternal insurance company. C) captive insurance company. D) special purpose reinsurance vehicle.

D) special purpose reinsurance vehicle.

3) A situation in which a person is held legally liable even though fault or negligence cannot be proven is an example of A) general damages. B) comparative negligence. C) an intentional tort. D) strict liability.

D) strict liability.

13) Sue's office building was damaged by a fire caused by a careless tenant. After paying Sue for the loss, the insurance company sued the tenant to recover its loss. This suit is based on the principle of A) warranty. B) insurable interest. C) utmost good faith. D) subrogation.

D) subrogation.

43) The bar at the local franchise of a national chain continued to serve a drunk customer. The customer left the bar and tried to drive home. The drunk driver hit and killed three people who were riding bicycles. The legal representatives of those who were killed filed a multi-million dollar lawsuit against the national chain. As jury awards in the city where the incident occurred tend to be high, insurers marketing liquor liability insurance refused to issue new coverage until the case was resolved. Because no admitted insurers were willing to offer the coverage, liquor liability in this case is considered a(n) A) residual line. B) mandatory coverage. C) orphan policy. D) surplus line.

D) surplus line.

6) Common sources of underwriting information for life and health insurance include all of the following EXCEPT A) the application. B) a physical examination. C) the Medical Information Bureau. D) the applicant's income tax return.

D) the applicant's income tax return.

40) Jan was injured in a work-related auto accident. She sued the other driver, and the case went to court. While questioning Jan, the defendant's lawyer asked if her injuries and lost earnings were covered under workers compensation. Jan's lawyer objected to the question. The judge ruled the question was improper and instructed the jury to disregard the question. Based on the judge's reaction to the question, we can conclude that which of the following rules is in force where this trial took place? A) the joint and several liability rule B) the last clear chance rule C) the comparative negligence rule D) the collateral source rule

D) the collateral source rule

21) David owns a liquor store in a high-crime area. In order to obtain a reduced insurance premium, David promised to have a burglar alarm operating at the store when the store was closed. This agreement, which was incorporated into the insurance contract, is an example of a A) representation. B) binder. C) rider. D) warranty.

D) warranty.

Dan picked up his friend Rodney to drive to their softball game. Both Dan and Rodney have a Personal Auto Policy (PAP) with $5,000 of medical payments coverage. Dan hit a parked car, and Rodney was injured, incurring $9,000 of medical expenses. How will this claim be settled under the other insurance provision of the PAP?

Dan's insurer will pay $5,000 and Rodney's insurer will pay $4,000.

Which of the following statements about the use of deductibles is (are) true? I. They represent risk retention by insurance purchasers. II. They tend to increase the cost of adjusting small claims.

I only

Which of the following statements about the medical insurance (Part B) portion of Medicare is (are) true? I. Participation in Part B of Medicare is voluntary. II. It is provided at no cost to anyone who is fully insured under Medicare.

I only

Which of the following statements about workers compensation insurance is (are) true? I. Under the workers compensation part of the policy, the insurer agrees to pay the benefits required by the workers compensation law of any state listed in the policy declarations. II. The employers liability part of the policy provides workers compensation benefits when employees are working in states other than those listed in the policy declarations.

I only

Which of the following statements concerning directors and officers liability insurance (D&O) is (are) true? I. D&O policies usually cover the personal liability of directors and officers. II. D&O policies exclude coverage for claims brought by employees and by stockholders.

I only

Which of the following statements concerning the selection of risk management techniques and insurance market conditions is true? I. It's easier to purchase affordable insurance during a soft market than during a hard market. II. Retention is used more during a soft market than during a hard market.

I only

Which of the following statements is (are) true concerning the self-insured retention (SIR) under a commercial umbrella liability policy? I. The SIR applies to claims covered by the umbrella policy but not by any underlying insurance. II. The SIR only applies to those claims for which the underlying policy has paid first, and the umbrella policy is paying on an excess basis.

I only

Which of the following statements regarding insurance and hedging is (are) true? I. Insurance involves the transfer of insurable risk while hedging handles risk that is typically uninsurable. II. Both insurance and hedging rely on the law of large numbers to reduce risk. A) I only B) II only C) both I and II D) neither I nor II

a

A pure risk is defined as a situation in which there is a. only the possibility of loss or no loss b. only the possibility of profit c. a possibility of neither profit nor loss d. a possibility of either profit or loss

a

A risk that affects only individuals or small groups and not the entire economy is called a. diversifiable risk b. pure risk c. speculative d. non-diversifiable risk

a

A risk that affects only individuals or small groups and not the entire economy is called a A) diversifiable risk. B) pure risk. C) speculative risk. D) nondiversifiable risk.

a

ABC insurance retains the first $1 million of each property damage loss and purchases insurance for that part of any property loss that exceeds $1 million. The insurance for property losses above $1 million are called a. excess insurance b. liability insurance c. coinsurance d. primary insurance

a

Abandoning an existing loss exposure is an example of a. avoidance b. retention c. non-insurance transfer d. insurance transfer

a

All of the following are characteristics of insurance EXCEPT A) risk avoidance. B) pooling of losses. C) payment of fortuitous losses. D) indemnification.

a

All of the following are characteristics of insurance EXCEPT a. risk avoidance b. pooling of losses c. payment of fortuitous losses d. indemnification

a

All of the following are classified as casualty insurance EXCEPT A) life insurance. B) general liability insurance. C) workers compensation insurance. D) burglary and theft insurance

a

All of the following are classified as casualty insurance EXCEPT a. life insurance b. general liability insurance c. workers compensation insurance d. burglary and theft insurance

a

Which of the following statements regarding insurance and gambling is true? 1. Insurance is used to handle existing pure risks, while gambling creates a new speculative risk 2. Insurance usually involves risk avoidance, while typically involves only risk reduction a. 1 only b. 2 only c. both 1 and 2 d. neither 1 or 2

a

An insurance company estimates its objective risk for 10,000 exposures to be 10 percent. Assuming the probability of loss remains the same, what would happen to the objective risk if the number of exposures were to increase to 1 million? A) It would decrease to 1 percent. B) It would decrease to 5 percent. C) It would remain the same. D) It would increase to 20 percent.

a

An insurance company estimates its objective risk for 10,000 exposures to be 10 percent. Assuming the probability of loss remains the same, what would happen to the objective risk if the number of exposures were to increase to 1 million? a. it would decrease to 1 percent b. it would decrease to 5 percent c. it would remain the same d. it would increase to 20 percent

a

An insurance policy specifically written and designed to meet the needs of an insurance purchaser is called a a. manuscript policy b. bureau policy c. standard policy d. excess policy

a

Following good health habits can be categorized as A) loss prevention. B) risk retention. C) noninsurance transfer. D) personal insurance.

a

Following good health habits can be categorized as a. loss prevention b. risk retention c. non-insurance transfer d. personal insurance

a

Gina would like to buy a house. She will pay 10 percent of the cost of the house as a down payment and borrow the other 90 percent from a mortgage lender. The home will serve as collateral for the loan. The lender requires Gina to purchase property insurance on the home so that the collateral supporting the loan will be protected. This scenario illustrates which of the following benefits of insurance to society? A) enhancement of credit B) reduction of fear and worry C) source of investment funds D) incentives for loss prevention

a

Gina would like to buy a house. She will pay 10 percent of the cost of the house as a down payment and borrow the other 90 percent from a mortgage lender. The home will serve as collateral for the loan. The lender requires Gina to purchase property insurance on the home so that the collateral supporting the loan will be protected. This scenario illustrates which of the following benefits of insurance to society? a. enhancement of credit b. reduction of fear and worry c. source of investment funds d. incentives for loss prevention

a

Inland marine insurance provides coverage for A) goods being shipped on land. B) premature death of members of the armed forces. C) goods being shipped on ocean-going vessels. D) liability exposures of nonprofit organizations.

a

Inland marine insurance provides coverage for a. goods being shipped on land b. premature death of members of armed forces c. goods being shipped on on ocean-going vessels d. liability of exposures of non-profit organizations

a

Laura Evans is risk manager of LMN Company. Laura decided to retain certain property losses. All of the following methods which Laura can use to fund retained property losses EXCEPT a. private insurance b. current net income c. funded reserve d. borrowed funds

a

Loss severity is defined as the a. probable size of the losses which may occur during some period b. probable number of losses that may occur during some period c. probability that any particular piece of property may be totally destroyed d. probability that a liability judgement may exceed a firms net worth

a

MLX Drug Company would like to market a new hypertension drug. While the Food and Drug Administration (FDA) was testing the drug, it discovered that the drug produced a harmful side effect. When MLX learned of the FDAȇs test result, MLX abandoned its plan to produce and distribute the drug. MLXȇs reaction illustrates A) risk avoidance. B) hedging. C) risk transfer. D) risk retention.

a

MLX Drug Company would like tot market a new hypertension drug. While the Food and Drug Administration (FDA) was testing the drug, it discovered that the drug produced a harmful side effect. When MLX learned of the FDA's test result, MLX abandoned its plan to produce and distribute the drug. MLX's reaction illustrates a. risk avoidance b. hedging c. risk transfer d. risk retention

a

Methods by which insurers may minimize or avoid catastrophic losses include which of the following? I. The use of reinsurance II. Concentrating coverage written in one geographic region A) I only B) II only C) both I and II D) neither I nor II

a

Risk management is concerned with a. the identification and treatment of loss exposures b. the management of speculative risks only c. the management of pure risks that are uninsurable d. the purchase of insurance only

a

Which of the following statements regarding insurance and hedging is (are) true? 1. insurance involves the transfer of insurable risk while hedging handles risk that is typically uninsurable 2. both insurance and hedging rely on the law of large numbers to reduce risk a. 1 only b. 2 only c. both 1 and 2 d. neither 1 or 2

a

The long-run relative frequency of an event based on the assumption of an infinite number of observations with no change in the underlying conditions is called A) objective probability. B) objective risk. C) subjective probability. D) subjective risk.

a

The long-run relative frequency of an event is based on the assumption of an infinite number of observations with no change in the underlying conditions is called a. objective probability b. objective risk c. subjective probability d. subjective risk

a

The premature death of an individual is an example of a A) pure risk. B) speculative risk. C) fundamental risk. D) physical hazard.

a

The premature death of an individual is an example of a a. pure risk b. speculative risk c. fundamental risk d. physical hazard

a

Tyndal Products Company produces cereal. The company has entered into contracts to deliver 500,000 boxes of cereal during the next 18 months. The company is concerned that the prices of two ingredients, corn and wheat, may increase over the next 18 months. The company used grain futures contracts to hedge the price risk associated with these commodities. Tyndalȇs use of hedging illustrates which risk management technique? A) noninsurance transfer B) risk avoidance C) risk retention D) risk assumption

a

Tyndal Products Company produces cereal. The company has entered into contracts to deliver 500,000 boxes of cereal during the next 18 months. The company is concerned that the prices of two ingredients, corn and wheat, may increase over the next 18 months. The company used grain futures contracts to hedge the price risk associated with these commodities. Tyndal's use of hedging illustrates which risk management technique? a. non-insurance transfer b. risk avoidance c. risk retention d. risk assumption

a

Which of the following about statements about an excess insurance plan is true? a. the insurer does not participate in a loss until it exceeds the amount the firm has decided to retain b. the insurer pays first up to some specified level; the insured then pays all losses exceeding the insurer's retention level c. losses in excess of a specified amount are not covered d. the insured and insurer share equally in any loss that occurs

a

Which of the following is implied by the pooling of losses? A) sharing of losses by an entire group B) inability to predict losses with any degree of accuracy C) substitution of actual loss for average loss D) increase of objective risk

a

Which of the following is implied by the pooling of losses? a. sharing of losses by an entire group b. inability to predict losses with any degree of accuracy c. substitution of actual loss for average loss d. increase of objective risk

a

Which of the following statements about liability risks is (are) true 1. Future income and assets can be attached to pay judgements if inadequate insurance is carried 2. There is an upper limit on the amount of loss a. 1 only b. 2 only c. both 1 and 2 d. neither 1 or 2

a

Which of the following statements about liability risks is (are) true? I. Future income and assets can be attached to pay judgments if inadequate insurance is carried. II. There is an upper limit on the amount of loss. A) I only B) II only C) both I and II D) neither I nor II

a

Which of the following statements about self-insurance is (are) true? 1. It is a planned retention 2. state law usually prohibits its use for workers compensation a. 1 only b. 2 only c. both 1 and 2 d. neither 1 or 2

a

Which of the following statements about the characteristics of social insurance programs is true? A) Benefits are loosely related to a worker's earnings. B) Benefits are paid primarily on the basis of individual equity. C) Participation in social insurance programs is voluntary. D) Social insurance programs must be fully funded at all times to pay required benefits.

a

Which of the following statements about the use of deductibles is (are) true? 1. They represent risk retention by insurance purchasers 2. they tend to increase the cost of adjusting small claims a. 1 only b. 2 only c. both 1 and 2 d. neither 1 or 2

a

Which of the following statements concerning the selection of risk management techniques and insurance market conditions is true? 1. It's easier to purchase affordable insurance during a "soft" market than during a "hard" market 2. Retention is used more during a "soft" market than during a "hard" market a. 1 only b. 2 only c. both 1 and 2 d. neither 1 or 2

a

Which of the following statements regarding insurance and gambling is (are) true? I. Insurance is used to handle existing pure risks, while gambling creates a new speculative risk. II. Insurance usually involves risk avoidance, while gambling typically involves only risk reduction. A) I only B) II only C) both I and II D) neither I nor II

a

ABC Health Insurance Company sells health insurance in one state. Recently, that state's legislature passed a law forbidding health insurers from considering an individual's health history when selecting applicants to insure. This change in law will increase the possibility of unprofitable results for ABC. This type of hazard is an example of a. physical hazard b. legal hazard c. moral hazard d. attitudinal hazard

b

ABC Health Insurance Company sells health insurance in one state. Recently, that stateȇs legislature passed a law forbidding health insurers from considering an individualȇs health history when selecting applicants to insure. This change in law will increase the possibility of unprofitable results for ABC. This type of hazard is an example of A) physical hazard. B) legal hazard. C) moral hazard. D) attitudinal hazard.

b

According to the law of large numbers, what happens as the number of exposure units increases? A) Actual results will increasingly differ from probable results. B) Actual results will more closely approach probable results. C) Nondiversifiable risk will decrease. D) Objective risk will increase.

b

According to the law of large numbers, what happens as the number of exposure units increases? a. actual results will increasingly differ from probable results b. actual results will more closely approach probable results c. non-diversifiable risk will decrease d. objective risk will increase

b

Acme Company has three identical manufacturing plants, one on the Texas Gulf Coast, one in southern Alabama, and one in Florida. Each plant is valued at $50 million. Acme's risk manager is concerned about the damage which could be caused by a single hurricane. The risk manager believes there is an extremely low probability that a single hurricane could destroy two or all three plants because they are located so far apart. What is the probable maximum loss associated with a single hurricane a. $0 million b. $50 million c. $100 million d. $150 million

b

All of the following are benefits to society that result from insurance EXCEPT A) less worry and fear. B) elimination of moral hazard. C) indemnification for loss. D) loss prevention

b

All of the following are benefits to society that result from insurance EXCEPT a. less worry and fear b. elimination of moral hazard c. indemnification for loss d. loss prevention

b

All of the following are burdens to society because of the presence of risk EXCEPT A) The size of an emergency fund must be increased. B) Individuals may profit from accepting a speculative risk. C) Society is deprived of certain goods and services. D) Mental fear and worry are present.

b

All of the following are burdens to society because of the presence of risk EXCEPT a. The size of an emergency fund must be increased b. Individuals may profit from accepting a speculative risk c. Society is deprived of certain goods and services d. Mental fear and worry are present

b

All of the following are commercial risks EXCEPT A) the loss of business income. B) the risk of insufficient retirement income. C) the risk of being sued. D) the risk of property damage.

b

All of the following are commercial risks EXCEPT a. the loss of business income b. the risk of insufficient retirement income c. the risk of being sued d. the risk of property damage

b

All of the following are methods of non-insurance transfer EXCEPT a. entering into a hold-harmless agreement b. avoiding dangerous activities c. hedging risk using future contacts d. incorporating a business

b

All of the following are methods of noninsurance transfer EXCEPT A) entering into a hold-harmless agreement. B) avoiding dangerous activities. C) hedging risk using futures contracts. D) incorporating a business

b

All of the following changes would reduce the long-range deficit for the Social Security program EXCEPT A) increase the OASDI taxable wage base. B) delay scheduled increases in the full retirement age. C) make all OASDI benefits fully taxable. D) use general revenues of the federal government to help pay benefits.

b

Alpha Insurance Company insures a broad range of risks, including whatever is not covered by fire, marine, and life insurers. Which term best describes the wide range of risks written by Alpha Insurance? A) fidelity insurance B) casualty insurance C) social insurance D) property insurance

b

Alpha Insurance Company insures a broad range of risks, including whatever is not covered by fire, marine, and life insurers. Which term best describes the wide range of risks written by Alpha Insurance? a. fidelity insurance b. casualty insurance c. social insurance d. property insurance

b

An automobile that is a total loss as a result of a collision is an example of which of the following types of risk? I. Speculative risk II. Diversifiable risk A) I only B) II only C) both I and II D) neither I nor II

b

An automobile that is a total loss as a result of a collision is an example of which of the following types or risk? 1. Speculative Risk 2. Diversifiable Risk a. 1 only b. 2 only c. both 1 and 2 d. neither 1 or 2

b

An earthquake is an example of a A) moral hazard. B) peril. C) physical hazard. D) objective risk.

b

An earthquake is an example of a a. moral hazard b. peril c. physical hazard d. objective risk

b

Ashley opened an all-you-can-eat buffet restaurant. The cost per-person was based upon what Ashley believed an average restaurant patron would consume. The restaurant began to lose money. Ashley concluded that her patrons hadȱȈabove averageȈȱappetites, and were attracted to her restaurant because they could eat as much as they wanted while being charged an average price. A similar phenomenon exists in insurance markets. This problem is called A) moral hazard. B) adverse selection. C) attitudinal hazard. D) fundamental risk.

b

Ashley opened an all-you-can-eat buffet. The cost per-person was based upon what Ashley believed an average restaurant patron would consume. The restaurant began to lose money. Ashley concluded that her patrons had "above average" appetites, and were attracted to her restaurant because they could eat as much as they wanted while being charged an average price. A similar phenomenon exists in insurance markets. The problem is called a. moral hazard b. adverse selection c. attitudinal hazard d. fundamental risk

b

Barb, who is self employed, is the main breadwinner for her family. Barb does not have a disability income insurance because she has never stopped to consider the impact of long-term disability upon her family. Barb's treatment of the risk of disability is best described as a. risk transfer b. passive retention c. risk avoidance d. active retention

b

Ben is concerned that if he injures someone or damages someone's property he could be held legally responsible and required to pay damages. This type of risk is called a. speculative risk b. liability risk c. non-diversifiable risk d. property risk

b

Ben is concerned that if he injures someone or damages someoneȇs property he could be held legally responsible and required to pay damages. This type of risk is called a A) speculative risk. B) liability risk. C) nondiversifiable risk. D) property risk.

b

Beth was injured at work and is eligible to receive workers compensation benefits. All of the following benefits are provided under workers compensation EXCEPT A) disability income. B) retirement benefits. C) rehabilitation. D) medical care.

b

Brad started a pest control business. To protect his personal assets against liability arising out of the business, Brad incorporated the business. Bradȇs use of the corporate form of organization to shield against personal liability claims illustrates A) fundamental risk. B) noninsurance transfer. C) risk retention. D) objective risk.

b

Brad started a pest control business. To protect personal assets against liability arising out of the business, Brad incorporated the business. Brad's use of the corporate form of organization to shield against personal liability claims illustrates a. fundamental risk b. non-insurance transfer c. risk retention d. objective risk

b

Characteristics of a fortuitous loss include which of the following 1. The loss is certain to occur 2. The loss occurs as a result of chance a. 1 only b. 2 only c. both 1 and 2 d. neither 1 or 2

b

Characteristics of a fortuitous loss include which of the following? I. The loss is certain to occur. II. The loss occurs as a result of chance. A) I only B) II only C) both I and II D) neither I nor II

b

Curt borrowed money from a bank to purchase a fishing boat. He purchased property insurance on the boat. Curt had difficulty making loan payments because he did not catch many fish, and fish prices were low. Curt intentionally sunk the boat, collected from his insurer, and paid off the loan balance. This scenario illustrates the problem of A) adverse selection. B) moral hazard. C) nondiversifiable risk. D) attitudinal hazard.

b

Curt borrowed money from a bank to purchase a fishing boat. He purchased property insurance on the boat. Curt had difficulty making loan payments because he did not catch many fish, and fish prices were low. Curt intentionally sunk the boat, collected from his insurer, and paid off the loan balance. This scenario illustrates a problem of a. adverse selection b. moral hazard c. non-diversifiable risk d. attitudinal risk

b

From the viewpoint of the insurer, all of the following are characteristics of an ideally insurable risk EXCEPT A) The loss must be accidental. B) The loss should be catastrophic. C) The premium must be economically feasible. D) There must be a large number of exposure units

b

From the viewpoint of the insurer, all of the following characteristics of an ideally insurable risk EXCEPT a. the loss must be accidental b. the loss should be catastrophic c. the premium must be economically feasible d. there must be a large number of exposure units

b

In reviewing his company's operations, a risk manager noticed that all of the company's finished goods were stored in a single warehouse. The risk manager recommended that the finished goods be divided among three warehouses to prevent all of the finished goods from being destroyed by the same peril. Dividing the finished goods among three warehouses illustrates a. risk avoidance b. risk control c. insurance d. non-insurance transfer

b

Katelyn was just named Risk Manager of ABC Company. She has decided to create a risk management program which considers all of the risks faced by ABC-pure, speculative, operational, and strategic-in a single risk management program. Such a program is called a(n) A) financial risk management program. B) enterprise risk management program. C) fundamental risk management program. D) consequential risk management program.

b

Kathleen was just named Risk Manager of ABC company. She has decided to create a risk management program which considers all of the risk faced by ABC-pure, speculative, operational, and strategic-in a single risk management program. Such program is called a. financial risk management program b. enterprise risk management program c. fundamental risk management program d. consequential risk management program

b

Loss frequency is defined as the a. probable size of the losses that may occur during some period b. probable number of losses that may occur during some period c. probability that any particular piece of property may be totally destroyed d. probability that a liability judgment may exceed a firms net worth

b

Low-frequency, low-severity loss exposure are best handled by a. avoidance b. retention c. insurance d. non-insurance transfer

b

Objective risk is defined as A) the probability of loss. B) the relative variation of actual loss from expected loss. C) uncertainty based on a personȇs mental condition or state of mind. D) the cause of loss.

b

Objective risk is defined as a. the probability of loss b. the relative variation of actual loss from expected loss c. uncertainty based on a person's mental condition or state of mind d. the cause of loss

b

One branch of government insurance programs has a number of distinguishing characteristics. These programs are compulsory, they are financed by mandatory contributions rather than general tax revenues, and benefits are weighted in favor of low-income groups. These government insurance programs are called A) welfare programs. B) social insurance programs. C) casualty insurance programs. D) private insurance programs.

b

One branch of government insurance programs has a number of distinguishing characteristics. These programs are compulsory, they are financed by mandatory contributions rather than general tax revenues, and benefits are weighted in favor of low-income groups. These government insurance programs are called a. welfare programs b. social insurance programs c. casualty insurance programs d. private insurance programs

b

Parker Department Stores has been hurt in recent months by a large increase in shoplifting losses. Parker's risk manager concluded that while frequency of shoplifting losses was high, the severity is still relatively low. What is (are) the appropriate risk management techniques to apply to this problem? a. retention b. loss control and retention c. transfer through insurance d. avoidance

b

Purchasing health insurance illustrates the use of which personal risk management technique? a. avoidance b. risk transfer c. loss control d. risk retention

b

Sources of information that can be used by a risk manager to identify pure loss exposures include all of the following EXCEPT a. risk analysis questionnaires b. currency and exchange rates c. physical inspections d. past losses

b

The use of fire-resistive materials when constructing a building is an example of A) risk transfer. B) loss control. C) risk avoidance. D) risk retention.

b

The use of fire-resistive materials when constructing a building is an example of a. risk transfer b. loss control c. risk avoidance d.risk retention

b

The worst possible loss that is likely to happen is referred to as the a. maximum possible loss b. probable maximum loss c. frequency of loss d. severity of loss

b

Uncertainty based on a person's mental condition or state of mind is known as a. objective risk b. subjective risk c. objective probability d. subjective probability

b

Uncertainty based on a personȇs mental condition or state of mind is known as A) objective risk. B) subjective risk. C) objective probability. D) subjective probability.

b

Under one type of Medicare Advantage Plan, members of the plan can see any doctor or health services provider that accepts Medicare patients. If members receive care outside the network of member physicians and care facilities, they must pay higher out-of-pocket costs. This type of Medicare Advantage Plan is a A) Medicare HMO. B) Medicare PPO. C) Medicare Special Needs Plan. D) Medicare Private Fee-for-Service Plan.

b

Under the Medicare Prescription Drug Program, a coverage gap (also called a "donut hole") exists after the beneficiary and drug plan pay a certain amount for covered drugs. The coverage gap refers to A) the large, up-front deductible that must be satisfied if the patient has a prescription for a covered brand-name drug. B) the temporary gap in coverage that begins when the beneficiary and drug plan pay a certain amount for covered drugs during the year and ends when the catastrophic limit is reached and coverage resumes. C) the temporary gap in coverage that begins after beneficiaries reach the lifetime limit on catastrophic drug expenses and ends when a new deductible is met and coverage resumes. D) the temporary gap in coverage that begins when prescription benefits terminate for beneficiaries who attain age 68 and resumes when beneficiaries attain age 72.

b

Which of the following conditions is (are) appropriate for using retention? 1. losses are difficult to predict 2. the worst possible loss is not serious a. 1 only b. 2 only c. both 1 and 2 d. neither 1 or 2

b

Which of the following expenses is covered under the medical insurance (Part B) portion of the Medicare program? A) a long-term stay in rest home B) the fee charged by a surgeon for an operation C) the cost of semis-private room during a hospital stay D) routine prescription drugs outside of the hospital

b

Which of the following is (are) true concerning private insurance? 1. social insurance programs are private insurance programs 2. both individuals and businesses purchase private insurance a. 1 only b. 2 only c. both 1 and 2 d. neither 1 or 2

b

Which of the following is a form of casualty insurance? A) fire insurance B) general liability insurance C) inland marine insurance D) ocean marine insurance

b

Which of the following is a reason that social insurance programs exist? A) to compete with private insurance programs B) to provide a base of economic security C) to provide need-tested benefits to low-income individuals D) to reduce involvement of the government in insurance markets

b

Which of the following is a result of adverse selection? a. the insurer's financial results will be substantially improved b. persons most likely to have losses are also most likely to seek insurance at standard rates c. it is unnecessary for the insurance company to use underwriting d. insurance can be written only by the federal government

b

Which of the following statements about a personal risk management program is (are) true? 1. insurance and retention are the only technique used to handle potential losses 2. the steps in a personal risk management process are the same steps used by businesses a. 1 only b. 2 only c. both 1 and 2 d. Neither 1 or 2

b

Which of the following statements about captive insurance companies is (are) true? 1. a captive insurance company established by a US company must be domiciled in the United States 2. a captive insurance company may be owned by several parents a. 1 only b. 2 only c. both 1 and 2 d. neither 1 or 2

b

Which of the following statements about chance of loss and risk is (are) true? 1. If the chance of loss is identical for two groups, the objective risk must be the same 2. Two individuals may perceive differently the risk inherent in a given activity a. 1 only b. 2 only c. both 1 and 2 d. neither 1 or 2

b

Which of the following statements about chance of loss and risk is (are) true? I. If the chance of loss is identical for two groups, the objective risk must be the same. II. Two individuals may perceive differently the risk inherent in a given activity. A) I only B) II only C) both I and II D) neither I nor II

b

Which of the following statements is (are) true concerning private insurance? I. Social insurance programs are private insurance programs. II. Both individuals and businesses purchase private insurance. A) I only B) II only C) both I and II D) neither I nor II

b

Which of the following statements regarding the Emergency Unemployment Compensation (UEC) program is true? A) EUC benefits are limited to individuals who have exhausted regular state benefits and have annual incomes below the federal poverty level. B) EUC benefits are funded entirely by the federal government. C) EUC benefits are limited to employees who have lost their job because of a labor dispute. D) EUC benefits are available on a case-by-case based and awarded at the discretion of the Secretary of Labor.

b

Which of the following statements regarding the use of retention is (are) true? 1. retention is best used for loss exposures that have a low frequency and a high severity 2. a financially strong firm can have a higher retention level than a firm whose financial position is weak a. 1 only b. 2 only c. both 1 and 2 d. neither 1 or 2

b

*All of the following are covered persons under the medical payments coverage of the PAP EXCEPT*

b) a pedestrian struck by the named insured's auto

If the value of a vehicle is increased after repairs, such as repainting an entire auto when only one fender or door is damaged, the insurer will not pay for the increase in value. Another name for the increase in value is

betterment

Which of the following is covered under the liability coverage of a businessowners policy?

bodily injury liability

In addition to marketing life insurance, life insurers typically sell which of the following products? 1. retirement annuities 2. disability income insurance a. 1 only b. 2 only c. both 1 and 2 d. neither 1 or 2

c

Which of the following statements about errors and omissions insurance is (are) true? I. It is purchased by professionals to protect against claims arising for negligent acts, errors, and omissions. II. It is designed to meet the needs of stock brokers, attorneys, engineers, architects and other professionals who give advice to clients.

both I and II

Which of the following statements about the liability coverage of the Business Auto Coverage Form is (are) true? I. The insurer agrees to defend the insured and pay all legal defense costs arising out of an accident involving a covered auto. II. The insured is covered for property damage and bodily injury arising out of the use of a covered auto.

both I and II

Which of the following statements regarding employer liability insurance under the workers compensation insurance coverage form is (are) true? I. Employer liability insurance covers an employee injury that occurs on the job but may not be considered work-related. II. If an injured employee sues a negligent third-party, and the third-party sues the employer for contributory negligence, the lawsuit is covered under employer liability insurance.

both I and II

Which statement about a company's cost of risk is (are) true? I. Cost of risk includes insurance premiums and retained losses. II. Reducing the cost of risk increases profitability.

both I and II

According to the law of large numbers, what should happen as an insurer increases the number of units insured? A) The amount the insurer expects to pay in claims should decrease. B) Underwriting expenses should decrease. C) Actual results will more closely approach expected results. D) The insurerȇs profitability should become more variable

c

All of the following are disadvantages of non-insurance transfers EXCEPT a. the party to whom the potential loss is transferred may be unable to pay b. the transfer may fail because the contract language is ambiguous c. the only potential losses that can be transferred are those that are not commercially insurable d. the non-insurance transfer may be costly

c

All of the following are disadvantages of using insurance in a risk management program EXCEPT a. there is an opportunity cost because premiums must be paid in advance b. considerable time and effort must be spent selecting and negotiating coverages c. it results in considerable fluctuations in earnings after losses occur d. attitudes toward loss control may become lax when losses are insured

c

All of the following are examples of direct property losses EXCEPT A) the theft of a personȇs jewelry. B) the destruction of a firmȇs manufacturing plant by an earthquake. C) the cost of renting a substitute vehicle while a collision-damaged car is being repaired. D) the vandalism of a personȇs automobile.

c

All of the following are examples of direct property losses except a. the loss of a person's jewerly b. the destruction of a firm's manufacturing plant by an earthquake c. the cost of renting a substitute vehicle while a collision-damaged car is being repaired d. the vandalism of a person's vehicle

c

All of the following provisions under the Affordable Care Act have an impact on the current Medicare Program EXCEPT A) free preventative care for seniors. B) additional funding to reduce fraud in the Medicare Program. C) free rest home care for beneficiaries over age 90. D) rebates for the Part D coverage gap (donut hole).

c

All of the following statements about captive insurers are true EXCEPT a. they may act as a profit center by insuring parties other than the parent company b. they provide a way to obtain types of insurance that may be unavailable from commercial insurers c. they increase the volatility of the parent company's earnings d. they make it easier for a firm to have access to reinsurance

c

All of the following statements about risk retention are true EXCEPT A) It may be used intentionally if commercial insurance is unavailable. B) It may be used passively because of ignorance. C) Its use is most appropriate for low-frequency, high-severity types of risks. D) Its use results in cost savings if losses are less than the cost of insurance.

c

All of the following statements about risk retention are true EXCEPT a. It may be used intentionally if commercial insurance is unavailable b. It may be used passively because of ignorance c. Its use is most appropriate for low-frequency, high severity types of risks d. Its use results in cost savings if losses are less than the cost of insurance

c

All of the following statements about the administration of a risk management program are true EXCEPT a. the risk manager is an important part of a firm's management team b. a risk management policy statement can be used to educate top executives about the risk management process c. if a risk management program is properly designed, periodic review of the program is unnecessary d. in order to properly identify loss exposures, the risk manager needs the cooperation of other departments

c

An individuals personal estimate of the chance of loss is a. an objective probability b. an objective risk c. a subjective probability d. an a priori probability

c

An individualȇs personal estimate of the chance of loss is A) an objective probability. B) an objective risk. C) a subjective probability. D) an a priori probability.

c

Apex Insurance Company wrote a large number of property insurance policies in an area where earthquake losses could occur. When the president of Apex was asked if she feared that a severe earthquake might put the company out of business, she responded,ȱȈNot a chance. We transferred most of that risk to other insurance companies.ȈȱAn arrangement by which an insurer that initially writes insurance transfers to another insurer part or all of the potential losses associated with such insurance is called A) hedging. B) speculating. C) reinsurance. D) loss avoidance.

c

Apex insurance company wrote a large number of property insurance policies in an area where earthquake losses could occur. When the president of Apex was asked she feared that a severe earthquake might put the company out of business, she responded, "Not a chance. We transferred most of that risk to other insurance companies." An arrangement by which the insurer that initially writes insurance transfers to other insurer part of all of the potential losses associated with such an insurance is called a. hedging b. speculating c. reinsurance d. loss avoidance

c

As an alternative to the Original Medicare Plan, beneficiaries can elect to enroll in private health insurance plans that cover all services that the Original Medicare Plan covers except hospice care. These private health insurance plans that are an alternative to the Original Medicare Plan are called A) Medigap Insurance Programs. B) PACE Programs. C) Medicare Advantage Plans. D) Medicare Prescription Drug Plans.

c

Bev lives in the suburbs and works downtown. She drives to work, and her most direct route to work would require her to pass through an area where carjacking and drive-by-shootings are common. Bev does not drive through this area. Instead she uses a route which adds ten minutes to her commute. Which risk management technique is Bev using with respect to the risk of injury while driving through a dangerous area? a. non-insurance transfer b. avoidance c. passive retention d. loss reduction

c

Cal was just hired as a XYZ Company's first risk manager. Cal would like to employ the risk management process. The first step in the process Cal should follow is to a. evaluate potential losses faced by XYZ company b. formulate a treatment plan for XYZ Company's loss exposures c. identify the potential losses faced by XYZ Company d. implement and and administer a risk management plan for XYZ Company

c

Cathy's car hit a patch of ice on the road. The car skidded off the road and hit a tree. The presence of ice on the road is best described as a a. peril b. subjective risk c. physical hazard d. indirect loss

c

Cathyȇs car hit a patch of ice on the road. The car skidded off the road and hit a tree. The presence of ice on the road is best described as a(n) A) peril. B) subjective risk. C) physical hazard. D) indirect loss.

c

Dense fog that increases the chance of an automobile accident is an example of a A) speculative risk. B) peril. C) physical hazard. D) moral hazard.

c

Dense fog that increases the chance of an automobile accident is an example of a a. speculative risk b. peril c. physical hazard d. moral hazard

c

Factors a risk manager must consider in selecting an insurer include which of the following 1. the availability of risk management services 2. the financial strength of the insurer a. 1 only b. 2 only c. both 1 and 2 d. neither 1 or 2

c

Faking an accident to collect insurance proceeds is an example of a A) physical hazard. B) objective risk. C) moral hazard. D) attitudinal hazard.

c

Faking an accident to collect insurance proceeds is an example of a a. physical hazard b. objective risk c. moral hazard d. attitudinal hazard

c

From the standpoint of the insurer, all of the following are characteristics of an ideally insurable risk EXCEPT A) The loss must be unintentional. B) The chance of loss must be calculable. C) The loss must be indeterminable. D) The loss must be measurable

c

From the standpoint of the insurer, all of the following are characteristics of an ideally insurable risk EXCEPT a. the loss must be unintentional b. it eliminates intentional losses c. the loss must be indeterminable d. the loss must be measurable

c

In addition to marketing life insurance, life insurers typically sell which of the following products? I. Retirement annuities II. Disability income insurance A) I only B) II only C) both I and II D) neither I nor II

c

LMN Insurance markets homeowners insurance. The LMN homeowners policy combines property and casualty insurance in the same contract. Insurance policies combining property and casualty coverage in the same contract are called A) mono-line policies. B) multi-year policies. C) multiple-line policies. D) manuscript policies.

c

LMN Insurance markets homeowners insurance. The LMN homeowners policy combines property and casualty coverage in the same contract are called a. mono-line policies b. multi-year policies c. multiple-line policies d. manuscript policies

c

Loss control includes which of the following? 1. Loss Reduction 2. Loss Prevention a. 1 only b. 2 only c. both 1 and 2 d. neither 1 or 2

c

Loss control includes which of the following? I. Loss reduction II. Loss prevention A) I only B) II only C) both I and II D) neither I nor II

c

Marion owns substantial financial assets. She was surprised that she qualified for Social Security retirement benefits when she retired because of her high investment income. What characteristic of social insurance programs is Marion overlooking with respect to her Social Security retirement benefits? A) Social insurance programs are financially self-supporting. B) Social insurance program benefits are loosely related to earnings. C) Social insurance benefits are not means tested. D) Full funding of social insurance programs is unnecessary.

c

Pre-loss objectives of risk management include which of the following 1. preparing for potential losses in the most economical way 2. reduction of anxiety a. 1 only b. 2 only c. both 1 and 2 d. neither 1 or 2

c

Reasons why market, financial, and production risks are often uninsurable include which of the following? I. The potential to produce a catastrophic loss is great. II. The chance of loss cannot be accurately estimated. A) I only B) II only C) both I and II D) neither I nor II

c

Reasons why market, financial, and production risks are often uninsurable include which of the following? 1. The potential to produce a catastrophic loss is great 2. The chance of loss cannot be accurately measured a. 1 only b. 2 only c. both 1 and 2 d. neither 1 or 2

c

Ryan decided to review his personal risk management program. His car is 10 years old, and he would receive little money from his insurer if the car was damaged or destroyed. Ryan decided to drop the physical damage insurance of his car. From a risk management perspective, dropping the physical damage insurance of the car is best described as a. increasing the use of avoidance in the risk management program b. increasing the use of non-insurance transfer in the risk management program c. increasing the use of retention in the risk management program d. increasing the use of risk control in the risk management program

c

The requirement that losses should be accidental and unintentional in order to be insurable results in which of the following? I. Decrease in moral hazard II. More accurate prediction of future losses A) I only B) II only C) both I and II D) neither I nor II

c

The requirement that losses should be accidental and unintentional in order to be insurable results in which of the following 1. Decrease in moral hazard 2. More accurate prediction of future losses a. 1 only b. 2 only c. both 1 and 2 d. neither 1 or 2

c

The term "multiple-line insurance" is used to describe a type of insurance that combines which of the following coverages into a single contract? a. workers compensation and health insurance b. life and disability insurance c. property and casualty insurance d. pensions and annuities

c

The termȱȈmultiple-line insuranceȈȱis used to describe a type of insurance that combines which of the following coverages into a single contract? A) workers compensation and health insurance B) life and disability insurance C) property and casualty insurance D) pensions and annuities

c

To better understand her company's operations, a risk manager asked a production manager to draw a diagram tracing the steps in the production and distribution of the company's products. Such a diagram, which is useful in risk identification, is called a a. financial statement b. risk management matrix c. flowchart d. risk management audit

c

Traditionally, risk has been defined as A) any situation in which the probability of loss is one. B) any situation in which the probability of loss is zero. C) uncertainty concerning the occurrence of loss. D) the probability of a loss occurring.

c

Traditionally, risk has been defined as a. any situation in which the probability of loss is one b. any situation in which the probability of loss is zero c. uncertainty concerning the occurrence of loss d. the probability of a loss occuring

c

Which of the following are often consequences of long-term disability 1. Continuing medical expenses 2. Loss of reduction of employee benefits a. 1 only b. 2 only c. both 1 and 2 d. neither 1 or 2

c

Which of the following are often consequences of long-term disability? I. Continuing medical expenses II. Loss or reduction of employee benefits A) I only B) II only C) both I and II D) neither I nor II

c

Which of the following is a reason why premature death death may result in economic insecurity 1. Additional expenses associated with death may be incurred 2. The income of the deceased person's family may be inadequate to meet its basic needs a. 1 only b. 2 only c. both 1 and 2 d. neither 1 or 2

c

Which of the following is a reason why premature death may result in economic insecurity? I. Additional expenses associated with death may be incurred. II. The income of the deceased personȇs family may be inadequate to meet its basic needs. A) I only B) II only C) both I and II D) neither I nor II

c

Which of the following is implied by the requirement that a loss should be determinable and measurable to be insurable? I. The loss must be definite as to place. II. The loss must be definite as to amount. A) I only B) II only C) both I and II D) neither I nor II

c

Which of the following is implied by the requirement that a loss should be determinable and measurable to be insurable? 1. The loss must be definite as to place 2. The loss must be definite as to amount a. 1 only b. 2 only c. both 1 and 2 d. neither 1 or 2

c

Which of the following is least likely to occur during a "hard" insurance market period? a. difficulty in obtaining insurance b. tightening underwriting standards c. high insurer profits d. increasing premiums

c

Which of the following statements about a priori probabilities is correct? A) They are subjective probabilities based on ambiguity in the way probability is perceived. B) They are subjective probabilities that may vary among individuals because of factors such as age, gender, education, and the use of alcohol. C) They are objective probabilities that can be determined by deductive reasoning. D) They are objective probabilities that can be determined by subjective reasoning.

c

Which of the following statements about a priori probabilities is correct? a. they are subjective probabilities based on ambiguity in the way in the way probability is perceived b. they are subjective probabilities that may vary among individuals because of factors such as age, gender, education, and the use of alcohol c. they are objective probabilities that can be determined by deductive reasoning d. they are objective probabilities that can be determined by subjective reasoning

c

Which of the following statements about speculative risks is true? A) They are almost always insurable by private insurers. B) They are more easily predictable than pure risks. C) They may benefit society even though a loss occurs. D) They involve only a chance of loss.

c

Which of the following statements about speculative risks is true? a. they are almost always insurable by private owners b. they are more easily predictable than pure risks c. they may benefit society even though a loss occurs d. they involve a chance of loss

c

Which of the following statements about the financing of unemployment insurance benefits is true? A) Each state maintains its own trust fund at the state level, and the state pays qualified beneficiaries from the trust fund the state administers. B) While experience rating is used for other types of insurance, it is not used in unemployment insurance. C) Part of an employer's contribution is used for administrative expenses, to fund extended benefits, and to maintain a loan fund that can be drawn upon if the state's account is depleted. D) Most state programs are financed primarily by payroll taxes paid by employees.

c

Which of the following statements about the insurance industry as a source of investment funds is (are) true? I. These funds result in a lower cost of capital than would exist in the absence of insurance. II. These funds tend to promote economic growth and full employment. A) I only B) II only C) both I and II D) neither I nor II

c

Which of the following statements about the insurance industry as a source of investments are true? 1. These funds result in a lower cost of capital than would exist in the absence of insurance 2. These funds tend to promote economic growth and full employment a. 1 only b. 2 only c. both 1 and 2 d. neither 1 or 2

c

Which of the following statements about unemployment insurance benefits is true? A) Benefits are a flat amount regardless of a worker's previous wages. B) Because of federal legislation, the maximum benefit is the same in all states. C) Most states pay regular benefits for a maximum duration of 26 weeks. D) Under the extended benefits program, the federal government continues benefit payments for up to 3 years for workers who have exhausted their regular benefits.

c

All of the following are considered financial risks EXCEPT A) the decline in the value of a bond portfolio because of rising interest rates. B) increased cost of production because of rising commodity prices. C) loss of money because of adverse movements in currency exchange rates. D) destruction of a production facility caused by an explosion.

d

All of the following are considered financial risks EXCEPT a. the decline in value of a bond portfolio because of rising interest rates b. increased cost of production because of rising commodity prices c. loss of money because of adverse movements in currency exchange rates d. destruction of a production facility caused by an explosion

d

All of the following are examples of personal risks EXCEPT A) poor health. B) unemployment. C) premature death. D) flood.

d

All of the following are examples of personal risks EXCEPT a. poor health b. unemployment c. premature death d. flood

d

All of the following are social costs associated with insurance EXCEPT A) insurance company operating expenses. B) fraudulent claims. C) inflated claims. D) increased cost of capital.

d

All of the following are social costs associated with insurance EXCEPT a. insurance company operating expenses b. fraudulent claims c. inflated claims d. increased cost of capital

d

All of the following statements about avoidance are true EXCEPT a. certain loss exposures are never acquired b. certain loss exposures may be abandoned c. the chance of loss for certain loss exposures be reduced to zero d. it can be used for any loss exposure facing a firm

d

Brenda identified all of the pure loss exposures her family faces. Then she analyzed these loss exposures, developed a plan to treat these risks, and implemented the plan. the process Brenda conducted is called a a. personal insurance programming b. personal estate planning c. personal financial planning d. personal risk management

d

Bronson Casualty Company sells casualty insurance only. Which of the following coverages could you purchase from Bronson Casualty Company? a. life insurance b. fire insurance c. marine insurance d. liability insurance

d

Bronson Casualty Company sells casualty insurance only.ȱȱWhich of the following coverages could you purchase from Bronson Casualty Company? A) life insurance B) fire insurance C) marine insurance D) liability insurance

d

Carelessness or indifference to a loss is an example of a. physical hazard b. objective probability c. moral hazard d. attitudinal hazard

d

From the insured's perspective, the use of deductibles insurance contracts is an example of a. risk transfer b. loss control c. risk avoidance d. risk retention

d

From the insuredȇs perspective, the use of deductibles in insurance contracts is an example of A) risk transfer. B) loss control. C) risk avoidance. D) risk retention.

d

JKL Insurance Company estimates that 14 out of every 100 homeowners it insures will file a claim each year. Last year, JKL insured 200 homeowners. According to the law of large numbers, what should happen if JKL insures 2,000 homeowners this year? A) The total number of claims filed by JKL policyowners should decrease. B) The total dollar value of claims will decrease. C) The average size of loss will decline in value. D) The actual results will more closely approach the expected results.

d

JKL insurance company estimates that 14 out of every 100 homeowners it insures will file a claim each year. Last year, JKL insured 200 homeowner. According to the law of large numbers, what should happen if JKL insures 2,000 homeowners this year? a. the total number of claims filed by JKL policy owners should increase b. the total dollar value of claims will decrease c. the average size of loss will decline in value d. the actual results will more closely approach the expected results

d

Jenna opened a successful restaurant. One night, after the restaurant had closed, a fire started when the electrical system malfunctioned. In addition to the physical damage to the restaurant, Jenna also lost profits that could have been earned while the restaurant was closed for repairs. The lost profits are an example of A) direct loss. B) nondiversifiable risk. C) speculative risk. D) indirect loss.

d

Jenna opened a successful restaurant. One night, after the restaurant had closed, a fire started when the electrical system malfunctioned. In addition to the physical damage to the restaurant, Jenna also lost profits that could have been eared while the restaurant was closed for repairs. The lost profits are examples of a. direct loss b. non-diversifiable risk c. speculative risk d. indirect loss

d

Jim and Paula Franklin started a dry cleaning business. The business may be succesful or it may fail. The type of risk that is present when either a profit or loss could occur is called a. pure risk b. subjective risk c. non-diversifiable risk d. speculative risk

d

Mark owns a 1998 sedan. The last time Mark renewed his auto insurance, he decided to drop the physical damage insurance on this vehicle. How is Mark dealing with the auto physical damage exposure in his personal risk management program? a. risk transfer b. passive retention c. avoidance d. planned retention

d

Members of Mid-SOuth Petroleum Distributors, a trade group, had trouble obtaining affordable pollution liability insurance. The members formed a group captive that is exempt from many state laws that apply to other insurers. This group captive is called a a. reinsurance pool b. Lloyd's association c. alien insurer d. risk retention group

d

One insured status under Social Security requires you to have earned at least six credits duing the last 13 calendar quarters ending with the quarter of death, disability, or entitlement to retirement benefits. This insured status is A) disability insured. B) temporarily insured. C) fully insured. D) currently insured.

d

Prior to the passage of workers compensation laws, all of the following common law defenses could be used by employers to avoid financial responsibility for work-related injuries EXCEPT A) contributory negligence. B) the fellow-servant doctrine. C) assumption of the risk. D) joint and several liability.

d

Some characteristics of the judicial system and regulatory environment increase the frequency and severity of loss. This hazard is called A) moral hazard. B) physical hazard. C) attitudinal hazard. D) legal hazard.

d

Some characteristics of the judicial system and regulatory environment increase the frequency and the severity of loss. The hazard is called a. moral hazard b. physical hazard c. attitudinal hazard d. legal hazard

d

Taylor Tobacco Company is concerned that the company may be held liable in a court of law and ordered to pay a large damage award. The characteristics of the judicial system that increase the frequency and severity of losses are known as A) moral hazard. B) particular risk. C) speculative risk. D) legal hazard.

d

Taylor tobacco company is concerned that the company may be held liable in court of law and ordered to pay a large damage award. The characteristics of the judicial system that increase the frequency and severity of losses are known as a. moral hazard b. particular risk c. speculative risk d. legal hazard

d

The extra expense incurred by a business to stay in operation following a fire is an example of a(n) A) fundamental risk. B) speculative risk. C) direct loss. D) indirect loss

d

The extra expense incurred by a business to stay in operation following a fire is an example of a a. fundamental risk b. speculative risk c. direct loss d. indirect loss

d

The tendency for unhealthy people to seek life or health insurance at standard rates is an example of A) moral hazard. B) fundamental risk. C) attitudinal hazard. D) adverse selection.

d

The tendency for unhealthy people to seek life or health insurance at standard rates is an example of a. moral hazard b. fundamental risk c. attitudinal hazard d. adverse selection

d

Under one of the Medicare Advantage Plans, beneficiaries can receive care at a discounted rate by using health care providers that belong to the plan network. Care outside the network is also covered, but beneficiaries must pay higher out-of-pocket costs. This type of Medicare Advantage Plan is a A) Point of Service (POS) plan. B) Medical Savings Account (MSA) plan. C) Health Maintenance Organization (HMO) plan. D) Preferred Provider Organization (PPO) plan.

d

What is Acme's maximum possible loss associated with a single hurricane? a. $0 million b. $50 million c. $100 million d. $150 million

d

Which of the following is a current problem with workers compensation programs? A) declining involvement of lawyers in workers compensation claims B) exclusion of deaths and injuries attributable to terrorism C) reduced benefits for occupational illness D) rising share of medical costs as a share of total benefits

d

Which of the following is a result of adverse selection? A) The insurerȇs financial results will be substantially improved. B) Persons most likely to have losses are also most likely to seek insurance at standard rates. C) It is unnecessary for the insurance company to use underwriting. D) Insurance can be written only by the federal government.

d

Which of the following statements about financial risk is (are) true? I. Enterprise risk does not include financial risk. II. Financial risk is easily addressed through the purchase of insurance. A) I only B) II only C) both I and II D) neither I nor II

d

Which of the following types of loss exposures are best handled by the use of avoidance? a. low-frequency, low-severity b.low-frequency, high-severity c. high-frequency, low-severity d. high-frequency, high-severity

d

Which of the following types of risks is normally uninsurable by private insurers? A) personal risks B) property risks C) liability risks D) market risks

d

Examples of Excluded Perils

flood, earth movement, and nuclear radiation or radioactive contamination

To better understand her companyȇs operations, a risk manager asked a production manager to draw a diagram tracing the steps in the production and distribution of the companyȇs products. Such a diagram, which is useful in risk identification, is called a

flowchart.

Low-frequency, low-severity loss exposures are best handled by

retention

Declarations Page

statements that provide information about the particular property or activity to be insured Can usually be found on the first page of the policy In property insurance, it contains name of the insured, location of property, period of protection, amount of insurance, premium and deductible information

In addition to the named insured, all of the following are insureds under the Commercial General Liability Policy EXCEPT

suppliers of products to the named insured's business

Sarah purchased a Personal Auto Policy with liability limits of 50/100/25. Sarah ran a stop sign and hit a van. The van sustained $15,000 in damages. The following bodily injuries were suffered by passengers in the van: Passenger #1, $15,000; Passenger #2, $60,000; and Passenger #3, $10,000. Sarah sustained $5,000 in medical expenses, and Sarah's car sustained $10,000 in damages. How much will Sarah's insurer pay under Part A: Liability Coverage?

$90,000

All of the following persons are eligible for a benefit under the retirement portion of the Social Security program EXCEPT A) a divorced spouse, age 55, who was married to the retired worker for 6 years. B) a retired worker's 63 year-old spouse who is no longer caring for children. C) a retired worker's 52 year-old spouse who is caring for a 12 year-old daughter of the retired worker. D) a retired worker's unmarried 20 year-old son who has been severely disabled because of an automobile accident while he was in elementary school.

a

As of this year, Brad, age 50, has 40 credits under the Social Security program. These credits were all earned in the last 10 years. What is Brad's insured status under the program? A) He is currently and fully insured. B) He is currently insured, but not fully insured. C) He is fully insured, but not currently insured. D) He is neither currently insured nor fully insured.

a

Which of the following statements about the characteristics of social insurance programs is true? A) They are designed to provide a floor of income with respect to the risks that are covered. B) The method of determining benefits is determined by insurance contractual provisions. C) Participants are required to satisfy a means test to receive benefits. D) With few exceptions, participation in social insurance programs is voluntary.

a

All of the following persons are eligible for survivor benefits under Social Security EXCEPT A) dependent parents age 62 or older. B) unmarried children between the ages of 18 and 22 who are attending college. C) a surviving spouse age 60 or older. D) a surviving spouse with eligible children younger than age 16.

b

All of the following statements about the Medicare Prescription Drug Plan are true EXCEPT A) Medicare prescription drug coverage is available to all Medicare beneficiaries. B) Medicare prescription drug coverage is funded exclusively through the Part A payroll tax. C) The Medicare prescription drug program provides financial help for beneficiaries with limited income and financial resources. D) In addition to the initial deductible, there is a coverage gap where the beneficiary must pay the entire cost of prescription drugs.

b

Dale, age 65, was dismayed to learn about all of the deductibles, co-pays, limits, and exclusions in the Medicare program. Dale bought a type of health insurance specifically designed to supplement Medicare, and selected his coverage from among 10 standard policies that private insurers offer. What type of health insurance did Dale purchase? A) long-term care insurance B) Medigap insurance C) major medical insurance D) viatical insurance

b

Ken purchased a PAP with liability limits of 100/300/50, medical payments coverage, and collision coverage. Ken fell asleep while driving late at night. He crossed the center line and hit a car approaching from the other direction. The following losses occurred: —The driver of the other car suffered $30,000 in bodily injuries. —Ken's car sustained $5,000 in damages. —Ken incurred $5,000 in medical expenses. —The car that Ken hit was a total loss. Which of Ken's Personal Auto Policy (PAP) coverages will cover the other driver's medical expenses?

bodily injury liability

In addition to most persons over age 65, which of the following persons is (are) eligible for Medicare benefits? I. Persons under age 65 who have been entitled to Social Security disability benefits for at least 24 months II. Persons under 65 who need long-term kidney dialysis treatment or a kidney transplant

both

Objectives of unemployment insurance include which of the following? I. To help unemployed workers find jobs. II. To encourage employers to stabilize employment.

both

Which of the following statements about the coverage of health care services under Part A of Medicare is (are) true? I. Services in the patient's home are covered if the patient requires skilled care and meets certain conditions. II. Hospice care is available for beneficiaries with a terminal illness.

both

In order to receive unemployment insurance benefits, an unemployed worker must meet all of the following eligibility requirements EXCEPT A) have qualifying wages and employment. B) be able to work. C) be actively seeking work. D) satisfy a 6-month waiting period.

d

Duties of an insured after a collision loss covered under the PAP include which of the following? I. Take reasonable steps to protect the vehicle from further damage. II. Admit fault if the insured believes he or she caused the collision.

I only

Which of the following statements about participation in the Social Security program is (are) true? I. Most private sector employees are covered under the Social Security program. II. Each employee has the right to "opt out" of the Social Security program if the employee does not wish to participate in the program.

I only

Which of the following statements about problems and issues in workers compensation insurance is (are) true? I. The cost of medical care under workers compensation continues to rise. II. Enactment of workers compensation statutes has eliminated litigation with respect to work-related illnesses and injuries.

I only

Which of the following statements about the financing of OASDI and Medicare is (are) true? I. There is a maximum taxable wage base for OASDI. II. There is a maximum taxable wage base for Part A of Medicare.

I only

Which of the following statements about the payment of defense costs by the PAP is (are) true? I. They are paid in addition to the policy limits. II. They are payable even after the limit of liability is exhausted

I only

Which of the following persons is (are) insured under the uninsured motorists coverage of the PAP? I. A pedestrian struck by a covered auto if he or she has no insurance to pay medical expenses II. The spouse of a named insured who is killed by an uninsured motorist

II only

Which of the following situations would be covered under the liability section of the PAP? I. A mechanic is sued by a pedestrian who is injured when the mechanic has an accident while road testing the insured's auto. II. The daughter of the named insured is sued after she has an accident when a new friend she just met at a campus hangout lets her drive his car.

II only

Which of the following statements about the role of physicians with respect to Medicare claims is (are) true? I. Physicians who do not accept an assignment of Medicare claims can charge as much as 200 percent of the Medicare-approved fee. II. Physicians who accept assignment agree to accept the Medicare-approved amount as payment in full.

II only

Larry has $25,000 of bodily injury liability coverage under his PAP. This limit is the minimum amount required by his state to be considered financially responsible. While on a vacation, Larry visited a neighboring state which has a minimum financial responsibility limit of $50,000 for bodily injury. Which of the following statements describes the situation for Larry while he was in the neighboring state?

Larry's policy automatically provided $50,000 of liability coverage.

Which of the following statements about the liability limits of the PAP is (are) true? I. The policy can be written with split limits of liability. II. Prejudgment interest is considered part of the damage award and is subject to the policy limit

both I and II

Which of the following statements about the uninsured motorists coverage of the PAP is true?

The coverage applies only if the uninsured motorist is legally liable

Which of the following is considered to be a collision loss under Part D (coverage for damage to your auto) of the PAP?

The covered auto is damaged when it slid off an icy road and hit a fence

All of the following statements about the termination provisions of the PAP are true EXCEPT

The insurer can cancel the policy after it has been in force for 60 days only if the insured has three or more traffic violations.

EFG Company experienced a reduced demand for its products during a recession. EFG managers were considering laying off some workers when the personnel director said, "Let's not lay off these workers. If we do, our unemployment insurance premiums will increase. The state considers employment stability when determining our premium." Considering the firm's employment record when determining the rate to charge for unemployment insurance is called A) experience rating. B) class rating. C) schedule rating. D) retrospective rating.

a

Which of the following statements is true with regard to the Medicare program? A) While Social Security old-age benefits are available at age 62, Medicare Part A benefits are not available, in most cases, until age 65. B) If an individual delays collecting Social Security old-age benefits until after the full retirement age, he or she is not eligible for Medicare benefits until he or she begins to collect old-age benefits. C) Part B of Medicare is funded through a payroll tax. D) Beneficiaries must pay a monthly premium for Medicare Part A coverage

a

Which of the following would help to reduce the long-range OASDI actuarial deficit ? A) increasing the payroll tax rate B) increasing benefits C) reducing the taxable wage base D) increasing the cost of living adjustment

a

Frank is doing some life insurance planning. A financial advisor said, "be sure to consider Social Security when examining sources of funds available for family support if you die." The financial advisor was referring to which Social Security benefit? A) retirement benefits B) survivor benefits C) disability benefits D) health insurance benefits

b

Tony is 48 years old. He earned the maximum taxable wage base under Social Security annually between the time when he was 26 and when he was 40. At age 40, he dropped out of the workforce to care for children as his wife had a high-paying job. Which statement is true regarding Tony's insured status under Social Security? A) Tony is fully insured and currently insured. B) Tony is fully insured, but not currently insured. C) Tony is currently insured, but not fully insured. D) Tony is neither fully insured nor is he currently insured.

b

Which of the following statements about benefits provided by workers compensation programs is true? A) Occupational disability and nonoccupational disability are covered. B) Death benefits and benefits to eligible survivors are payable if a worker is killed on the job. C) Medical benefits are usually subject to deductibles, coinsurance, and numerous exclusions. D) Retirement benefits are payable to retired workers through workers compensation.

b

Which of the following statements about the financing of the Social Security program is (are) true? I. Although the self-employed pay an OASDI tax rate that is twice the employee rate, they are allowed certain deductions that reduce the effective tax rate. II. The earnings base on which OASDI taxes are paid increases annually based on changes in average wages in the national economy.

both

Which of the following statements about the problems and issues of unemployment insurance programs is (are) true? I. Many state programs are inadequately financed. II. Only a small portion of the total unemployed receive benefits at any time.

both

Which of the following statements is (are) true with respect to the Social Security earnings test? I. The earnings test does not apply to dividends, interest, and rental income. II. The earnings test does not apply to work earnings for those individuals who have reached the full retirement age under Social Security.

both

The insurance company's options for settling a collision loss to a covered auto under the PAP include which of the following? I. Pay the loss in money. II. Repair or replace the damaged auto

both I and II

Which of the following persons is (are) covered for liability insurance under the PAP? I. a family member who drives a covered auto II. a family member who occasionally drives a friend's auto

both I and II

All of the following statements about the Medicare prescription drug benefit are true EXCEPT A) Beneficiaries are required to pay a monthly premium for the benefit. B) The plan provides protection against catastrophic prescription drug expenditures. C) Coverage is limited to when a beneficiary is hospitalized or a resident of rest home. D) The plan has a coverage gap before a beneficiary has coverage for catastrophic costs

c

Which of the following statements about workers compensation programs is true? A) Injured workers must prove the employer was responsible for their injuries to collect benefits. B) Workers compensation disability income benefits (amount and duration) are uniform from state to state. C) Options available for complying with the law may include one or more of the following: self-insurance, private insurance, or insurance through a state fund. D) Most states have elective laws whereby the employer can choose whether or not to provide workers compensation coverage to employees.

c

Marco, a risk manager in California, is interviewing for a new position in the state of Washington. When Marco asked about methods of providing workers compensation, the answer surprised him. In Washington, employers can self-insure the risk or obtain coverage through a state fund. Private insurers do not market workers compensation insurance in Washington. Washington uses a A) competitive state fund. B) guaranty fund. C) reinsurance facility. D) monopoly state fund.

d

Under Part A of Medicare, how is reimbursement for inpatient hospital care determined? A) The amount billed to Medicare by the hospital is the amount Medicare pays. B) Eighty percent of the hospital's regular charge for care is paid. C) The hospital's usual, reasonable, and customary charge is paid. D) Hospital care is classified into diagnosis-related groups (DRGs) and the amount reimbursed depends on the DRG classification of the care delivered.

d

Which of the following statements about disability benefits under Social Security is true? A) There is a one-year waiting period before benefits are payable. B) Benefits are payable as long as the disabled worker is unable to perform his or her regular occupation. C) Benefits are payable only if the condition causing the disability is expected to result in death prior to age 65. D) In addition to the disabled worker receiving the benefits, benefits can also be paid to eligible dependents.

d

Which of the following statements about the full retirement age under the Social Security program is true? A) The current retirement age for full benefits is age 62. B) Beginning in the year 2020, the retirement age will be gradually increased to age 72. C) Early retirement with reduced benefits is allowed as early as age 55. D) Delayed retirement beyond the age for full benefits increases the amount of the monthly benefit.

d

The 2005 PAP states that the insurer has no duty to provide coverage if the insured fails to comply with certain listed duties. In practice, however, the insurer is only relieved of its duty to provide coverage if

failure to comply

In addition to providing coverage in the United States, its territories and possessions, and Puerto Rico, where else does the PAP provide coverage?

in Canada

The purpose of the Miscellaneous-Type Vehicle Endorsement to the PAP is to

insure motorcycles, mopeds, motor scooters, and similar vehicles

Which of the following statements about a worker's primary insurance amount is (are) true? I. It is the monthly amount paid to a worker who elects early retirement at age 62. II. It is equal to 50 percent of the worker's final average pay that was subject to Social Security taxes.

neither

Which of the following statements concerning Medicare Prescription Drug Plans is (are) true? I. The monthly premium is based on the health status of the beneficiary and the number of prescriptions the beneficiary needs. II. After the beneficiary has paid an initial deductible, Medicare pays the entire cost of all prescription drugs the beneficiary needs.

neither

Which of the following statements is (are) true with respect to the collision damage waiver on rented cars? I. It is inexpensive and is provided at no charge by most rental car companies. II. It waives the renter's liability for bodily injury liability arising out of use of the rented auto.

neither I nor II

Which statement concerning towing and labor coverage under the PAP is (are) true? I. There's no coverage for towing if the auto breaks down—towing is only covered if the auto needs to be towed after a collision has occurred. II. Towing and labor coverage pays for repairs at a service station or garage.

neither I or II

A vehicle is considered a constructive total loss when

the repair cost exceeds the actual cash value.


Related study sets

CHAPTER 18- Taxes Affecting Real Estate

View Set

Quiz 6: Routing Protocols (Chapter 10)

View Set